Pharmacology Review

Ace your homework & exams now with Quizwiz!

Amphotericin B MoA

-Binds ergosterol (unique to fungi); forms membrane pores that allow leakage of electrolytes. -Amphotericin "tears" holes in the fungal membrane by forming pores

H2 Blockers

-Block histamine-induced gastric acid secretion from the parietal cells of the gastric mucosa (lining of the stomach) -Used to treat GERD and ulcers Examples: (-tidine) cimetidine (Tagamet) famotidine (Pepcid) ranitidine (Zantac)

Progestins (mechanism, use)

-Mechanism: bind progesterone receptors, decrease growth and increase vascularization of the endometrium -Used in oral contraceptives and to treat endometrial cancer, abnormal uterine bleeding.

Etanercept

-Mechanism: fusion protein (receptor for TNF-α + IgG1 Fc), produced by recombinant DNA. Etanercept is a TNF decoy receptor. -Use: rheumatoid arthritis, psoriasis, ankylosing spondylitis

Hydroxyurea (mechanism, use, toxicity)

-Mechanism: inhibits ribonucleotide reductase decreasing DNA Synthesis (S-phase specific). -Use: melanoma, CML, sickle cell disease ( HbF). -Toxicity: severe myelosuppression, GI upset.

Drugs for simple seizure

-Carbamazepine is 1st choice -All other epilepsy drugs can be used except for ethosuxamide and the benzodiazepines

Danazol (mechanism, use, toxicity)

-Mechanism: synthetic androgen that acts as partial agonist at androgen receptors. -Use: endometriosis, hereditary angioedema. -Toxicity: weight gain, edema, acne, hirsutism, masculinization, decreased HDL levels, hepatotoxicity.

a. A history of peptic ulcers

A patient is receiving instructions regarding the use of caffeine. The nurse shares that caffeine should be used with caution if which of these conditions is present? a. A history of peptic ulcers b. Asthma c. Migraine headaches d. A history of kidney stones

B. Ventricular arrhythmias

A patient has been prescribed lidocaine (Xylocaine). What does the nurse understand as the reason for this medication order? A. Bradycardia B. Ventricular arrhythmias C. Atrial arrhythmias D. Heart block

a. Hypertension

A patient has been taking selegiline (Eldepryl), 20 mg per day for 1 month. Today, during his office visit, he tells the nurse that he forgot and had a beer with dinner last evening, and "felt awful." What did the patient most likely experience? a. Hypertension b. Gastrointestinal upset c. Hypotension d. Urinary discomfort

-statin

HMG-CoA reductase inhibitor

Nicotinic Acid (Niacin, B3)

Antihyperlipidemic - flushing, itching, GI

c. Urge incontinence

The nurse is reviewing a patient's medication history and notes that the patient is taking the cholinergic blocker tolterodine (Detrol). Which is an indication for this medication? a. Irritable bowel disease b. Reduction of secretions preoperatively c. Urge incontinence d. Induction of mydriasis

c. ketoconazole (Nizoral)

The nurse is reviewing the medication orders for a patient who will be taking an H2 antagonist. Which drug may have an interaction if taken along with the H2 antagonist? a. tetracycline (Doryx) b. ranitidine (Zantac) c. ketoconazole (Nizoral) d. ibuprofen (Motrin)

List the dihydropyridine calcium channel blockers

Amlodipine, clevidipine, nicardipine, nifedipine, nimodipine -Act on vascular smooth muscle

C atenolol (Tenormin).

An example of a cardioselective beta blocker includes A propranolol (Inderal). B labetalol (Normodyne). C atenolol (Tenormin). D sotalol (Betapace).

5-fluorouracil (5-FU) mechanism

-Pyrimidine analog bioactivated to 5F-dUMP, which covalently complexes folic acid. -This complex inhibits thymidylate synthase decreasing dTMP and decreasing DNA synthesis.

ANTIANXIETY Nitrous Oxide/Oxygen Sedation

-Rapid onset & rapid recovery LOW solubility in blood -Excreted by exhalation o No biotransformation(metabolizing) inhaled and exhaled in same form

Terazosin

-Selective α1-blocker -Uses: urinary symptoms of BPH; -Hypertension -Toxicity: 1st-dose orthostatic hypotension, dizziness, headache

Hormone replacement therapy

-Used for relief or prevention of menopausal symptoms (e.g., hot flashes, vaginal atrophy), osteoporosis (increased estrogen, decreased osteoclast activity). -Unopposed estrogen replacement therapy increases risk of endometrial cancer, so progesterone is added. Possible increased cardiovascular risk.

Amphetamine & -Like Nursing Process

-assess for contraindications -monitor for SE's;report abnormals - -take before meals -avoid alcohol -use sugarless gum to relieve dry mouth -monitor wt q2w -do not abruptly d/c drug -read OTC drug labels -report tachycardia/palpitations - -monitor children for tourettes

ANTIHYPERTENSIVE Angiotensin Receptor Blockers (ARBS)

-attach to angiotensin II - block the effect of the angiotensin II -More specific than ACE Inhibitors - so fewer adverse reactions -Effectiveness reduced by NSAIDS

Anorexiants Nursing Process

-emphasize nutritious diet, exercise, and behavior modification. -don't rely on appetite suppressants. -for short-term use -Long term use frequently results in severe SE's nervousness, restlessness, irritability, insomnia, palpitations, HTN.

In order for a dentist to prescribe medicine two conditions must be met?

1 patient of record. 2 Dental condition.

Bronchoconstriction is mediated by

(1) inflammatory processes and (2) parasympathetic tone; therapy is directed at these 2 pathways

aspirin (ASA) Toxicity

(salicylism)

ANTIBIOTICS: Penicillins (-cillin)

-B-lactam ring -Narrow spectrum (less side effects) -bactericidal -Many bacteria are resistant (staphylococci infections) -Most common for allergic reaction -Most common antibiotic used in dentistry -Safe for pregnancy

Drugs for status epilepticus

-Benzodiazepines (diazepam, lorazepam) are first line for acute -Phenytoin is first line prophylaxis

Tacrolimus MoA

-Calcineurin inhibitor; binds FK506 binding protein (FKBP). -Blocks T-cell activation by preventing IL-2 transcription.

4th generation cephalosporins

-Cefepime -Use: gram-negative organisms, with activity against Pseudomonas and gram-positive organisms.

Fluticasone, budesonide

-Corticosteroids -Inhibit the synthesis of virtually all cytokines. Inactivate NF-κB, the transcription factor that induces production of TNF-α and other inflammatory agents. 1st-line therapy for chronic asthma.

Gastroesophageal reflux disease (GERD) Treatment:

-Decrease acid in stomach Histamine2 (H2) Blockers Proton pump inhibitors (PPIS) -Constricting cardiac sphincter GI stimulants -Relief of symptoms Antacids

Latanoprost as a glaucoma drug

-Decrease outflow of aqueous humor -Side effect: darkens color of iris (browning)

SSRIs use

-Depression, generalized anxiety disorder, panic disorder, OCD, bulimia, social phobias, PTSD.

Adverse reactions of aspirin:

-GI irritation contraindicated with peptic ulcer -Hypersensitivity - 15% of patients -Can cause Reye syndrome in children contraindicated for children -Contraindicated with warfarin (Coumadin) drug interaction & increased bleeding

Adverse reactions of ibuprofen and naproxen:

-GI irritation- contraindicated with peptic ulcer -Increased bleeding contraindicated with warfarin -Xerostomia

Febuxostat

-Gout -Inhibits xanthine oxidase

Pyrazinamide

-Mechanism uncertain. Pyrazinamide is a prodrug that is converted to the active compound pyrazinoic acid. -Use: Mycobacterium tuberculosis. -Toxicity: Hyperuricemia, hepatotoxicity.

Memantine

-Mechanism: NMDA receptor antagonist; helps prevent excitotoxicity (mediated by Ca2+). -Use: Alzheimer disease -Toxicity: dizziness, confusion, hallucinations

Tamoxifen (mechanism, use, toxicity)

-Mechanism: antagonist at breast; agonist at bone, uterus; -Used to treat and prevent recurrence of ER/PR ⊕ breast cancer; -Increased risk of thromboembolic events and endometrial cancer (as opposed to raloxifine)

Benztropine

-Mechanism: antimuscarinic; improves tremor and rigidity but has little effect on bradykinesia -Use: Parkinson disease therapy

Tolcapone

-Mechanism: blocks conversion of dopamine to DOPAC by inhibiting central COMT -Use: Parkinson disease therapy

Phentolamine

-Nonselective α-blocker -Give to patients on MAO inhibitors who eat tyramine containing foods -Toxicity: orthostatic hypotension, reflex tachycardia

Lasix

What is the common Trade name for furosemide?

Calcium Channel Blocker

What pharmacologic category does Norvasc belong to?

A. CBC

When assessing for potential toxicity to PTU, the nurse will monitor the patient for changes in which laboratory test? A. CBC B. BNP C. Serum electrolytes D. Renal function tests

a, b, c

When titrating intravenous nitroglycerin, which assessment findings does the nurse monitor? (Select all that apply.) A. Blood pressure B. Heart rate C. Chest pain D. Respiratory rate

B. insulin aspart (NovoLog)

Which is a rapid-acting insulin with an onset of action of less than 15 minutes? A. insulin glargine (Lantus) B. insulin aspart (NovoLog) C. regular insulin (Humulin R) D. insulin detemir (Levemir)

D Headache

Which is an adverse effect of bethanechol (Urecholine)? A Constipation B Hypertension C Tachycardia D Headache

B. Decreased heart rate

Which is an expected outcome associated with the administration of digoxin? A. Increased heart rate B. Decreased heart rate C. Decreased urinary output D. Increased blood pressure

A. clopidogrel

Which medication has antiplatelet properties? A. clopidogrel B. enoxaparin C. heparin D. alteplase

a. CD4 count

Which of the following laboratory tests best evaluates HIV disease? a. CD4 count b. Liver function studies c. Platelets d. Complete blood count

d. The indication for administration was achieved without serious side effects.

Which of the following would most accurately indicate that a therapeutic range for a medication had been reached? a. The medication was effective, but the patient experienced a lethal dysrhythmia. b. A pre-specified amount (in milligrams) was administered c. No serious adverse effects are experienced following administration. d. The indication for administration was achieved without serious side effects.

C. "I will increase the dark green leafy vegetables in my diet."

Which statement when made by the patient indicates deficient knowledge regarding warfarin? A. "I will avoid contact sports." B. "I will take my medication in the early evening each day." C. "I will increase the dark green leafy vegetables in my diet." D. "I will contact my physician if I develop excessive bruising."

C. 4 to 5 hours before travel

Which would be the most appropriate application time for a patient prescribed a scopolamine patch for motion sickness? A. At bedtime B. Every 4 hours as needed C. 4 to 5 hours before travel D. At the first sign of motion sickness

a. Hypothermia

You are precepting a new graduate nurse in the ICU. The two of you are caring for a patient who is receiving large volumes of crystalloid fluid as a result of shock. What would you teach the new nurse to monitor the patient for symptoms of? a. Hypothermia b. Bradycardia c. Coffee ground emesis d. Pain

Analgesics Safe for use:

acetaminophen (Tylenol)

acetaminophen is drug of choice for patients on

anticoagulants (warfarin) or with peptic ulcers

which drug is Reye's syndrome associated with?

aspirin.

the Pans does what to pupil size?

constrict. called myosis

Glycogenolysis

conversion of glucogen to glucose

List the non-dihydropyridine calcium channel blockers

diltiazem, verapamil -Act on the heart

disp

dispense

Excretion

drug can be excreted unchanged or as a metabolite (modified)

Distribution

drug distributed by blood plasma Factors -Organ size and blood flow -Drug solubility -Plasma protein binding power. -Free drug - exerts the effect of drug -Barriers -blood brain barrier, placental barrier

sig

label, instructions for use

What cholinergic drug is used to treat Xerostomia?

pilocarpine (salagen).

Anti-Tuberculosis drugs

rifampin isoniazid pyrazinamide ethambutol

Distribution is determined by what factors?

size of the organ, blood flow to the organ, solubility of the drug. plasma protein-binding capacity. presence of barriers.

The MOST important site for drug absorption of orally administered drugs is the

small intestine.

How many pregnancy categories are there?

they are A, B,C,D, and X

TI of>10 needed for

useful drug

What would Heroin be schedule as?

1, highest abuse potential. No accepted medical use; experimental use, only in research.

Q: TCAS are cOntraindicated with What?

A: epinephrine Hypertensive crisis

BOARD ALERT - do not use these drugs with herpes- will suppress immune system and worsen the condition as Herpes is a virus

ADRENOCORTICOSTEROID DRUGS

Irbesartan

ARB (-sartan)

Schedule IV

Abuse Potential less antianxiety -Xanax tramadol

Nonspecific depressant withdrawal

Anxiety, tremor, seizures, insomnia

Exposure to syphili

Benzanthine penicillin G

What effect do nitrates have on MVO2?

Decreased

Name the long acting insulins

Detemir, Glargine

Potency

Determined by the amount of a drug that is required to produce the desired effect.

1st generation H1 blockers

Diphenhydramine, dimenhydrinate, chlorpheniramine

Daclizumab, basiliximab toxicity

Edema, HTN, tremor

Why give a local anesthetic with vasoconstrictors?

Enhance local action - decrease bleeding, increase anesthesia by decreasing systemic concentration

Brand Name: Plavix

Generic Name: Clopidogrel

Weak bases are better absorbed when the ph is _____ than the pka

Greater

Levothyroxine

Hypothyroidism/ Pituitary TSH supression No effects on dental treatment.

Adalimumab, infliximab clinical use

IBD, rheumatoid arthritis, ankylosing spondylitis, psoriasis

Cocaine intoxication

Impaired judgment, pupillary dilation, hallucinations (including tactile), paranoid ideations, angina, sudden cardiac death. Treatment: α-blockers, benzodiazepines. β-blockers not recommended.

HMG-CoA reductase mechanism of action

Inhibit conversion of HMG- CoA to mevalonate, a cholesterol precursor; Decreased mortality in CAD patients

B. less sedative effect.

Loratadine (Claritin) has an advantage over traditional antihistamines such as diphenhydramine (Benadryl) in that loratadine has A. less risk of cardiac arrhythmias. B. less sedative effect. C. increased bronchodilating effects. D. less gastrointestinal upset.

Sertraline

Major Depressive Disorder/ OCD/ Panic Disorder/ PTSD/ Premenstrual Dysphoric Disorder/ Social Anxiety Disorder Dental considerations: Xerostomia, possible prolonged bleeding time

Natalizumab clinical use

Multiple sclerosis, Crohn disease

Bradycardia, hypotension

Name two common side effects of beta blockers:

List the atypical antipsychotics

Olanzapine, clozapine, quetiapine, risperidone, aripiprazole, ziprasidone

Denosumab clinical use

Osteoperosis; inhibits osteoclast maturation (mimics osteoprotegerin)

What coagulation pathway is affected by heparin?

PTT (intrinsic pathway)

Cholinergic

Parasympathetic

B. Miosis

Parasympathomimetic ophthalmic drugs such as pilocarpine (Pilocar) reduce intraocular pressure in the treatment of glaucoma by which mechanism of action? A. Mydriasis B. Miosis C. Decrease in drainage of aqueous humor D. Pupillary dilatation

Pregnant woman carrying group B strep

Penicillin G

-tropin

Pituitary hormone

Benzodiazepines withdrawal

Sleep disturbance, depression, rebound anxiety, seizure

Antidote for Digoxin toxicity

Slowly normalize K+, cardiac pacer, anti-digoxin Fab, Mg2+

Difference between tamofixen and raloxifene?

Tamoxifen has increased risk of endometrial carcinoma whereas raloxifene does not because it is an estrogen receptor antagonist in endometrial tissue

Zithromax, Z-Pak

Give two common Trade names for Azithromycin:

Fentanyl adverse effects

-CSA Schedule 2 -As for morphine (except histamine release) -Chest muscle rigidity if infused IV too fast -CI: As for morphine -Use w/ MAOIs (or 2 weeks afterwards)

OPIOID NARCOTIC ANALGESICS

Bind to receptors in the CNS •Pharmacologic Effects: -Analgesia -Sedation/euphoria Larger doses euphoria -Cough suppression -GI Effects • Decreases GI motility/contractions Used to treat diarrhea

Bismuth, sucralfate mechanism

Bind to ulcer base, providing physical protection and allowing HCO3- secretion to reestablish pH gradient in the mucus layer

-dronate

Bisphosphonate

Cephalosporin toxicity

-Hypersensitivity reactions, autoimmune hemolytic anemia, disulfiram-like reaction, vitamin K deficiency. -Exhibit cross-reactivity with penicillins. -Increased nephrotoxicity of aminoglycosides.

Order of loss with local anesthetic

1. pain 2. temp 3. touch 4. pressure

Rate of excretion of a drug

On what is the duration of action dependent?

Thyroid product

What is the pharmacologic category for Synthroid?

Beta-1 selective blocker

What is the pharmacologic category for atenolol?

Advantage of parenteral route?

injection results in fast absorption, which produces a rapid onset and more predictable response than oral administration.

Bisphosphonates =

osteonecrosis of the mandible

pKa

related to onset of action -lower = better absorbed

lipid solubility

related to potency -more lipid soluble = more potent

if redistribution occurs between specific sties and nonspecific sites, a drugs action will be terminated? true false

yeah true.

-zosin

α1-antagonist

Tamsulosin

α1-antagonist used to treat BPH by inhibiting smooth muscle contraction. Selective for α1A,D receptors (found on prostate) vs. vascular α1B receptors

-olol

β-blocker

-terol

β2-agonist

Heparin-induced thrombocytopenia (HIT)

Development of IgG antibodies against heparin- bound platelet factor 4 (PF4). Antibody-heparin-PF4 complex activates platelets leading to thrombosis and thrombocytopenia.

Metformin

Diabetes Mellitus Type 2 Dental considerations: Taste disorder

C. Apical pulse and blood pressure

Before administering a dose of an antidysrhythmic drug to an assigned patient, which assessments would be of highest priority? A. Urine output and specific gravity B. Temperature and pulse rate C. Apical pulse and blood pressure D. Peripheral pulses and level of consciousness

Cidofovir clinical use

CMV retinitis in immunocompromised patients; acyclovir-resistant HSV.

B Lowering intraocular pressure in patients with glaucoma

Cholinergic (parasympathomimetic) drugs are indicated for which situation? A Treating a postoperative patient who has bradycardia B Lowering intraocular pressure in patients with glaucoma C Inhibiting muscular activity in the bladder D Preventing salivation and sweating

B Increased gastrointestinal motility

Cholinergic (parasympathomimetic) drugs have which therapeutic effect? A Urinary retention B Increased gastrointestinal motility C Mydriasis D Vasoconstriction

Quinidine toxicity

Cinchonism (headache, tinnitus)

Rivaroxaban (Xarelto)

DOAC - selective/reversible direct inhibitor Xa - only 1x/day - Cant monitor, no antidote - Perisurgical bleeding may be prolonged, pruritus, thrombocytopenia, increase liver enzymes -Toxicity increased by 3A4 inhibitors such as macrocodes - NO NSAID

What effect do nitrates have on ejection time?

Decrease

Brimonidine (α2 agonist)

Decrease aqueous humor synthesis

What effect do nitrates have on EDV (end diastolic volume)?

Decreases

A. Tachycardia

Discharge teaching to a patient receiving a beta-agonist bronchodilator should emphasize reporting which side effect? A. Tachycardia B. Nonproductive cough C. Hypoglycemia D. Sedation

Receptor sites

Drugs exert an effect by binding to ________ on a cell membrane or within a cell.

Schedule I

Drugs with the highest potential for abuse are found in which schedule?

B urinary atony.

During postoperative teaching, the nurse explains that the patient is receiving bethanechol (Urecholine) to treat A postoperative hypotension. B urinary atony. C respiratory atelectasis. D postoperative ischemic colitis.

Amphetamines intoxication

Euphoria, grandiosity, pupillary dilation, prolonged wakefulness and attention, hypertension, tachycardia, anorexia, paranoia, fever. Severe: cardiac arrest, seizure

True/False, there are many generic brands of the same trade name?

False, Many trade names only one generic name.

6 months (5 months + prescription)

How many refills can you write for a Schedule III drug?

B. It binds to bile in the intestinal tract, forming an insoluble complex that is excreted in the feces.

How will the nurse explain to the patient the action of cholestyramine (Questran) and how it decreases lipid levels? A. It inhibits absorption of dietary cholesterol in the small intestine. B. It binds to bile in the intestinal tract, forming an insoluble complex that is excreted in the feces. C. It inhibits the biosynthesis of cholesterol in the liver. D. It stimulates the biliary system to increase excretion of dietary cholesterol.

What effect do nitrates have on HR?

Increase (reflex response)

Hydralazine mechanism

Increase cGMP causing smooth muscle relaxation. Vasodilates arterioles > veins; afterload reduction.

SNRIs mechanism

Inhibit 5-HT and norepinephrine reuptake.

Glucocorticoids MoA

Inhibit NF-κB. Suppress both B- and T-cell function by transcription of many cytokines.

Azoles MoA

Inhibit fungal sterol (ergosterol) synthesis by inhibiting the cytochrome P-450 enzyme that converts lanosterol to ergosterol.

How long does it take for antidepressants to have an effect?

It normally takes 4-8 weeks for antidepressants to have an effect

Adverse effect of sulfonamides during pregnancy

Kernicterus

Fibrate effect on lipid levels

LDL Δ: down HDL Δ: up TG Δ: TRIPLE DOWN!!!

b, e, f

Levothyroxine (Synthroid) has been prescribed for a patient with hypothyroidism. The nurse provides information to the patient about the medication and tells the patient to contact the prescriber if which potential adverse effect occurs? (Select all that apply.) a. Constipation b. Chest pains c. Drowsiness d. Fatigue e. Rapid heart rate f. Palpitations

List the class IB sodium channel blockers

Lidocaine, mexiletine Phenytoin can also fall into this category

Treatment of Vancomycin-resistant enterococci (VRE)

Linezolid and streptogramins (quinupristin, dalfopristin).

Bumetanide

Loop diuretic -Dehydration, low K+, oral lesions, most significant xerostomia - NSAIDs potentially decrease diuretic effectiveness

Furosemide

Loop diuretic -Dehydration, low K+, oral lesions, most significant xerostomia - NSAIDs potentially decrease diuretic effectiveness

2nd generation H1 blockers

Loratadine, fexofenadine, desloratadine, cetirizine

List the HMG-CoA reductase inhibitors

Lovastatin, pravastatin, simvastatin, atorvastatin, rosuvastatin

Entacapone

Misc Parkinson's Disease - diarrhea, avoid sudden d/c

Hepatic Microsomal enzymes of the liver

Most drugs are metabolized by the ______.

Fibrates side effects/problems

Myopathy (increase risk with statins), cholesterol gallstones

Acetaminophen toxicity antidote

N-acetylcysteine (replenishes glutathione)

Edoxaban (Savaysa)

NEW, DOAC - hypertension, nosebleed, major bleed 2.2% - NO NSAID

Side effect of haloperidol

NMS, tardive dyskinesia

A. It is the blood volume within the heart.

Nitrates relieve angina pain by reducing preload. How would the nurse explain the term preload to a patient? A. It is the blood volume within the heart. B. It is the pressure within the superior vena cava. C. It is the pressure against which the heart must pump. D. It is the oxygen demand of the heart.

List the nitrates

Nitroglycerin, isosorbide dinitrate, isosorbide mononitrate

Meperidine use

No longer first line analgesic due to high adverse effect profile

Side effects of low potency typical antipsychotics

Non-neurologic side effects (anticholinergic, antihistamine, and α1-blockade effects).

D supraventricular dysrhythmias.

Nonselective beta blockers may be used to treat hypertension and A chronic obstructive pulmonary disease (COPD). B heart failure. C heart block. D supraventricular dysrhythmias.

Antacids

OTC meds that help neutralize hydrochloric acid of the stomach o Sodium bicarbonate -Cardiovascular patients that are on sodium restriction should avoid these due to the sodium content

Name those proton pump inhibitors

Omeprazole, lansoprazole, esomeprazole, pantoprazole, dexlansoprazole.

A "If you take your pulse and it is less than 60, hold your medicine and call your health care provider for instructions."

Patient teaching for a patient being discharged on a beta blocker includes which statement? A "If you take your pulse and it is less than 60, hold your medicine and call your health care provider for instructions." B "If you become dizzy, do not take your medication for 2 days and then restart on the third day." C "This medication may make you fatigued; increasing caffeine in your diet may help alleviate this problem." D "Increase intake of green leafy vegetables to prevent bleeding problems that can be caused by this medication."

A. "Increase your fluid intake to 3 L per day."

Patient teaching for a patient receiving allopurinol (Zyloprim) should include which information? A. "Increase your fluid intake to 3 L per day." B. "Include salmon and organ meats in your diet on a weekly basis." C. "Take the medication with an antacid to minimize gastrointestinal distress." D. "This medication may cause your urine to turn orange."

a excretion

Patients with renal failure would most likely have problems with drug a excretion. b absorption. c metabolism. d distribution.

PPI use

Peptic ulcer, gastritis, esophageal reflux, Zollinger-Ellison syndrome

H2 blocker clinical use

Peptic ulcer, gastritis, mild esophageal reflux

B. Vasodilation

Phosphodiesterase inhibitors have an added advantage in treating heart failure. The drugs cause a positive inotropic effect and what other effect? A. Vasoconstriction B. Vasodilation C. Platelet inhibition D. Bronchodilation

Direct cholinomimetics as glaucoma drugs

Pilocarpine, carbachol -Mechansism: increase outflow of aqueous humor via contraction of ciliary muscle and opening of trabecular meshwork -Use pilocarpine in emergencies—very effective at opening meshwork into canal of Schlemm -Side effects: miosis and cyclospasm (contraction of ciliary muscle)

Esters (local anesthetics)

Procaine, cocaine, tetracaine

-prazole

Proton pump inhibitor

Niacin side effects/problems

Red, flushed face, which is decreased by NSAIDs or long term use; hyperglycemia, hyperuricemia (gout)

Nitrate toxicity

Reflex tachycardia (treat with β-blockers), hypotension, flushing, headache, "Monday disease" in industrial exposure: development of tolerance for the vasodilating action during the work week and loss of tolerance over the weekend - tachycardia, dizziness, headache upon reexposure.

Clinical use of aldesleukin

Renal cell carcinoma, metastatic melanoma

Factors predisposing to digoxin toxicity

Renal failure (decreased excretion), hypokalemia (permissive for digoxin binding at K+-binding site on Na+/K+ ATPase), verapamil, amiodarone, quinidine (digoxin clearance; displaces digoxin from tissue-binding sites).

Parasympathetic Response (PANS)

Rest and digest - PEACE -Lowered blood pressure -Decreased heart rate -Increased blood flow to GI system -Increased peristalsis / GI motility -Constricts bronchioles -COnstricts pupils (miOsis) -Increased salivary flow *Cholinergic agents / parasympathomimetics (Chol. goes w/ 'C' of PEACE)

Nicotine intoxication

Restlessness

Caffeine intoxication

Restlessness, increase diuresis, muscle twitching

the autonomic nervous system is divided in two called?

SANS and PANS. each division has afferent (sensory) fibers, and efferent motor preganglionic and post ganglionic.

Venlafaxine (Effexor)

SNRI - dizziness, anxiety, tremor, BP increase - additive CNS depressant effects with trazodone and opioids - less dry mouth than TCA - increase BP (increased NE activity) - used for neuropathic or chronic pain - Prolongation QT, sexual dysfunction, discontinuation syndrome

Duloxetine (Cymbalta)

SNRI - nausea, dry mouth (0-1+), constipation, fatigue - additive CNS depressant effects with trazodone and opioids - less dry mouth than TCA - increase BP (increased NE activity) - used for neuropathic or chronic pain (diabetic neuropathy) - less BP increase compared to venlafaxine

DOC for depression

SSRIs

DOC for generalized anxiety disorder

SSRIs, SNRIs

DOC for OCD

SSRIs, clomipramine

DOC for PTSD

SSRIs, venlafaxine

DOC for panic disorder

SSRIs, venlafaxine, benzodiazepines

DOC for social phobias

SSRIs, β-blockers

β-blockers (class II antiarrhythmics) clinical use

SVT, ventricular rate control for atrial fibrillation and atrial flutter

Define SLUD

Salivation,lacrimation, urination, and defecation. seen in large dose toxic effects of Cholinergic drugs.

HIV / AIDS

Several drugs are combined for an improved effect —-- Highly Active Antiretroviral Therapy (HAART) -Slows the replication rate of the virus -Sometimes called a drug cocktail these are constantly changing -usually take a combination of 3 drugs •Nucleoside Reverse Transcriptase Inhibitors (NRTI) •Non-nucleoside Reverse Transcriptase Inhibitors (NNRTI) •Protease Inhibitors (PI)

Parasympathetic Nervous System

Slows the heart, speeds SLUD

Anesthetic cartridges: Antioxidant Preservative

Sodium bisulfite or sodium metabisulfite -Contraindicated in patients with sulfite allergies or sensitivities

A. Class I

Sodium channel blockers are considered which class of antidyshythmic drugs? A. Class I B. Class II C. Class III D. Class IV

Sympathetic Nervous System

Speeds the heart, slow SLUD

Sulfonylureas clinical use

Stimulate release of endogenous insulin in type 2 DM. Require some islet function, so useless in type 1 DM.

Oxytocin clinical use

Stimulates labor, uterine contractions, milk let-down; controls uterine hemorrhage

Mechanism of resistance of cephalosporins

Structural change in penicillin-binding proteins (transpeptidases)

A. Thyroid hormones

Sympathomimetic mydriatics have the potential to react with other drugs if sufficient topical absorption occurs. When given concurrently, drugs in which class may interact with the sympathomimetic mydriatics? A. Thyroid hormones B. Oral contraceptives C. Nonsteroidal antiinflammatories D. Calcium channel blockers

Levothyroxine (T4), triiodothyronine (T3) toxicity

Tachycardia, heat intolerance, tremors, arrhythmias

What is a drug that is concentrated in the Gingival crevicular fluid?

Tetracycline.

c. "I may have cramping and expel worms for a few days after I finish the medication."

The client is being treated for pinworms with mebendazole (Vermox). The nurse has completed medication education and evaluates that learning has occurred when the client makes which statement? a. "I must avoid aspirin while I am on this medication." b. "I need high-fiber foods to help with passage of the worms." c. "I may have cramping and expel worms for a few days after I finish the medication." d. "I need to have three negative stool cultures before I am cured."

a, c, d, e

The client is taking acetylsalicyclic acid (ASA), four to five times a day for severe osteoarthritic pain. Which teaching interventions should the nurse discuss with the client? Select all that apply. a. Maintain a serum salicylate level of between 15 and 30 mg/dL. b. Explain that ringing in the ears is a common side effect. c. Do not drink any type of alcoholic beverages. d. Inform the dentist about taking high soses of ASA. e. Keep the ASA bottle out of the reach of children.

b. The client should avoid crowds

The client receives infliximab (Remicade), an immunosuppressant medication. What is important for the nurse to teach the client about this medication? a. The client should drink plenty of fluids b. The client should avoid crowds c. The client should get adequate exercise d. The client should eat plenty of fresh fruits and vegetables.

b. Dissolve the oral tablet in your mouth and then swallow it.

The client receives nystatin (Nilstat) for a fungal infection in the mouth. The nurse plans to do medication education prior to discharge. What will the best plan of the nurse include? a. Swallow the oral tablet whole without chewing or crushing it. b. Dissolve the oral tablet in your mouth and then swallow it. c. Take the oral suspension with a straw to prevent tooth discoloration. d. Crush the oral tablet, mix it with orange juice, and then swallow it

b. "This should not hurt you because vitamin deficiencies do not occur for some time."

The female client having her annual physical exam tells the clinic nurse, "I take vitamins daily but I have not had the money to buy any for the past week." Which response is most appropriate for the nurse? a. "As long as you eat a balanced diet, you do not need to take vitamins." b. "This should not hurt you because vitamin deficiencies do not occur for some time." c. "Daily vitamins are necessary, so please get them as soon as possible." d. "I will have the physician give you a prescription for some vitamins."

B. warming the eardrops to room temperature before administration helps reduce a vestibular-type reaction.

The nurse bases the plan of care regarding administration of eardrops on knowledge that A. proper administration includes holding the pinna up and out in an infant. B. warming the eardrops to room temperature before administration helps reduce a vestibular-type reaction. C. the pinna of an adult should be held down and back. D. eardrops may be warmed in the microwave before administration.

a, d, e, f

The nurse has an order for administering a medication to the patient. Which providers have legal authority to prescribe medications for patients? (Select all that apply.) a. Physician b. Physical therapist c. Pharmacist d. Dentist e. Physician assistant f. Nurse practitioner

a. It is a local anesthetic that causes temporary relief of pain.

The nurse is administering an otic preparation as part of treatment for a severe fungal ear infection. Why is pramoxine added to the otic medication? a. It is a local anesthetic that causes temporary relief of pain. b. It is a corticosteroid that reduces inflammation. c. It is an emulsifying drug that works to soften the cerumen. d. It works synergistically to enhance the effect of the antifungal drug.

c verapamil (Calan), a calcium channel-blocking drug

The nurse is admitting an 82-year-old patient for treatment of heart failure. During assessment of the patient's history, the nurse notes a subjective complaint of chronic constipation. Of the prescribed medications the patient was taking prior to admission, which would the nurse suspect might contribute to this gastrointestinal complaint? a naproxen (Aleve), a nonsteroidal antiinflammatory drug b warfarin sodium (Coumadin), an anticoagulant c verapamil (Calan), a calcium channel-blocking drug d spironolactone (Aldactone), a potassium-sparing diuretic

C. 5% albumin

The nurse is assessing a patient noted to have third spacing and edema of the hands and feet at +3, yet the patient is having signs and symptoms of intravascular dehydration. The patient's lab results report a total protein level of 4.6 g/dL. What fluid does the nurse anticipate the provider will order for this patient? A. Normal saline B. Lactated Ringer's C. 5% albumin D. Whole blood

C. "Can you tell me what vitamins you take and how many and how often?"

The nurse is educating an elderly patient on vitamin supplementation. The patient states that she takes "lots of vitamins every day" to stay healthy. Based on this information and knowledge of vitamins, what is the nurse's best response? A. "That's great. You should be very healthy." B. "You need to make certain that is okay with your health care provider." C. "Can you tell me what vitamins you take and how many and how often?" D. "You don't need vitamins if you eat well."

B. Improved hearing

The nurse is evaluating the effectiveness of the otic medication used for OE. What will the nurse evaluate for to determine if the goals of therapy have been met? A. Increased pain in ear B. Improved hearing C. Redness around ear D. Discharge in canal

c 0.2 mL

The nurse is preparing to administer an intramuscular medication using an airlock to prevent leakage of the medication into the subcutaneous space. Which amount of air should the nurse withdraw for the air lock? a 0.05 mL b 0.1 mL c 0.2 mL d 0.3 mL

b. Drainage from the ear canal

The nurse is preparing to give an earwax emulsifier to a patient and will assess the patient for which contraindication before administering the drops? a. Partial deafness in the affected ear b. Drainage from the ear canal c. Allergy to penicillin d. Excessive earwax in the outer ear canal

b "Confirm with your health care provider that any herbs you take will not interact with prescribed medications."

The nurse is providing education to a group of patients interested in complementary medicine. The nurse will include which teaching point as priority education for the group? a "Understand the use of any herb before taking it." b "Confirm with your health care provider that any herbs you take will not interact with prescribed medications." c "Read the directions and labels of all herbs before taking." d "Stop taking any herb if you note any adverse effects."

C. It must not be touched or handled by his daughter due to teratogenic effects.

The nurse is providing education to a patient and his caregiver—his pregnant daughter—about dutasteride (Avodart), which he will be taking for benign prostatic hyperplasia (BPH). What important teaching would the nurse provide to the patient and his daughter about the administration of this medication? A. It should be taken with food containing dairy. B. It may be placed in daily medication container with other medications. C. It must not be touched or handled by his daughter due to teratogenic effects. D. For best results, it needs to be taken at hour of sleep on empty stomach.

a, c, d

The nurse is providing education to a patient on the primary uses of cholinergic drugs. Which uses would the nurse include in the teaching? (Select all that apply.) A To stimulate peristalsis B To elevate heart rate C To decrease intraocular pressure D To stimulate bladder emptying E To dilate pulmonary airways

a. Iron will cause the stools to darken in color.

The nurse is providing education to a patient with iron deficiency anemia who has been prescribed iron supplements. What will the nurse include in her teaching? a. Iron will cause the stools to darken in color. b. Limit foods high in fiber due to the risk for diarrhea. c. Take the iron with dairy products to enhance absorption. d. Increase the intake of vitamin E to enhance absorption.

d. Calcium channel blockers with thiazide diuretics

The nurse is reviewing drug therapy for hypertension. According to the JNC 7 guidelines, antihypertensive drug therapy for a newly diagnosed stage 1 hypertensive African-American patient would most likely include which drug or drug classes? a. Vasodilators alone b. ACE inhibitors alone c. Beta blockers with thiazide diuretics d. Calcium channel blockers with thiazide diuretics

b, d, e

The nurse is reviewing medications for the treatment of asthma. Which drugs are used for acute asthma attacks? (Select all that apply.) a. fluticasone (Flovent) Rotadisk inhaler b. aminophylline IV infusion c. salmeterol (Serevent) inhaler d. albuterol (Proventil) nebulizer solution e. epinephrine f. montelukast (Singulair)

a. phenobarbital

The nurse is reviewing the dosage schedule for several different antiepileptic drugs (AEDs). Which antiepileptic drug allows for once-a-day dosing? a. phenobarbital b. valproic acid (Depakote) c. gabapentin (Neurontin) d. levetiracetam (Keppra)

a. Nicotinic

The nurse is reviewing the mechanism of action of cholinergic drugs. The undesired effects of cholinergic drugs come from the stimulation of which receptors? a. Nicotinic b. Ganglionic c. Muscarinic d. Cholinergic

c. "No, it could be excreted in your milk and affect the baby."

The nursing mother asks the nurse if it is all right to take St. John's wort for mild depression. What is the best response by the nurse? a. "No, it will affect the taste of your milk, and your baby might reject nursing." b. "No, it will probably cause your baby to have more allergies." c. "No, it could be excreted in your milk and affect the baby." d. "No, because it might decrease the amount of milk you produce."

Half-life

Time required fir the concentration of a drug in the blood stream to be reduced by 50%.

Nondepolarizing neuromuscular blocking drugs

Tubocurarine, atracurium, mivacurium, pancuronium, vecuronium, rocuronium—competitive antagonists—compete with ACh for receptors.

C. Vitamin D

What vitamin can be synthesized by the skin when exposed to sunlight but can also be supplemented as needed? A. Vitamin C B. Vitamin K C. Vitamin D D. Vitamin B complex

B. Impaired gas exchange related to thickened respiratory secretions

What would be a priority nursing diagnosis for a patient receiving anticholinergic (parasympatholytic) drugs? A. Risk for injury related to excessive central nervous system stimulation B. Impaired gas exchange related to thickened respiratory secretions C. Urinary retention related to loss of bladder tone D. Deficient knowledge related to pharmacologic regimen

b insert the needle at a 45- or 90-degree angle, depending on patient size, to penetrate subcutaneous tissue.

When administering a medication subcutaneously, the nurse will a use a 1- to 1½-inch, 25-gauge needle. b insert the needle at a 45- or 90-degree angle, depending on patient size, to penetrate subcutaneous tissue. c aspirate with heparin and insulin injections. d use the landmark between the greater trochanter and superior iliac crest for the vastus lateralis site.

D a negative inotropic, negative chronotropic, and negative dromotropic effect.

When assessing for cardiovascular effects of a beta blocker, the nurse understands that these drugs produce A a positive inotropic, positive chronotropic, and positive dromotropic effect. B a positive inotropic, negative chronotropic, and negative dromotropic effect. C a negative inotropic, positive chronotropic, and positive dromotropic effect. D a negative inotropic, negative chronotropic, and negative dromotropic effect.

C. Hypokalemia

When assessing for potential side effects of fludrocortisone (Florinef), the nurse monitors for signs and symptoms of which adverse effect? A. Hyponatremia B. Hypercalcemia C. Hypokalemia D. Hypovolemia

B. Maintain infusion rate at no greater than 20 mEq/hr.

When caring for a patient with serum potassium of 2.8 mEq/L, which is a priority nursing intervention when administering intravenous replacement therapy? A. Administer potassium as a bolus over 10 minutes. B. Maintain infusion rate at no greater than 20 mEq/hr. C. Apply ice packs to site of intravenous administration. D. Teach the patient and family the signs and symptoms of hypokalemia.

b, c, d, e

When diarrhea occurs in the pediatric patient, the nurse will advise the parents to seek emergency medical treatment for the patient if which symptoms occur? (Select all that apply.) A. Urination every 3 to 4 hours B. Malaise or lethargic C. Severe abdominal pain D. Firm or rigid abdomen E. Bloody diarrhea

C. flushed skin and increased thirst.

When planning administration of hypertonic saline solution to treat a patient with severe hyponatremia, the nurse monitors for signs and symptoms of overdose as manifested by A. lethargy and hypotension. B. vomiting and diarrhea. C. flushed skin and increased thirst. D. confusion and seizures.

D. silver sulfadiazine (Sulfadine)

When planning care for a burn patient, the nurse would anticipate applying which topical medication? A. erythromycin (E-Mycin) B. lindane C. minoxidil (Rogaine) D. silver sulfadiazine (Sulfadine)

c Have the patient perform a return demonstration of the procedure.

When planning care for an assigned patient, the nurse identifies the outcome of "Patient will be able to safely self-administer enoxaparin (Lovenox) subcutaneously upon discharge." Which method best evaluates the patient's achievement of this outcome? a Have the patient verbalize the correct procedure step by step. b Visually demonstrate the correct procedure to the patient. c Have the patient perform a return demonstration of the procedure. d Give the patient detailed written instructions illustrating the procedure.

c the majority of medication errors result from weaknesses within the system rather than individual shortcomings.

When planning interventions aimed at reducing medication errors, the nurse recognizes that a only 10% of all preventable adverse drug reactions (ADRs) begin at the medication ordering (prescribing) stage. b disciplinary action is necessary to increase the nurse's vigilance in preventing medication errors. c the majority of medication errors result from weaknesses within the system rather than individual shortcomings. d the use of trailing zeros (i.e., 1.0 mg) and omission of leading zeros (i.e., .25 mg) reduces transcription errors.

B. Do not use in patients younger than 2 years of age, unless prescribed.

When providing general education on use of over-the-counter medications for allergies, which is the highest priority for the nurse to include? A. Discontinue use 4 days before allergy testing. B. Do not use in patients younger than 2 years of age, unless prescribed. C. It may cause dry mouth. D. The medication treats the symptoms but is not a cure.

B Take the medication 30 minutes before meals.

When providing teaching to a patient diagnosed with myasthenia gravis, which instruction regarding the administration of physostigmine (Antilirium) is most appropriate? A Increase fluid and fiber in the diet to prevent constipation. B Take the medication 30 minutes before meals. C If a dose is missed, double the next dose to prevent withdrawal. D Common side effects include tachycardia and hypertension.

C. Place the patch on clean, dry, shaved scrotal skin.

When teaching a patient regarding proper application of a Testoderm transdermal patch, the nurse will instruct the patient to perform which action? A. Replace the patch every 72 hours. B. Replace the patch every 36 hours. C. Place the patch on clean, dry, shaved scrotal skin. D. Place the patch on the back, abdomen, upper arms, or thigh.

a First trimester

When teaching a pregnant mother about the effects of medication on the fetus, the nurse recognizes that the greatest harm from maternally ingested medications occurs during which time period? a First trimester b Second trimester c Third trimester d Birthing process

d Patient is unable to verbalize reason for taking the medication.

Which assessment findings best support the nursing diagnosis of "Deficient knowledge related to medication therapy"? a Patient is not taking medication as directed. b Patient is not reporting side effects as directed. c Patient is unwilling to comply with medication therapy. d Patient is unable to verbalize reason for taking the medication.

C. Hypokalemia

Which laboratory value depicts a known side effect of furosemide (Lasix)? A. Hyperchloremia B. Hypernatremia C. Hypokalemia D. Hypophosphatemia

B. Glucose levels

Which laboratory value might indicate an adverse response to hydrochlorothiazide (HydroDIURIL)? A. Sodium levels B. Glucose levels C. Calcium levels D. Chloride levels

A. Liver enzymes

Which laboratory value would the nurse assess before administering zafirlukast (Accolate) to a patient? A. Liver enzymes B. Cardiac enzymes C. Renal function tests D. Complete blood count

b. Vancomycin (Vancocin)

Which of the following is an antibiotic responsible for causing red-man syndrome as an adverse effect? a. Erythromycin (E-mycin) b. Vancomycin (Vancocin) c. Tetracycline (Sumycin) d. Cefotaxime (Claforan)

FDA

Which regulatory agency has jurisdiction over the advertising of most drugs?

a Administration of some drugs may elicit varied responses in specific racial-ethnic groups.

Which statement best reflects the nurse's understanding of cultural influences on drug therapy and other health practices? a Administration of some drugs may elicit varied responses in specific racial-ethnic groups. b Regardless of one's cultural background, it is crucial to always adhere to recommended medical practices. c Most cultures are fairly standard in reference to the use of medications during illness. d Dietary habits and practices can be of little value to the care of an ill adult.

B. "This medication will help prevent the inflammatory response of my allergies."

Which statement by the patient demonstrates understanding of action or use of beclomethasone diproprionate (Beconase)? A. "I will need to taper off the medication to prevent acute adrenal crisis." B. "This medication will help prevent the inflammatory response of my allergies." C. "I will need to monitor my blood sugar more closely because it may increase secondary to Beconase." D. "I only need to take this medication when my symptoms get bad."

B. "Smoking decreases the effects of this medication, so I should look into cessation programs."

Which statement demonstrates understanding of patient teaching regarding the use of histamine2-receptor antagonists? A. "Since I am taking this medication, it is OK for me to eat spicy foods." B. "Smoking decreases the effects of this medication, so I should look into cessation programs." C. "I should take this medication 1 hour after each meal in order to maximally decrease gastric acidity." D. "I should decrease bulk and fluids in my diet to prevent diarrhea."

B. "I will increase fiber in my diet."

Which statement will indicate to the nurse that the patient understands the discharge instructions regarding cholestyramine (Questran)? A. "I will take Questran 1 hour before my other medications." B. "I will increase fiber in my diet." C. "I will weigh myself weekly." D. "I will have my blood pressure checked weekly."

C. "Wait 1 to 2 minutes before you take a second puff of the same drug."

Which statement would be included when teaching a patient about the proper use of metered-dose inhalers? A. "After you inhale the medication once, repeat until you obtain relief." B. "Make sure that you puff out air repeatedly after you inhale the medication." C. "Wait 1 to 2 minutes before you take a second puff of the same drug." D. "Hold the inhaler in your mouth, take a deep breath, and then compress the inhaler."

A. "I will check my blood pressure every day and take my medication when it is over 140/90."

Which statement, if made by your patient, signifies that additional patient teaching regarding antihypertensive treatment is required? A. "I will check my blood pressure every day and take my medication when it is over 140/90." B. "I will include rest periods during the day to help me tolerate the fatigue my medicine may cause." C. "I will change my position slowly to prevent feeling dizzy." D. "I will not mow my lawn until I see how this medication makes me feel."

C. "I will take the medication first thing in the morning with 8 ounces of water and remain upright for 30 minutes."

Which statement, made by the patient, indicates an understanding of discharge teaching regarding alendronate (Fosamax)? A. "I need to decrease my intake of dairy products so as to prevent hypercalcemia." B. "I need to take this medication with food to prevent damage to my esophagus." C. "I will take the medication first thing in the morning with 8 ounces of water and remain upright for 30 minutes." D. "This medication will help relieve the bone pain I have from my osteoporosis."

B. Liver function tests

Which test will the nurse use to assess for adverse reactions to HMG-CoA reductase inhibitors? A. Serum cholesterol levels B. Liver function tests C. Serum electrolytes D. Complete blood count

a produces deep muscle relaxation and loss of consciousness.

While completing preoperative patient teaching, the nurse explains that general anesthesia a produces deep muscle relaxation and loss of consciousness. b results in moderate sedation in which the patient can follow commands but will not remember anything following the procedure. c provides anesthesia to a specific region of the body and generalized sedation. d typically is achieved using only one type of medication.

A. tirofiban (Aggrastat)

While preparing a patient with acute chest pain for an emergency angioplasty, the nurse would anticipate administering which medication to prevent platelet aggregation? A. tirofiban (Aggrastat) B. protamine sulfate C. warfarin (Coumadin) D. aminocaproic acid (Amicar)

Topical Anesthetics Cetacaine

Yellow-Smells like bananas benzocaine 14%, butamben 2.0%, tetracaine HCI 2.0%

Passive transport

____ requires no energy and moves from and area of high concentration to low concentration.

Active transport

_______ requires and energy source and moves from an area of low concentration to high concentration.

idiosyncrasy.

a reaction that is neither the drugs side effect nor an allergic reaction. caused by genetric abnormal reactions.

***articaine and bupivacaine will ALWAYS have

a vasoconstrictor***

vasodilating properties

affect potency and duration of action -less vasodilation (vasoconstrictor added) = more potent and longer duration

pc

after meals

penicillin is most likely antibiotic to cause

allergic reaction and anaphylaxis

Buspirone

anti-anxiety -dizziness, nausea, headache, nervousness - treatment of bruxism - xerostomia, CNS depressants additive

Ticagrelor (Brilinta)

anti-platelet effect -Toxicity increased by 3A4 inhibitors such as macrocodes - discontinue 5d prior to major surgery but consult MD

Tetracyclines (-cycline)

bacteriostatic *Don't take with milk / dairy -Main use is to treat acne *causes photosensitivity

Metoprolol

beta-blocker (+, selective beta-1) - Can use up to 0.2mg epic with + agents. - NSAIDs may reduce effectiveness, may cause xerostomia

PO

by mouth.

Common medications that cause xerostomia:

diuretics anticholinergic agents antidepressants antihistamines adrenergic agents and many more!!

Lethal dose

dose that kills 50% of animals in laboratory studies

Effective dose

dose that produces effect in 50% of animals

Common examples of a -Blockers

doxazosin (Cardura) terazosin (Hytrin) A(lpha) to Z(osin)!

routes can be classified as Enteral or parenteral. define the two.

drugs given by the enteral route are placed directly into the GI tract by oral or rectal administration. Paraenteral route bypasses the gi tract and includes injection routes, inhalation, and topical administration.

Sans: is designed for

emergencies, the fight or flight response.

Gastric ulcerations do NOT cause

gingival bleeding

State 2 Mixed infection

gram-positive cocci and gram-negative anaerobic organisms -Gram-positive cocci - same as stage 1 drugs -Anaerobic organisms - metronidazole or clindamycin **Clindamycin - treats both gram-positive cocci and gram-negative anaerobes.

and Agonist drug is?

has affinity for the receptor, combines with the receptor and produces an effect.

Aspirin should be avoided in asthmatics because

increased risk of aspirin hypersensitivity

Indirect acting parasympathomimetic agents act by?

inhibiting the enzyme cholinesterase.

The absorption phase is bypassed when a drug is administered

intravenously. ***No LA should be delivered into blood stream

Efficacy

is the maximal intensity of effect or response that can be produced by a drug.

nonopioids primarily affect the-

peripheral nerve endings . although their antipyretic effect is mediated centrally.

protein binding capacity

related to duration of action -stronger binding = longer duration

phramacokinetics?

the study of how a drug enters the body, circulates within the body, is changed by the body and leave the body.

the ratio LD50/ED50 is

the therapeutic index TI of a drug..

anticholinergic effects on smooth muscle?

they relax the muscles in respiratory and GI tracts. Think asthma medication for the trachea and spasmolytic agents are used to reduce GI motility.

tid

three times a day

Wide therapeutic index (large TI) -

toxicity less likely - safer drug

penicillin erythromycin, acetaminophen and LA are considered safe? true/false

true

Trade name

(Brand name) o First letter capitalized o May have multiple trade names Ex: Motrin, Advil

anticoagulant heparin

(Sounds like H-aspirin) -One of most common used anticoagulant used in Hospitalized patients -Given by injection only

Anticholinergic Drugs

(act opposite of PANS - block acetylcholine receptors) -Mydriasis - dilation -Reduced secretions yerostomic affect -Reduced GI motility Uses: ****ATROPINE - used pre-operatively to decrease salivary flow for dental procedures**** - ipratropium for COPD - relaxes smooth muscle in respiratory system -loperamide (Imodium) - used to treat diarrhea -copolamine- used for motion sickness prevention

Colchicine

-Acute gout -Binds and stabilizes tubulin to inhibit microtubule polymerization, impairing neutrophil chemotaxis and degranulation. -Acute and prophylactic value. GI side effects.

Calcium channel blockers mechanism

-Block voltage-dependent L-type calcium channels of cardiac and smooth muscle to decrease muscle contractility. -Vascular smooth muscle—amlodipine = nifedipine > diltiazem > verapamil. -Heart—verapamil > diltiazem > amlodipine = nifedipine (verapamil = ventricle).

Methadone side effects

-CSA Schedule 2 drug -Same as morphine, plus prolonged QT interval, and cardiac arrhythmia. -CI: ↓metabolism (by drug-drug interaction or impaired hepatic function) ↑risk of respiratory depression & cardiac adverse effects. Potential for drug-drug interaction w/ MAOIs & SSRIs (due to weak inhibition of serotonin reuptake)→ potential for Serotonin Syndrome

3rd generation cephalosporins

-Ceftriaxone, cefotaxime, ceftazidime) -Use: serious gram-negative infections resistant to other β-lactams. Ceftriaxone—meningitis, gonorrhea, disseminated lyme disease; ceftazidime—Pseudomonas

Drugs for absence seizures

-Ethosuxamide is 1st line -Lamotragine and valproate may also be used

Ulcers Causes:

-Helicobacter pylori (tx-antibiotic) -Use of nonsteroidal anti-inflammatory drugs (NSAIDS) Tx- antibiotic, H2 blocker, PPI **not associated with GERD**

Donepezil, galantamine, rivastigmine

-Mechanism: AChE inhibitors -Use: Alzheimer disease -Toxicity: nausea, dizziness, insomnia

Demeclocycline (mechanism, use, and toxicity)

-Mechanism: ADH antagonist (member of tetracycline family). -Use: SIADH. -Toxicity: Nephrogenic DI, photosensitivity, abnormalities of bone and teeth.

Metoclopramide (mechanism, clinical use and toxicity)

-Mechanism: D2 receptor antagonist. Increase resting tone, contractility, LES tone, motility. Does not influence colon transport time. -Use: Diabetic and postsurgery gastroparesis, antiemetic. -Toxicity: Increased parkinsonian effects, tardive dyskinesia. Restlessness, drowsiness, fatigue, depression, diarrhea. Drug interaction with digoxin and diabetic agents. Contraindicated in patients with small bowel obstruction or Parkinson disease (due to D1-receptor blockade).

Leuprolide (mechanism, use, toxicity)

-Mechanism: GnRH analog with agonist properties when used in pulsatile fashion; antagonist properties when used in continuous fashion (downregulates GnRH receptor in pituitary causing decreased FSH/LH). -Use: infertility (pulsatile), prostate cancer (continuous use following androgen receptor blockade with flutamide), uterine fibroids (continuous), precocious puberty (continuous). -Toxicity: antiandrogen, nausea, vomiting.

Orlistat (mechanism, clinical use and toxicity)

-Mechanism: Inhibits gastric and pancreatic lipase breakdown and absorption of dietary fats. -Use: Weight loss. -Toxicity: Steatorrhea, decreased absorption of fat-soluble vitamins.

Cinacalcet (mechanism, use, and toxicity)

-Mechanism: Sensitizes Ca2+-sensing receptor (CaSR) in parathyroid gland to circulating Ca2+ and thus decreases PTH. -Use: Hypercalcemia due to 1° or 2° hyperparathyroidism. -Toxicity: Hypocalcemia.

Cyclobenzaprine

-Mechanism: centrally acting skeletal muscle relaxant. Structurally related to TCAs, similar anticholinergic side effects. -Use: muscle spasms

Paclitaxel, other taxols (mechanism, use, toxicity)

-Mechanism: hyperstabilize polymerized microtubules in M phase so that mitotic spindle cannot break down (anaphase cannot occur). "It is taxing to stay polymerized." -Use: ovarian and breast carcinomas. Toxicity: myelosuppression, alopecia, hypersensitivity.

ARBS commonly end in

-Sartan

ADHD S&S

-inattentiveness -inability to concentrate -restlessness (fidgety) -hyperactivity -inability to complete tasks -impulsivity

within 6 months

How long can you wait before you fill your prescription for a Schedule III drug?

pre-med regimen If allergic to penicillin cephalexin (Keflex)

2000 mg one hour prior

Suggested pre-med regimen if needed: amoxicillin

2000mg (2g) (4)-500mg pills 30 minutes - 1 hour prior to dental appointment

The parasympathetic division originates in the nuclei of cranial nerves?

3,7,9 ,10 and sacral segments of the spinal cord.

2 years

If a dentist gives a patient a controlled substance, such as Percocet, how long must they maintain records of this?

pre-med regimen If allergic to penicillin azithromycin

500 mg one hour prior

pre-med regimen If allergic to penicillin clarithromycin

500 mg one hour prior

Ganciclovir MoA

5′-monophosphate formed by a CMV viral kinase. Guanosine analog. Triphosphate formed by cellular kinases. Preferentially inhibits viral DNA polymerase. Preferentially inhibit viral DNA polymerase by chain termination.

B. Otic and oral antimicrobials

A 14-year-old patient is diagnosed with perforated tympanic membrane, OE, and otitis media. What does the nurse anticipate the provider will prescribe? A. Otic antimicrobials only B. Otic and oral antimicrobials C. Oral antimicrobial only D. Otic antimicrobial and otic antifungal

b. "The full therapeutic effects may not occur for 3 to 4 weeks."

A 19-year-old student was diagnosed with hypothyroidism and has started thyroid replacement therapy with levothyroxine (Synthroid). After 1 week, she called the clinic to report that she does not feel better. Which response from the nurse is correct? a. "Is it possible that you did not take your medication as instructed?" b. "The full therapeutic effects may not occur for 3 to 4 weeks." c. "It will probably require surgery for a cure to happen." d. "Let's review your diet; it may be causing absorption problems."

B. Congenital glaucoma

A child born with increased intraocular pressure is likely to be diagnosed with which type of glaucoma? A. Angle-closure glaucoma B. Congenital glaucoma C. Pigmentary glaucoma D. Open-angle glaucoma

Heparin use

Immediate anticoagulation for pulmonary embolism (PE), acute coronary syndrome, MI, deep venous thrombosis (DVT). Used during pregnancy (does not cross placenta). Follow PTT.

d. Fever and chills

A client is receiving amphotericin B, a systemic antifungal. The nurse will reinforce teaching by telling the client that he should watch for: a. Constipation b. Heartburn c. Headache d. Fever and chills

c, e, f

A client is receiving monthly doses of chemotherapy for treatment of stage III colon cancer. The nurse should report which of the following laboratory results to the oncologist before the next dose of chemotherapy? SATA a. Hemoglobin of 14.5 g/dL b. Blood Urea Nitrogen (BUN) level of 12 mg/dL c. Temperature of 101.2 F d. Urine specific gravity of 1.020 e. White Blood Cell Count of 2,300/mm f. Platelet count of 40,000/mm

b, c, e

A client who has been diagnosed with tuberculosis has been placed on drug therapy. The medication regimen includes rifampin (Rifadin). Which of the following instructions should the nurse include in the client's teaching plan related to the potential adverse effects of rifampin? Select all that apply. a. Having eye examinations every 6 months b. Maintaining follow-up monitoring of liver enzymes c. Avoiding alcohol intake d. Decreasing protein intake in the diet e. The urine may have an orange color

B. History of thromboembolic events

A female patient arrives in the clinic for counseling on potential hormone replacement therapy. When taking the patient history, which finding would the nurse consider as a contraindication to use of hormone replacement therapy for the patient? A. High cholesterol B. History of thromboembolic events C. Early menstrual onset D. High number of pregnancies

b. "You will have to use two contraceptive methods while on this drug."

A female patient will be starting therapy with oral isotretinoin (Amnesteem) as part of treatment for severe acne, and the nurse is providing teaching. Which teaching point will the nurse include in her teaching plan about isotretinoin? a. "You will have to avoid pregnancy for 2 weeks after taking this drug." b. "You will have to use two contraceptive methods while on this drug." c. "You must avoid sexual activity while on this drug." d. "If you are taking an oral contraceptive, you may take this drug."

D. Longer duration of action

A nurse is providing education to a patient taking two different medications. The nurse identifies which characteristic as the advantage of salmeterol (Serevent) over other beta2 agonists such as albuterol (Proventil)? A. Shorter onset of action B. Better side effect profile C. Quicker peak action D. Longer duration of action

a, b, d

A nurse is providing teaching to a client who is to start colchicine (Colgout) for acute gouty arthritis. The nurse should advise the client to do which of the following? Select all that apply. a. Avoid aspirin or products containing salicylates. b. Avoid alcohol use. c. Decrease fluid intake. d. Notify the provider of bleeding, bruising, or sore throat. e. Take the medication on an empty stomach to increase absorption.

a. Drug interactions

A nurse working with elderly patients is concerned about the number of medications each patient is taking. Which will the nurse assess as the highest priority for the patients related to polypharmacy? a. Drug interactions b. Cost of medications c. Schedule of medications d. Nonadherence to drug regimen

B buspirone (BuSpar)

A patient diagnosed with an anxiety disorder has been using lorazepam (Ativan) but finds the side effect of drowsiness to be interfering with life. Which anxiolytic medications might be a better option for this patient? A alprazolam (Xanax) B buspirone (BuSpar) C chlordiazepoxide (Librium) D hydroxyzine hydrochloride salt (Vistaril)

b. Rheumatoid arthritis

A patient has an order for the monoclonal antibody adalimumab (Humira). The nurse notes that the patient does not have a history of cancer. What is another possible reason for administering this drug? a. Osteoporosis b. Rheumatoid arthritis c. Thrombocytopenia d. Severe anemia

c. Convulsions

A patient has been started on therapy of a continuous infusion of lidocaine after receiving a loading dose of the drug. The nurse will monitor the patient for which adverse effect? a. Dry mouth b. Drowsiness c. Convulsions d. Nystagmus

D. Dairy

A patient informs the nurse she must take her iron with a meal to avoid stomach upset. To increase uptake of oral iron, which food group should the nurse instruct the patient to avoid? A. Proteins B. Fruits C. Vegetables D. Dairy

b. polyethylene glycol (GoLYTELY)

A patient is about to undergo a diagnostic bowel procedure. The nurse expects which drug to be used to induce total cleansing of the bowel? a. docusate sodium (Colace) b. polyethylene glycol (GoLYTELY) c. magnesium hydroxide (milk of magnesia) d. mineral oil

b. Fresh frozen plasma

A patient is admitted to the ICU with a diagnosis of bleeding esophageal varices. The patient has a comorbidity of cirrhosis. An attempt to stop the bleeding has been only partially successful. What would the critical care nurse expect to have ordered for this patient? a. Vitamin K b. Fresh frozen plasma c. Heparin drip d. Oral anticoagulants

b methadone

A patient is admitted to the psychiatric unit for treatment of narcotic addiction. The nurse would anticipate administration of which medication? a morphine b methadone c meperidine d naloxone

a. Diarrhea

A patient is concerned about the adverse effects of the fibric acid derivative she is taking to lower her cholesterol level. Which is an adverse effect of this class of medication? a. Diarrhea b. Constipation c. Joint pain d. Dry mouth

b. amiodarone (Cordarone)

A patient is in the intensive care unit because of an acute myocardial infarction. He is experiencing severe ventricular dysrhythmias. The nurse will prepare to give which drug of choice for this dysrhythmia? a. adenosine (Adenocard) b. amiodarone (Cordarone) c. verapamil (Calan) d. diltiazem (Cardizem)

A. Antihistamines

A patient is making an appointment for allergy testing. The nurse instructs the patient not take what class of medications at least 4 days before allergy testing? A. Antihistamines B. Decongestants C. Bronchodilators D. Antitussives

B. Bacterial infection

A patient is prescribed bacitracin topical ointment. What does the nurse suspect as the possible diagnosis based on the medication prescribed? A. Fungal infection B. Bacterial infection C. Parasitic infection D. Viral infection

d. Risk for falls related to decreased sensorium

A patient is recovering from a minor automobile accident that occurred 1 week ago. He is taking cyclobenzaprine (Flexeril) for muscular pain and goes to physical therapy three times a week. Which nursing diagnosis would be appropriate for him? a. Risk for addiction related to psychological dependency b. Decreased fluid volume related to potential adverse effects c. Disturbed sleep pattern related to the drug's interference with REM sleep d. Risk for falls related to decreased sensorium

c Complementary medicine

A patient provides a list of her current medications to the nurse. The patient tells the nurse that she takes medications prescribed by her physician as well as herbs from the local health food store. Based on this information, the patient uses which type of medicine? a Western medicine b Eastern medicine c Complementary medicine d Traditional medicine

c "Vasoconstriction caused by epinephrine enhances the duration of action for lidocaine and minimizes bleeding at the laceration site."

A patient questions the use of epinephrine for repair of a laceration, stating, "I thought that was the drug they use in the emergency room for patients who are coding." Which is the nurse's best response? a "Epinephrine is used with lidocaine to prevent side effects." b "The systemic absorption of lidocaine is maximized by the epinephrine, and the anesthetic effect is reached more quickly." c "Vasoconstriction caused by epinephrine enhances the duration of action for lidocaine and minimizes bleeding at the laceration site." d "Epinephrine is metabolized more quickly than lidocaine so that the anesthetic effect wears off more quickly once the laceration is sutured."

B. altered coagulation studies.

A patient receiving vitamin K most likely has demonstrated A. diarrhea. B. altered coagulation studies. C. seizure activity. D. confusion.

a, c, d

A patient taking spironolactone (Aldactone) requests assistance with dietary choices. The nurse would recommend which food choices? (Select all that apply.) A. Lean meat B. Bananas C. Apples D. Squash

B. vitamin K

A patient who has been anticoagulated with warfarin (Coumadin) is admitted with gastrointestinal bleeding. The nurse will anticipate administering which substance? A. vitamin E B. vitamin K C. protamine sulfate D. calcium gluconate

B Wearing-off phenomenon

A patient who has been taking antiparkinson medications for years begins to have increased symptoms on a constant basis. In documenting these symptoms, what term will the nurse use? A On-off phenomenon B Wearing-off phenomenon C Chorea D Dystonia

Pharmacokinetics ADME

Absorption Distribution Metabolism Excretion

C. Paroxysmal supraventricular tachycardia (PSVT)

Adenosine is used to treat which condition? A. Atrial fibrillation B. Atrial flutter C. Paroxysmal supraventricular tachycardia (PSVT) D. Second-degree atrioventricular block

predictable

Adverse/side effects are normally _____.

Adverse effects

Affects that are caused by a drug, other than the intended effects.

B. 0.5 mg

An adult patient presents with symptomatic bradycardia. The nurse prepares to administer which dose of atropine intravenously? A. 0.3 mg B. 0.5 mg C. 1.25 mg D. 2 mg

B. Venous thromboembolism

An elderly woman being treated for osteoporosis with a selective estrogen receptor modulator (SERM) should be alerted to the potential for an increased risk of which condition? A. Elevated blood pressure B. Venous thromboembolism C. Hypercalcemia D. Skin color changes

b. Cells lack an adequate blood supply and are deprived of oxygen and nutrients.

An understanding of the pathophysiologic rationale behind shock is something every nurse needs to have. Which of the following statements best describes the pathophysiology for shock? a. Blood is shunted from vital organs to peripheral areas of the body. b. Cells lack an adequate blood supply and are deprived of oxygen and nutrients. c. Circulating blood volume is decreased. d. Hemorrhage occurs as a result of trauma.

Montelukast

Asthma, seasonal allergies, prevention of exercise-induced bronchoconstriction Dental considerations: Dental pain, increased bleeding tendency/thrombocytopenia

penicillin is NOT effective against penicillinase producing bacteria... BUT

Augmentin IS effective - it prevents penicillinase from breaking down amoxicillin o Augmentin = amoxicillin with clavulanic acid

Rituximab clinical use

B-cell non-Hodgkin lymphoma, CLL, RA, ITP

What effect do β-blockers have on ejection time?

Increase

SNRIs toxicity

Increase BP most common; also stimulant effects, sedation, nausea.

a Decrease the amount of drug given

Knowing that the albumin in neonates and infants has a lower binding capacity for medications, the nurse can expect the prescriber to perform which action to minimize the risk of toxicity? a Decrease the amount of drug given b Increase the amount of drug given c Shorten the time interval between doses d Administer the medication intravenously

Which benzos are used in the elderly?

Lorazepam, oxazepam, temazepam "The elderly like Benzos a LOT"

B. inhibiting prostaglandin production.

Most nonsteroidal antiinflammatory drugs (NSAIDs) work by A. enhancing pain perception. B. inhibiting prostaglandin production. C. increasing blood flow to painful areas. D. increasing the supply of natural endorphins.

N-acetylcysteine

Mucolytic—can loosen mucous plugs in CF patients by disrupting disulfide bonds. Also used as an antidote for acetaminophen overdose.

Dicyclomide

-Muscarinic antagonist -Antispasmodics for IBS

Benzodiazepines Examples:

-diazepam (Valium) -lorazepam (Ativan) -alprazolam (Xanax) -midazolam (Versed) short half life: IV benzodiazepine used in dentistry -triazolam (Halcion) greatest amnesia produced

Diuretics commonly end in

-ide

Carbon monoxide toxicity antidote

100% O2, hyperbaric O2 Penicillamine

A trade name is protected by a federal patent for how long?

20 years

Aspirin has how many 'A' effects?

4

pre-med regimen If allergic to penicillin clindamycin

600 mg one hour prior C.diff

Hypertension with diabetes mellitus

ACE inhibitors/ARBs, Ca2+ channel blockers, thiazide diuretics, β-blockers. ACE inhibitors/ARBs are protective against diabetic nephropathy.

-stigmine

AChE inhibitor

Azithromycin

Acute otitis media No effects on dental treatment

Teratogenic

Adverse effects on the fetus

Chronic gout drugs

Allopurinol, febuxostat, pegloticase, probenecid

Tricyclic antidepressants

Amitriptyline, nortriptyline, imipramine, desipramine, clomipramine, doxepin, amoxapine.

Zaleplon

Anti-anxiety - Dizziness, blurred vision, fatigue - xerostomia, CNS depressants additive

Diphenhydramine

Anti-anxiety - xerostomia, CNS depressants additive -dry mouth, sedation, tachycardia - Atropine potentiates anticholinergic effects of antihistamines

List the protease inhibitors

Atazanavir Darunavir Fosamprenavir Indinavir Lopinavir Ritonavir Saquinavir

Potassium channel blockers clinical use

Atrial fibrillation, atrial flutter; ventricular tachycardia (amiodarone, sotalol)

DOC for schizophrenia

Atypical antipsychotics

adrenergic Bronchodilators

B2 receptors agonist -terol albuterol (Ventolin, ProAir, Proventil) levalbuterol (Xopenex) salmeterol (Serevent)

Lithium Carbonate

BIOPOLAR - tremor, GI, thirst, polyuria, edema, taste disturbance, abnormal facial movements -hypothyroidism - for pure mania

Patients on cimetidine (Tagamet) - likely being treated for

BOARD ALERTS: gastric ulcers

ac

Before meals

a, c, d

Beta blockers are used to treat which disorders? (Select all that apply.) A Hypertension B COPD C Angina pectoris D Cardiac dysrhythmias E Raynaud's disease

Trastuzumab clinical use

Breast cancer

Verapamil

CCB - gingival overgrowth, position change, drug intx including the potentially fatal intx with diltiazem/verapamil and macrolides

Diltiazem

CCB -gingival overgrowth, position change, drug intx including the potentially fatal intx with diltiazem/verapamil and macrolides

Rituximab target

CD20

Only in emergency situations

Can a prescription for a Schedule II drug be called in?

Drugs that cause aplastic anemia

Carbamazepine, Methimazole, NSAIDs, Benzene, Chloramphenicol, Propylthiouracil

Calcium channel blocker toxicity

Cardiac depression, AV block (non-dihydropyridines), peripheral edema, flushing, dizziness, hyperprolactinemia (verapamil), constipation, gingival hyperplasia.

Exposure to gonorrhea

Ceftriaxone

Chemical name

Chemical structure of the drug

-chol

Cholinergic agonist

Drugs that cause Torsades de pointes

Class III (e.g., sotalol) and class IA (e.g., quinidine) antiarrhythmics, macrolide antibiotics, antipsychotics, TCAs

What drugs can be used in a hypertensive emergency?

Clevidipine, fenoldopam, labetalol, nicardipine, nitroprusside

Drugs that cause pseudomembranous colitis

Clindamycin, ampicillin, cephalosporins

5-fluorouracil (5-FU) use

Colon cancer, pancreatic cancer, basal cell carcinoma (topical)

Side effect of chlorpromazine

Corneal deposits

Drugs that cause pancreatitis

Didanosine, Corticosteroids, Alcohol, Valproicacid, Azathioprine, Diuretics (furosemide, HCTZ)

natural opioid-like substances in the body

Endorphins

Dopamine agonists

Ergot—Bromocriptine Non-ergot (preferred)—pramipexole, ropinirole

Pharmacokinetics Absorption

Factors -Lipid solubility -Ionization- nonionic molecules cross membranes more readily -Molecular size -Site of absorption oral route - large amount absorbed from small intestine -Solubility -Infection more acidic tissue increased drug ionization - less penetration of membranes -Blood flow at injection site

Name the Sulfonylureas

First generation: Chlorpropamide, Tolbutamide Second generation: Glimepiride, Glipizide, Glyburide

NONOPIOID ANALGESICS (NONNARCOTICS)

For Pain **Act at peripheral nerve endings, not in CNS, opiods work in CNS. -Inhibit prostaglandin synthesis -Prostaglandins: •Sensitize pain receptors •Lower pain threshold •Cause inflammation and fever •Edema -Reduce fever through action on hypothalamus

Clevidipine clinical use

HTN urgency or emergency

NONE

How many refills can a prescription for a Schedule II drug be given?

Hypertension in pregnancy

Hydralazine, lebetalol, methyldopa, nifedipine

B. In the evening

Hydroxymethylglutaryl-coenzyme A (HMG-CoA) reductase inhibitors (statins) are generally administered at which time? A. When the stomach is empty B. In the evening C. With breakfast D. With an antacid

c Fowler's position

In which position is it most appropriate to place a patient when administering medications via a nasogastric tube? a Left side b Supine c Fowler's position d Trendelenburg position

Spironolactone

Inhibits steroid binding, 17α-hydroxylase, and 17,20-desmolase.

Proton pump inhibitor mechanism

Irreversibly inhibits H+/K+ ATPase in stomach parietal cells

What effect does ezetimibe have on lipid levels?

LDL Δ: double decrease HDL Δ: none TG Δ: none

Niacin effect on lipid levels

LDL Δ: double down HDL Δ: double up TG Δ: down

Caffeine withdrawal

Lack of concentration, headache

PREGNANCY Local Anesthetics • Safe for use:

Lidocaine and prilocaine-

Drugs that cause hypothyroidism

Lithium, amiodarone, sulfonamides

Drugs that cause diabetes insipidus

Lithium, demeclocycline

Escitalopram

Major Depressive Disorder/ Generalized anxiety disorders Dental considerations: Xerostomia, toothache, possible bleeding complication, especially co-administered with other antiplatelet agents

C. Allergens

Mast cell stabilizers are most effective in treating bronchoconstriction associated with which condition? A. Emphysema B. Exposure to cold C. Allergens D. Infection

Propranolol side effects

May exacerbate vasospasm in Prinzmetal angina

Daclizumab, basiliximab MoA

Monoclonal antibodies; block IL-2R.

Transcription

On a prescription, this is directions to the patient:

Arylcyclohexylamines (Ketamine)

PCP analogs that act as dissociative anesthetics. Block NMDA receptors. Cardiovascular stimulants. Cause disorientation, hallucination, bad dreams. cerebral blood flow.

What coagulation pathway is affected by warfarin?

PT (extrinsic pathway)

Drugs with solubility in blood =

Rapid induction and recovery times.

Procainamide toxicity

SLE like syndrome (reversible)

Typical antipsychotic use

Schizophrenia (primarily positive symptoms), psychosis, acute mania, Tourette syndrome

Tacrolimus toxicity

Similar to cyclosporine, risk of diabetes and neurotoxicity; no gingival hyperplasia or hirsutism.

Cetuximab clinical use

Stage IV colorectal cancer, head and neck cancer

DOC for ADHD

Stimulants (e.g. methylphenidate)

Adverse effect of griseofulvin during pregnancy

Teratogenic

Adverse effect of ribavirin during pregnancy

Teratogenic

Azoles toxicity

Testosterone synthesis inhibition (gynecomastia, especially with ketoconazole), liver dysfunction (inhibits cytochrome P-450).

toxicology

The Study of the harmful effects of drugs.

Dose response curve

The curve that allows us to determine how much of a drug is needed to reach the ceiling effect.

B Cardiac dysrhythmias

The nurse explains to a patient using caffeine that which disease process/condition may be exacerbated by this drug? A Myelin degeneration B Cardiac dysrhythmias C Constipation D Heart block

b Polymorphism

The nurse is assessing a patient's culture and race on admission to the hospital. Which concept is important for the nurse to understand regarding drug therapy as it relates to different races of individuals? a Polypharmacy b Polymorphism c Pharmacokinetics d Pharmacodynamics

b, c

The nurse is educating a patient about medications used to treat erectile dysfunction. Which erectile dysfunction medications have the longest therapeutic effect when taken orally? (Select all that apply.) A. sildenafil (Viagra) B. vardenafil (Levitra) C. tadalafil (Cialis) D. alprostadil (Caverject)

C. Hepatic disease

The nurse would question an order for simvastatin (Zocor) in a patient with which condition? A. Leukemia B. Diabetes C. Hepatic disease D. Chronic obstructive pulmonary disease (COPD)

generic name

The official name of a drug that will never change.

Chlorthalidone

Thiazide diuretic - low K+, slight xerostomia, oral ulcerations - NSAIDs potentially decrease diuretic effectiveness

Tacrolimus clinical use

Transplant rejection prophylaxis

what test is commonly intradermal route?

Tuberculin skin test.

Clopidogrel

Unstable angina/ non ST-segment elevation MI, ST-segment elevation MI, recent MI/Stroke Dental considerations: After stent implantation, any elective surgery should be postponed for a year after the implantation

SNRIs

Venlafaxine, duloxetine

A. T-cells

Which cells are activated by aldesleukin (interleukin-2)? A. T-cells B. Red blood cells C. Undifferentiated cells D. Neutrophils

a, b, e

Which groups of individuals are at highest risk for development of iron deficiency anemia? (Select all that apply.) A. Women aged 12 to 40 B. Children C. Men aged 20 to 40 years D. Men over age 50 years E. Pregnant women

with all antidepressants

Xerostomia

ADME

absorption, distribution, metabolism, excretion.

Herpes Simplex Virus (HSV)

acyclovir (Zovirax) penciclovir (Denavir) valacyclovir (Valtrex) -Prodrug inactive drug is metabolized to active drug (acyclovir)

If a patient indicates they are taking rifampin or isoniazid only...

are likely taking it as a preventive agent

ud

as directed

ud

as directed (think you direct)

Nitrous oxide/oxygen sedation maintains patient in what Stage?

awake, Can talk open, close Stage I anesthesia

ac

before meals

Duration of action of anesthetics (longest)

bupivacaine

Drugs known to have narrow TI (more dangerous drugs)

carbamazepine, cyclosporine, DIGOXIN, levothyroxine, lithium, phenytoin, warfarin

anticholinergic effects on the eye?

causes dilation of pupil.

Direct acting Cholinergic agent includes?

choline derivatives and pilocarpine.

All opioids lead to

constipation... not diarrhea

antagonist drug?

counteracts the action of the agonist.

acetaminophen is contraindicated in

current or past alcoholics

qid

four times a day

List the fibrates

gemfibrozil, clofibrate, bezafibrate, fenofibrate

stat

immediately

SANS does what to pupil size?

increases. called mydriasis

BOARD ALERTS Local anesthetic is a weak base and works poorly where there is

infection due to the very acidic environment- increased dose may be needed

What route is the fastest drug response?

intravenous route.

epinephrine

is equal to adrenaline

weak acids are better absorbed when the Ph is _____ than the Pka

less

Topical Anesthetics benzocaine

most commonly used topical anesthetic

Less potent drug

need more to get an effect

When the half-life is short, the duration of action is?

short.

a person is suffering from a fever, arthritis, joint pain. the patient had started taking sulfonamides, this could be?

type 3 (immune complex/arthus/serum sickness) Mediated by IgG antibodies. the drug antigen antibody complex fixes complement and deposits int he vascular endothelium.

Side effect

undesirable (aka adverse reaction) *Expected response - but unwanted *Dose related *Nontarget organs

Potency and efficacy are

unrelated → drugs may have different potencies, but the same efficacy

Drugs with high first-pass effect require a larger oral dose as high percentage

will be deactivated.

Calcium channel blockers (class IV antiarrhythmics) clinical use

Prevention of nodal arrhythmias (e.g. SVT), rate control in atrial fibrillation

C. do not decrease the cytoprotective lining of the stomach.

The advantage of cyclooxygenase-2 (COX-2) inhibitors over other NSAIDs is that they A. have a longer duration of action. B. are less likely to cause hepatic toxicity. C. do not decrease the cytoprotective lining of the stomach. D. have a more rapid onset of action.

A Gingival hyperplasia

The nurse instructs a patient receiving phenytoin (Dilantin) to visit the dentist regularly and perform frequent oral hygiene. What common side effect is the nurse educating the patient about for this medication? A Gingival hyperplasia B Oral candidiasis C Increased risk of dental abscesses D Increased incidence of dental caries

a, c, d

The nurse is administering didanosine (Videx) to a client with HIV. Before administering this medication, the nurse should check which lab test results? Select all that apply. a. Elevated aspartate aminotransferase (AST) b. Elevated blood urea nitrogen (BUN) c. Elevated alanine aminotransferase (ALT) d. Elevated serum amylase e. Elevated serum creatinine

c. The corticosteroid reduces the inflammation and itching associated with ear infections.

The nurse is administering eardrops that contain a combination of an antibiotic and a corticosteroid. What is the rationale for combining these two drugs in eardrops? a. The corticosteroid reduces pain associated with ear infections. b. The combination works to help soften and eliminate cerumen. c. The corticosteroid reduces the inflammation and itching associated with ear infections. d. The drops help to eliminate fungal infections.

d. Hypocalcemia

The nurse is assessing the patient for the presence of a Chvostek's sign. What electrolyte imbalance does a positive Chvostek's sign indicate? a. Hypermagnesemia b. Hypercalcemia c. Hyperkalemia d. Hypocalcemia

a, b, c

The nurse is educating a patient who has been prescribed methylphenidate for narcolepsy about the drug's adverse effects. What potential adverse effects would the nurse include in the patient teaching? (Select all that apply.) A Weight Loss B Headache C Insomnia D Decreased blood pressure E Increased appetite

b. Perforated eardrum

The nurse is preparing to administer a new order for eardrops. Which is a potential contraindication to the use of many otic preparations? a. Ear canal itching b. Perforated eardrum c. Staphylococcus aureus otitis externa infection d. Escherichia coli ear infection

b Notify the physician and delay drug administration.

The nurse is preparing to administer an injection of morphine to a patient. Assessment notes a respiratory rate of 10 breaths/min. Which action will the nurse perform? a Administer a smaller dose and record the findings. b Notify the physician and delay drug administration. c Administer the prescribed dose and notify the physician. d Hold the drug, record the assessment, and recheck in 1 hour.

A. desmopressin (DDAVP)

The nurse is providing care to a patient following a non-accidental traumatic brain injury. The patient has developed diabetes insipidus due to the injury. What medication is most often used in the management of diabetes insipidus? A. desmopressin (DDAVP) B. corticotrophin (Acthar) C. octreotide (Sandostatin) D. somatropin (Humatrope)

a, b, d, e

The nurse is providing discharge teaching to an elderly patient with short-term memory problems. Which strategies will the nurse use in educating the patient? (Select all that apply.) a. Repeat information frequently. b. Provide written instructions for home use. c. Instruct patient to take all medications in the morning. d. Encourage use of daily medication containers with alarms. e. Perform several short teaching-learning sessions.

a, b, c, d, e, f

The nurse is providing medication education to a non-English-speaking patient. Who is qualified to act as a translator in this situation? (Select all that apply.) a. A certified translator b. A family member who does speak English c. Another health care provider who speaks the language d. Religious leader for the family e. A layperson versed in the person's language f. Translator system on computer/web-based

a, b, c

The nurse is providing patient teaching about antacids. Which statements about antacids are accurate? (Select all that apply.) a. Rebound hyperacidity may occur with calcium-based antacids. b. Antacids neutralize acid in the stomach. c. Magnesium-based antacids cause diarrhea. d. Antacids reduce the production of acid in the stomach. e. Aluminum-based antacids cause diarrhea.

a, c, d, e

The nurse is reviewing the uses of oral laxatives. Which conditions are general contraindications to or cautions about the use of oral laxatives? (Select all that apply.) a. Fecal impaction b. Ingestion of toxic substances c. Abdominal pain of unknown origin d. Acute abdominal pain e. Nausea and vomiting f. Irritable bowel syndrome

b. "Basal dosing delivers a constant dose of insulin."

The nurse is teaching a group of patients about management of diabetes. Which statement about basal dosing is correct? a. "With basal dosing, you can eat what you want and then give yourself a dose of insulin." b. "Basal dosing delivers a constant dose of insulin." c. "Basal-bolus dosing is the traditional method of managing blood glucose levels." d. "Glargine insulin is given as a bolus with meals."

d. Long-term use of these drugs may contribute to osteoporosis.

The nurse is teaching a patient who will be taking a proton pump inhibitor as long-term therapy about potential adverse effects. Which statement is correct? a. Proton pump inhibitors can cause diarrhea. b. These drugs can cause nausea and anorexia. c. Proton pump inhibitors cause drowsiness. d. Long-term use of these drugs may contribute to osteoporosis.

C. Seafood

The nurse is teaching the patient taking an antithyroid medication to avoid foods high in iodine. Which food will the nurse advise the patient against? A. Milk B. Eggs C. Seafood D. Chicken

b. Diabetes insipidus

The nurse notes in a patient's medication history that the patient has been taking desmopressin (DDAVP). Based on this finding, the nurse interprets that the patient has which disorder? a. Diabetes mellitus b. Diabetes insipidus c. Adrenocortical insufficiency d. Carcinoid tumor

B. Increased bleeding

The nurse would question the prescription for use of bismuth subsalicylate as an antidiarrheal in a patient on aspirin daily for anticoagulation therapy due to what potential concern? A. Nausea B. Increased bleeding C. Constipation D. Urinary retention

C. A 47-year-old patient with anuria

The nurse would question the use of mannitol for which patient? A. A 67-year-old patient who ingested a poisonous substance B. A 21-year-old patient with a head injury C. A 47-year-old patient with anuria D. A 55-year-old patient who receives cisplatin to treat ovarian cancer

A. "Draw up the regular insulin into the syringe first, followed by the cloudy NPH insulin."

The patient is prescribed 30 units regular insulin and 70 units NPH insulin subcutaneously every morning. The nurse will provide which instruction to the patient? A. "Draw up the regular insulin into the syringe first, followed by the cloudy NPH insulin." B. "Mixing insulins will help increase insulin production." C. "Rotate sites at least once weekly." D. "Use a 23- to 25-gauge syringe with a 1-inch needle for maximum absorption."

A fluid replacement.

The physician has ordered dopamine to treat the patient's hypovolemic shock secondary to severe blood loss. For the medication to be effective, the physician must also order A fluid replacement. B beta-stimulating drugs. C antibodies. D fluid restriction.

A Risk for decreased cardiac tissue perfusion related to effects of medication.

The priority nursing diagnosis for a patient taking metoprolol (Lopressor) would be A Risk for decreased cardiac tissue perfusion related to effects of medication. B Acute confusion related to adverse central nervous system effects of the drug. C Deficient knowledge related to therapeutic regimen. D Risk for injury related to possible side effects of the adrenergic blockers.

xerostomia

What is the major contraindication for the use of narcotic analgesics?

IBU has how many 'A' effects?

3, not antiplatelet

Brand Name: ProAir HFA

Generic Name: Albuterol

DOC for bulimia

SSRIs

mydriasis

dilation of pupils

q

every

Liver is the major site for drug

metabolism.

Class IB sodium channel blockers effect on action potential

-Decreases AP duration -Decreases slope of phase 0

Dobutamine

-β1 > β2, α direct agonist -Uses: heart failure (HF) (inotropic > chronotropic), cardiac stress testing.

Salmterol

-β2 > β1 direct agonist -Long term asthma or COPD control

epinephrine dental "cardiac dose"

0.04mg epi per dental appt ASA III or IV Pregnancy, diabetes, hypertension, etc

a TI of greater than ___ is usually needed to produce a therapeutically useful drug.

10

Function is lost in this order

1Autonomic 2temperature 3pain 4touch/pressure 5vibration and MOTOR (last thing to lose) o Regain function in the reverse order

1:200,000

=4 cartridges

D monoamine oxidase (MAO) inhibitors.

A hypertensive crisis may occur if adrenergic (sympathomimetic) drugs are given along with A beta blockers. B diuretics. C alpha1 blockers. D monoamine oxidase (MAO) inhibitors.

D. Tinea cruris

A male patient has been ordered clotrimazole (Lotrimin, Mycelex-G) after a diagnosis of jock itch. What is the technical name the nurse would use to document the diagnosis on the chart and the reason the patient has been prescribed this medication? A. Tinea corporis B. Tinea capitis C. Tinea pedis D. Tinea cruris

C. Dry mouth

A patient receiving an anticholinergic drug to treat nausea and vomiting should be taught to expect which side effect? A. Diarrhea B. Lacrimation C. Dry mouth D. Bradycardia

-pril

ACE inhibitor

Benazapril

ACE inhibitor (-pril) -HA, dizziness, hypotension, loss of taste, oral ulcers, cough, orofacial angioedema, scalded mouth syndrome -NSAIDs potentially decrease effectiveness, caution with position change, watch for hyperkalemia

Enalapril

ACE inhibitor (-pril) -HA, dizziness, hypotension, loss of taste, oral ulcers, cough, orofacial angioedema, scalded mouth syndrome -NSAIDs potentially decrease effectiveness, caution with position change, watch for hyperkalemia

Somatostatin (octeotride) clinical use

Acromegaly, carcinoid syndrome, gastrinoma, glucagonoma, esophageal varices

Morphine mechanism

Act as agonists at opioid receptors (μ = full, κ = full, decreased potency) to modulate synaptic transmission—open K+ channels, close Ca2+ channels decreasing synaptic transmission.

C. 2 mg

An adult patient presents to the emergency room with insecticide poisoning. The nurse prepares to administer which dose of atropine intravenously? A. 0.3 mg B. 0.5 mg C. 2 mg D. 4 mg

b. Hypotension

An elderly patient will be taking a vasodilator for hypertension. Which adverse effect is of most concern for the older adult patient taking this class of drug? a. Restlessness b. Hypotension c. Constipation d. Dry mouth

Nitrate clinical use

Angina, acute coronary syndrome, pulmonary edema

Apixaban (Eliquis)

Anticoagulation (DOAC), factor Xa Major bleed 1.5-1.7%, easy bruising NO NSAID no antidote, prevents more strokes per year than Warfarin and leads to fewer bleeds and deaths

Type III:

Arthus - serum sickness

Name the rapid acting insulins

Aspart, glulisine, lispro

Best way to reduce systemic toxicity from local anesthetic is:

Aspiration and inject slowly fast: finds path off least resistance ie: blood Stream

Analgesics Contraindicated in pregnancy

Aspirin NSAIDS

Heparin toxicity

Bleeding, thrombocytopenia (HIT), osteoporosis, drug-drug interactions. For rapid reversal (antidote), use protamine sulfate (positively charged molecule that binds negatively charged heparin).

Nifedipine

CCB -gingival overgrowth, position change, drug intx including the potentially fatal intx with diltiazem/verapamil and macrolides

Local anesthetic toxicity

CNS excitation, severe cardiovascular toxicity (bupivacaine), hypertension, hypotension, arrhythmias (cocaine), methemoglobinemia (benzocaine).

Class IB sodium channel blockers toxicity

CNS stimulation/depression, CV depression

Cyclosporine MoA

Calcineurin inhibitor; binds cyclophilin. Blocks T-cell activation by preventing IL-2 transcription.

Yes

Can a prescription for a Schedule III drug be called in?

When two formulations of a drug meet the chemical and physical standards established by the regulatory agencies

Chemically equivalent

Low potency typical antipsychotics

Chlorpromazine, thioridazine

Thiazide diuretics (drugs, mechanism, use, toxicity)

Chlorthalidone, hydrochlorothiazide. -Mechanism: inhibit NaCl reabsorption in early DCT decreasing diluting capacity of nephron. Decreased Ca2+ excretion. -Use: hypertension, HF, idiopathic hypercalciuria, nephrogenic diabetes insipidus, osteoporosis. -Toxicity: hypokalemic metabolic alkalosis, hyponatremia, hyperGlycemia, hyperLipidemia, hyperUricemia, hyperCalcemia. Sulfa allergy.

List the bile acid resins

Cholestyramine, colestipol, colesevelam

Ribavirin clinical use

Chronic HCV, also used in RSV (palivizumab preferred in children)

Warfarin use

Chronic anticoagulation (e.g., venous thromboembolism prophylaxis, and prevention of stroke in atrial fibrillation). Not used in pregnant women (because warfarin, unlike heparin, crosses placenta). Follow PT/INR.

Clinical use of interferon-αlpha

Chronic hepatitis B and C, Kaposi sarcoma, malignant melanoma

Class IA sodium channel blocker toxicity

Cinchonism (headache, tinnitus with quinidine), reversible SLE like syndrome (procainamide), heart failure (disopyramide), thrombocytopenia, torsades de pointes due to increased QT

Name the -azoles

Clotrimazole, fluconazole, itraconazole, ketoconazole, miconazole, voriconazole.

B. Pernicious anemia

Cyanocobalamin (vitamin B12) is indicated for the treatment of which condition? A. Acute lymphocytic leukemia B. Pernicious anemia C. Bone marrow suppression D. Optic nerve atrophy

Guaifenesin

Expectorant—thins respiratory secretions; does not suppress cough reflex.

Type I:

Immediate hypersensitivity- anaphylaxis

What is the prophylaxis for M. tuberculosis?

Isoniazid

Drugs that cause diarrhea

Metformin, Erythromycin, Colchicine, Orlistat, Acarbose

Clinical use of local anesthetics

Minor surgical procedures, spinal anesthesia. If allergic to esters, give amides

Eculizumab clinical use

Paroxysmal nocturnal hemoglobinuria

C. Bone pain

Patients receiving filgrastim (Neupogen) should be taught to expect which side effect? A. Hypotension B. Constipation C. Bone pain D. Insomnia

b. distribution rates among various body compartments.

Pharmacokinetics involves the study of a. physiologic interactions of drugs. b. distribution rates among various body compartments. c. interactions between various drugs. d. adverse reactions to medications.

Drugs that cause gingival hyperplasia

Phenytoin, Ca2+ channel blockers, cyclosporine

Simeprevir toxicity

Photosensitivity reactions, rash

Toxoplasmosis therapy

Pyrimethamine

OPIOID NARCOTIC ANALGESICS Adverse Reactions

Respiratory depression • Usually cause of death with overdose

Drugs compete for the same receptor sites. The ones that win are considered what?

The drug with the stronger affinity will win. they are also more potent than drugs with weaker affinity.

Poorly

The more ionized a drug is, the more _____poorly it is absorbed.

B. Administer aspirin 30 minutes before nicotinic acid.

The nurse plans which intervention to decrease the flushing reaction of niacin? A. Administer niacin with an antacid. B. Administer aspirin 30 minutes before nicotinic acid. C. Administer diphenhydramine hydrochloride (Benadryl) with the niacin. D. Apply cold compresses to the head and neck.

a. Anxiety disorder

The nurse reads in the patient's medication history that the patient is taking buspirone (BuSpar). The nurse interprets that the patient may have which disorder? a. Anxiety disorder b. Bipolar disorder c. Schizophrenia d. Depression

B Therapeutic

The nurse receives lab values for a patient with a theophylline level of 14 mcg/mL. How does the nurse interpret this theophylline level? A Subtherapeutic B Therapeutic C Toxic D Life-threatening

C. Atrial fibrillation

The nurse would question an order for pseudoephedrine in a patient with a history of which condition? A. Pneumonia B. Osteoporosis C. Atrial fibrillation D. Peptic ulcer disease

B. Acromegaly

The nurse would question an order for somatrem (Protropin) in a patient with which condition? A. Dwarfism B. Acromegaly C. Growth failure D. Hypopituitarism

Ceiling effect

The point at which increasing the dose of a drug no longer increases the effect of the drug is referred to as _____.

pharmacokinetics

What the body does to the drug to break it down and get rid of it.

B. Wheat bran

When teaching a patient regarding the administration of digoxin, the nurse instructs the patient not to take this medication with which food? A. Bananas B. Wheat bran C. French toast D. Scrambled eggs

hs

at bedtime

PO

by mouth

opioids,

narcotic, central, or strong analgesics.

bid

twice a day

c. Malignant hyperthermia

While monitoring a patient who had surgery under general anesthesia 2 hours ago, the nurse notes a sudden elevation in body temperature. This finding may be an indication of which problem? a. Malignant hypertension b. Tachyphylaxis c. Malignant hyperthermia d. Postoperative infection

a. Hives

While the nurse is taking an admission history, the patient reports having a previous allergic reaction to penicillins. What will the nurse document as part of the patient's allergic reaction response to penicillins? a. Hives b. Dry eyes c. Frequent urination d. Constipation

U.S. Adoptive Names Council

Who determines the generic name for a drug?

macrolide antibiotic

What pharmacologic category does azithromycin belong to?

antilipemic agent

What pharmacologic category is Lipitor?

a Medications given intravenously are not affected by the first-pass effect.

When a patient asks the nurse why a lower dose of intravenous pain medication is being given than the previous oral dose, what knowledge will the nurse draw on to respond to the patient? a Medications given intravenously are not affected by the first-pass effect. b Medications given orally bypass the portal circulatory system. c A large percentage of an intravenously administered drug is metabolized into inactive metabolites in the liver. d Drugs administered intravenously enter the portal system prior to systemic distribution.

b. Serum creatinine

When caring for the client who is receiving an aminoglycoside antibiotic, the nurse should monitor which of the following laboratory values? a. Serum calcium b. Serum creatinine c. Serum potassium d. Serum sodium

D. Change in level of consciousness

When developing a nursing care plan for a patient receiving epoetin alfa (Epogen), the nurse will include monitoring for which adverse effect? A. Chronic diarrhea B. Severe hypotension C. Impaired liver function D. Change in level of consciousness

A. They influence carbohydrate, fat, and protein metabolism.

When discussing glucocorticoids, what statement is accurate in relation to the action of these medications? A. They influence carbohydrate, fat, and protein metabolism. B. They are produced in decreased amounts during times of stress. C. They decrease serum sodium and glucose levels. D. They stimulate defense mechanisms to produce immunity.

a Ask specific questions to evaluate understanding.

When the nurse completes patient teaching on a new medication, which method will the nurse use to evaluate the patient's understanding of the information? a Ask specific questions to evaluate understanding. b Observe the patient taking the medication. c Assess the patient for response to the medication. d Document the education session in the patient chart.

a, b, c, e

Which are common side effects of fenofibrate (Tricor), a fibric acid derivative? (Select all that apply.) A. Nausea, vomiting, and abdominal pain B. Increase in gallstone formation C. Impotence D. Constipation E. Rash

a, b, c, e

Which are known contraindications for use of erythropoiesis stimulation agents in patients? (Select all that apply.) A. Uncontrolled hypertension B. Cancer of the neck C. History of thrombosis D. Renal failure E. Hemoglobin level of 15 g/dL

d. is over 60 years of age

Which of the following clients should receive shingeles vaccine (Zostavax)? A client who... a. has never had chicken pox b. is at risk for genital herpes c. has a compromised immune system d. is over 60 years of age

What is the median lethal does? LD50

the dose that kills 50% of experimental animals.

Therapeutic index: lethal dose divided by

the effective dose (LD50 / ED50)

Macrolides

(-mycin) -Closely resemble penicillin -Broad spectrum and very safe •erythromycin Increase effect of warfarin (increased bleeding) most common side effect is GI upset •azithromycin & clarithromycin

Examples: ibuprofen

(Advil, Motrin) drug of choice for dental pain

Examples: naproxen

(Aleve) longer half-life - given on 8-12 hour schedule

Finasteride

-A 5α-reductase inhibitor ( conversion of testosterone to DHT). -Useful in BPH and male-pattern baldness

Galantamine

-Anticholinesterse - increases ACh -Alzheimer disease

Drugs for complex seizure

-Carbamazepine is 1st choice -All other epilepsy drugs can be used except for ethosuxamide and the benzodiazepines

Calcium channel blockers (class IV antiarrhythmics) mechanism

-Decrease conduction velocity -Increase ERP and PR interval

Niacin mechanism

-Inhibits lipolysis (hormone sensitive lipase) in adipose tissue; -Reduces hepatic VLDL synthesis

Selegiline

-Mechanism: blocks conversion of dopamine by selectively inhibiting MAO-B. -Use: Parkinson disease therapy

Epinephrine (α1 agonist) as a glaucoma drug

-Mechanism: decrease aqueous humor synthesis via vasoconstriction -Side effects: mydriasis (α1); do not use in closed-angle glaucoma. Blurry vision, ocular hyperemia, foreign body sensation, ocular allergic reactions, ocular pruritus

Oxybutynin

-Muscarinic antagonist -Reduced bladder spasms and urge urinary incontinence

α-glucosidase inhibitors clinical use

-Used as monotherapy in type 2 diabetes or in combination with other agents

S&S of Narcolepsy

-falling asleep during normal waking hours -sleep paralysis (muscle paralysis) -unable to move -collapse

B-Blockers commonly end in

-olol

ACE Inhibitors commonly end in

-pril

what are the four functions divided in the ANS drugs?

1. a drug that stimulates the Pans is called P+ (cholinergic or parasympathomimetic) 2. a drug that blocks Pans is called P- ( anticholinegeric, parasympatholytic, or cholinergic blockers) 3. a drug that stimulates the Sans is called S+ (sympathomimetic or adrenergic) a drug that blocks the SANS is called S- (adrenergic blockers, sympathetic blockers, or sympatholytic)

a, b, c

A patient receiving valproic acid (Depakote) should be monitored for which adverse effects? (Select all that apply.) A Tremors B Weight gain C Hepatoxicity D Hypoglycemia E Insomnia

B diazepam (Valium)

A postoperative craniotomy patient is received in the intensive care unit. The nurse makes sure which prescribed drug is readily available to treat acute seizure activity? A gabapentin (Neurontin) B diazepam (Valium) C ethosuximide (Zarontin) D flumazenil (Romazicon)

Schedule I

Abuse Potential highest • No accepted medical use heroin, LSD, marijuana - (Still cat.1)

What are the adverse effects of adenosine?

Adverse effects include flushing, hypotension, chest pain, sense of impending doom, bronchospasm.

Typical antipsychotic mechanism

All typical antipsychotics block dopamine D2 receptors ( [cAMP])

Drugs that cause Stevens-Johnson syndrome

Anti-epileptic drugs (especially lamotrigine), allopurinol, sulfa drugs, penicillin

Sotalol

Antiarrhythmic - QT prolongation, bradycardia, chest pain, fatigue

Digoxin

Antiarrhythmic - anorexia, GI, HA, bradycardia - levels increased by BSDP, erythromycin, tetracycline, ibuprofen

Flecainide

Antiarrhythmic -bradycardia, dizziness, HA, GI, neutropenia

PCP intoxication

Belligerence, impulsivity, fever, psychomotor agitation, analgesia, vertical and horizontal nystagmus, tachycardia, homicidality, psychosis, delirium, seizures. Treatment: benzodiazepines, rapid-acting antipsychotic

amlodipine

CCB -gingival overgrowth, position change, drug intx including the potentially fatal intx with diltiazem/verapamil and macrolides

Adverse effect of fluorquinolones during pregnancy

Cartilage damage

C. Dual-action alpha1 and beta receptor blocker.

Carvedilol (Coreg) is classified as a A. beta blocker. B. alpha1 blocker. C. Dual-action alpha1 and beta receptor blocker. D. Calcium channel blocker.

Overuse of aluminum hydroxide

Causes constipation and hypophosphatemia; proximal muscle weakness, osteodystrophy, seizures

Prevention of postsurgical infection due to S. aureus

Cefazolin

1st generation cephalosporins

Cefazolin, cephalexin Use: Gram- positive cocci, Proteus mirabilis, E. coli, Klebsiella pneumoniae. Cefazolin used prior to surgery to prevent S. aureus wound infections

Exposure to meningococcal infection

Ceftriaxone, ciproflaxacin, or rifampin

ADP receptor inhibitors (drugs, mechanism, use, toxicity)

Clopidogrel, prasugrel, ticagrelor (reversible), ticlopidine. -Mechanism: inhibit platelet aggregation by irreversibly blocking ADP receptors. Prevent expression of glycoproteins IIb/IIIa on platelet surface. -Use: acute coronary syndrome; coronary stenting. Decreased incidence or recurrence of thrombotic stroke -Toxicity: neutropenia (ticlopidine). TTP may be seen.

Drugs that cause coronary vasospasm

Cocaine, sumatriptan, ergot alkaloids

Complications of depolarizing neuromuscular blocking drugs

Complications include hypercalcemia, hyperkalemia, malignant hyperthermia.

Drugs that cause osteoporosis

Corticosteroids, heparin

Drugs that cause hemorrhagic cystitis

Cyclophosphamide, ifosfamide

Type II:

Cytotoxic/cytolytic - lysis of red blood cells

Dabigatran (Pradaxa)

DOAC, direct fibrin/thrombin inhibitor - NO NSAID - GI bleeding, monitor with ECT or PTT - NO ANTIDOTE AND CANT MONITOR - fewer food and drug intrx. compared to Warfarin - Bleeding risk same as Warfarin

What are the major functions of the αlpha-2 receptor?

Decrease sympathetic outflow, decrease insulin release, decrease lipolysis, increase platelet aggregation, decrease aqueous humor production

What effect do nitrates have on BP?

Decreases

Treatment strategy with type 2 DM

Dietary modification and exercise for weight loss; oral agents, non-insulin injectables, insulin replacement

Digoxin mechanism

Direct inhibition of Na+/K+ ATPase causing indirect inhibition of Na+/Ca2+ exchanger. Increased [Ca2+]i creates positive inotropy. Stimulates vagus nerve to increase HR.

B. acetaminophen

Discharge teaching for a patient receiving glucocorticoids would include the preferred use of which medication for pain management? A. aspirin B. acetaminophen C. ibuprofen D. naproxen

A. Muscle pain

Discharge teaching for a patient receiving simvastatin (Zocor) would include the importance of reporting which symptoms that might indicate a serious adverse reaction to the medication? A. Muscle pain B. Headache C. Weight loss D. Fatigue

Antagonist

Drug counteracts the action of agonist —blocks receptor

Medicine

Drug used to diagnose, treat, or prevent disease.

B phentolamine (Regitine)

During assessment of a patient diagnosed with pheochromocytoma, the nurse auscultates a blood pressure of 210/110 mm Hg. The nurse would expect to administer which medication? A nadolol (Corgard) B phentolamine (Regitine) C dobutamine (Dobutrex) D verapamil (Calan)

Metoprolol side effects

Dyslipidemia

Prevention of gonococcal conjunctivitis in newborn

Erythromycin ointment

d transfer care of a patient to another professional nurse if caring for the patient would violate personal ethical principles.

Ethically, a nurse has the responsibility to a employ beneficence, the duty to do no harm to a patient. b withhold information from the patient as requested by the family. c impose his or her own values upon the patient when doing so would help the patient. d transfer care of a patient to another professional nurse if caring for the patient would violate personal ethical principles.

List the class IC sodium channel blockers

Flecainide, propafenone

Benzodiasepines toxicity antidote

Flumazenil

Bile acid resin side effects/problems

GI upset, decrease absorption of other drugs and fat-soluble vitamins

Drugs that cause agranulocytosis

Ganciclovir, Clozapine, Carbamazepine, Colchicine, Methimazole, Propylthiouracil

Brand Name: Lexapro

Generic Name: Escitalopram

Brand Name: Nexium

Generic Name: Esomeprazole

-tidine

H2-antagonist

Aspirin mechanism

Irreversibly inhibits cyclooxygenase (both COX-1 and COX-2) enzyme by covalent acetylation. Platelets cannot synthesize new enzyme, so effect lasts until new platelets are produced: increase bleeding time, decrease TXA2 and prostaglandins. No effect on PT or PTT.

Spironolactone

K+ sparing diuretic

Triamterene

K+ sparing diuretic

Ganciclovir toxicity

Leukopenia, neutropenia, thrombocytopenia, renal toxicity. More toxic to host enzymes than acyclovir.

Amides (local anesthetics)

Lidocaine, mepIvacaIne, bupIvacaIne (amIdes have 2 I's in name)

DOC for bipolar disorder

Lithium, valproic acid, atypical antipsychotics

Antihelminthic therapy drug regimen

Mebendazole, pyrantel pamoate, ivermectin, diethylcarbamazine, praziquantel.

Interferons toxicity

Neutropenia, myopathy.

c 10 to 12 years.

New drugs must go through extensive research and testing before approval for use in humans. The nurse is providing education to a patient on a new medication and will inform the patient that the average length of time a medication is researched before being prescribed for humans is a 2 to 4 years. b 6 to 8 years. c 10 to 12 years. d 14 to 16 years.

Subscription

On a prescription, this is directions to the pharmacist:

Signa

On a prescription, this is dosage information:

Atypical antipsychotics use

Schizophrenia—both positive and negative symptoms. Also used for bipolar disorder, OCD, anxiety disorder, depression, mania, Tourette syndrome.

C. Erythropoietin

The formation of erythrocytes and maturation of the red blood cell (RBC) is driven by what hormone? A. Progesterone B. Testosterone C. Erythropoietin D. Free thyroxin

better

The more lipid soluble a drug is, the ______ it is absorbed.

c, d, e

The nurse is discussing use of antihistamines for allergic rhinitis. Which medications would be included in the list of H1 antagonists used in the treatment of allergic rhinitis? (Select all that apply.) A. ranitidine (Zantac) B. nizatidine (Axid) C. fexofenadine (Allegra) D. loratadine (Claritin) E. cetirizine (Zyrtec)

a, c, d

The nurse is educating a patient newly diagnosed with open-angle glaucoma on the adverse effects of the indirect-acting miotic eye drops prescribed. What will the nurse include in the teaching as possible adverse effects? (Select all that apply.) A. Blurred vision B. Farsightedness C. Stinging on instillation D. Decreased nighttime vision E. Paralysis of eyelids

D) Hepatitis

The nurse would question the use of kava in a patient with a history of which condition? A) Anxiety B) Hypertension C) Cardiovascular disease D) Hepatitis

b Intravenously

To achieve the most rapid onset of action, the health care provider will prescribe the medication to be administered by which route? a Subcutaneously b Intravenously c Intramuscularly d Intrathecally

Azathioprine clinical use

Transplant rejection prophylaxis, rheumatoid arthritis, Crohn disease, glomerulonephritis, other autoimmune conditions.

Oseltamivir, zanamivir clinical use

Treatment and prevention of both influenza A and B.

Albuterol

Treatment and prevention of bronchospasm Dental considerations: Xerostomia

amlodipine

What is the generic name for Norvasc?

High cholesterol

What is the major use for Lipitor?

c. Bradycardia

When monitoring a patient who is taking a cholinergic drug, the nurse will watch for which cardiovascular effect? a. Vasoconstriction b. Palpitations c. Bradycardia d. Tachycardia

a, b

When teaching a patient about carvedilol (Coreg), the nurse explains that this medication reduces blood pressure by which actions? (Select all that apply.) A. Reducing heart rate B. Vasodilation C. Decreasing stress D. Increasing urine output

a, b, c,

When teaching a patient regarding desmopressin (DDAVP), the nurse will inform the patient to monitor for which potential side effects? (Select all that apply.) A. Headache B. Weight gain C. Nasal irritation D. Hyperglycemia E. Hypotension

C Eating aged cheese

Which activity should the patient be cautioned to avoid while taking an MAO inhibitor? A Participating in a bowling league B Sunbathing at the pool C Eating aged cheese D Smoking a low-nicotine cigarette

a, c, d

Which conditions is aminoglutethimide used to treat? (Select all that apply.) A. Cushing's syndrome B. Testicular cancer C. Adrenal cancer D. Metastatic breast cancer E. Thyroid cancer

Potency-

a function of the amount of the drug required to produce an effect.

What is the Principal mediator in the PANS?

acetylcholine

PRN

as required , if needed.

Which drug binds irreversibly to platelets, which drug binds reversibly?

aspirin irreversibly, and NSAIDS reversibly.

How does cholinergic drugs effect the cardiovascular?

decrease in cardiac output. relaxation and vasodilation. the indirect effect is an increase in heart rate and cardiac output. because the effects are opposite, the effect will depend on concentration of drug. generally causes bradycardia and decrease BP and cardiac output

Drug excretion elimination terminates

drug effect.

h

hour

in the presence of Infection, the acidity of the tissue...

increases (the ph decreases), and the effect of local anesthetics decreases.

What drug commonly uses subcutaneous route?

insulin

what is schedule 4 handling procedures?

less abuse potential. valium, darvon. prescriptions may be telephoned; no more than five in 6 months.

Topical Anesthetics Огаqix

lidocaine and prilocaine (2.5% : 2.5%) gel applied in periodontal pocket

Tetracyclines Examples (oral administration):

o tetracycline o doxycycline atridox o minocycline Arestin

The autonomic nervous system functions as an automatic modulating system. what are some examples it affects?

regulation of blood pressure, heart rate, gastrointestinal tract motility, salivary gland secretions, and bronchial smooth muscle.

ibuprofen can decrease

the effects of many drugs:

Rapid acting insulin mechanism

-Binds insulin receptor (tyrosine kinase activity) -Liver: increase glucose stored as glycogen -Muscle: increase glycogen, protein synthesis; increase K+ uptake -Fat: increase TG storage

Clindamycin

-Blocks peptide transfer (translocation) at 50S ribosomal subunit. Bacteriostatic. -Anaerobic infections (e.g., Bacteroides spp., Clostridium perfringens) in aspiration pneumonia, lung abscesses, and oral infections. Also effective against invasive group A streptococcal infection. -Treats anaerobic infections above the diaphragm vs. metronidazole (anaerobic infections below diaphragm) -Toxicity: pseudomembranous colitis (C. difficile -overgrowth), fever, diarrhea

Contraindications to nitrous oxide / oxygen sedation:

-COPD -Upper respiratory infections (stuffy nose) -Emotional /behavioral instability -Pregnancy -Cystic fibrosis -Language barrier

Simeprevir clinical use

-Chronic HCV in combination with ribavirin and peginterferon alfa. -Do not use as monotherapy.

Dopamine

-D1 = D2 > β > α direct agonist -Uses: unstable bradycardia, HF, shock; inotropic and chronotropic α effects predominate at high doses.

β-blockers (class II antiarrhythmics) mechanism

-Decrease SA and AV nodal activity by decreasing cAMP, and decreasing Ca2+ currents. Suppress abnormal pacemakers by decreasing slope of phase 4. -AV node particularly sensitive - increase PR interval. Esmolol very short acting

Isoniazid

-Decrease synthesis of mycolic acids. Bacterial catalase- peroxidase (encoded by KatG) needed to convert INH to active metabolite. -Use in Mycobacterium tuberculosis. The only agent used as solo prophylaxis against TB. -Toxicity: Neurotoxicity, hepatotoxicity. Pyridoxine (vitamin B6) can prevent neurotoxicity. -Resistance: mutations leading to underexpression of KatG.

Vasoconstrictors - Epinephrine Added to anesthetic for:

-Decreased systemic absorption - Lower risk of toxicity -Decreased absorption -Increased duration of action -Decreased bleeding

Class IA sodium channel blockers effect on action potential

-Decreases slope of phase 0 -Increase action potential duration -Increase effective refractory period -Increased QT interval

Cancer drugs - cell cycle

-G1: alkylating agents (carmustine, cisplatin, lomustine) -S: antimetabolites (azanthroprine, cladribine, cytarabine, 5-fluouracil, hydroxyurea, methotrexate, 6-MP, 6-thioguanine), also etoposide, teniposide -G2: bleomycin, etoposide, teniposide -M: microtubule inhibitors (paclitaxel), vinca alkaloids (vinblastine, vincristine)

Aspirin toxicity

-Gastric ulceration, tinnitus (CN VIII). Chronic use can lead to acute renal failure, interstitial nephritis, and upper GI bleeding. Reye syndrome in children with viral infection. -Overdose initially causes hyperventilation and respiratory alkalosis, but transitions to mixed metabolic acidosis-respiratory alkalosis.

Protease inhibitor toxicity

-Hyperglycemia, GI intolerance (nausea, diarrhea), lipodystrophy. -Nephropathy, hematuria (indinavir). -Rifampin (a potent CYP/UGT inducer) contraindicated with protease inhibitors because it can decrease protease inhibitor concentration.

Glucocorticoids toxicity

-Hyperglycemia, osteoporosis, central obesity, muscle breakdown, psychosis, acne, hypertension, cataracts, avascular necrosis. -Can cause iatrogenic Cushing syndrome.

Urine Ca2+ changes with diuretic therapy

-Increase with loop diuretics: decrease paracellular Ca2+ reabsorption causing hypocalcemia -Decreases with thiazides: enhanced Ca2+ reabsorption in DCT

Vancomycin

-Inhibits cell wall peptidoglycan formation by binding D-ala D-ala portion of cell wall precursors. Bactericidal. Not susceptible to β-lactamases. -Gram-positive bugs only—serious, multidrug-resistant organisms, including MRSA, S. epidermidis, sensitive Enteroccocus species, and Clostridium difficile (oral dose for pseudomembranous colitis). -Well tolerated in general—but NOT trouble free. Nephrotoxicity, Ototoxicity, Thrombophlebitis, diffuse flushing—red man syndrome (can largely prevent by pretreatment with antihistamines and slow infusion rate). -Resistance occurs in bacteria via amino acid modification of D-ala D-ala to D-ala D-lac. "Pay back 2 D-alas (dollars) for vandalizing (vancomycin)."

Warfarin mechanism

-Interferes with γ-carboxylation of vitamin K- dependent clotting factors II, VII, IX, and X, and proteins C and S. -Metabolism affected by polymorphisms in the gene for vitamin K epoxide reductase complex (VKORC1). In laboratory assay, has effect on EXtrinsic pathway and increase PT. -Long half-life.

Sirolimus (Rapamycin) clinical use

-Kidney transplant rejection prophylaxis. -Synergistic with cyclosporine. -Also used in drug- eluting stents

Baclofen

-Mechanism: inhibits GABA-B receptors at spinal cord level, inducing skeletal muscle relaxation. -Use: muscle spasms (e.g., acute low back pain).

2nd generation H1 blockers mechanism, use, and toxicity

-Mechanism: reversible inhibitors of H1 histamine receptors. -Use: Allergy. -Toxicity: far less sedating than 1st generation because of decreased entry into CNS.

1st generation H1 blockers mechanism, use, and toxicity

-Mechanism: reversible inhibitors of H1 histamine receptors. -Use: allergy, motion sickness, sleep aid. -Toxicity: sedation, antimuscarinic, anti-α-adrenergic.

Guanfacine

-Mechanism: works like clonidine→ CNS postsynaptic α-2A receptor agonist; ↑noradrenergic effects directly; phenobarbitol & phenytoin may ↓plasma levels -Use: improves attention, concentration, execution, wakefulness, hyperactivity; often used when too activated or oppositional→ tics, emotional outbursts -Toxicity: -Somnolence, headache, fatigue, upper abdominal pain, sedation, HoTN, dry mouth & constipation -CI: don't use w/ other sedative drugs, caution w/ drug inducers & inhibitors

Pseudoephedrine, phenylephrine (mechanism, use, toxicity)

-Mechanism: α-adrenergic agonists, used as nasal decongestants. -Use: reduce hyperemia, edema, nasal congestion; open obstructed eustachian tubes. Pseudoephedrine also illicitly used to make methamphetamine. -Toxicity: hypertension. Can also cause CNS stimulation/anxiety (pseudoephedrine).

Methylphenidate

-Mechanism: ↑dopamine & NE tone by blocking their reuptake & facilitating their release; -Use: ADHD; narcolepsy

Generic name

-Official name of drug -Lower case -Only one generic name -Active ingredient must enter bloodstream at same rate as trade name

Amoxicillin, ampicillin (aminopenicillins)

-Penicillinase-sensitive penicillins -Same mechanism as penicillin (inhibits peptidoglycan cross-linking) with wider spectrum; -Penicillinase sensitive (ombine with clavulanic acid to protect against destruction by β-lactamase) -Use: extended-spectrum penicillin—H. influenzae, H. pylori, E. coli, Listeria monocytogenes, Proteus mirabilis, Salmonella, Shigella, enterococci. -Toxicity: Hypersensitivity reactions; rash; pseudomembranous colitis. -Resistance: penicillinase in bacteria (a type of β-lactamase) cleaves β-lactam ring.

ANTIHYPERTENSIVE Alpha Blockers (a -Adrenergic Blocking Agents)

-produce peripheral vasodilation - reducing vascular resistance - lowering BP -Also used to benign prostatic hypertrophy used in men that have prostate problems and high blood pressure -Effectiveness reduced by NSAIDS Adverse Reactions -Orthostatic hypotension

Norepinephrine

-α1 > α2 > β1 direct agonist -Hypotension (butrenal perfusion). Significantly weaker effect at β2-receptor than epinephrine.

Phenylephrine

-α1 > α2 direct agonist -Uses: hypotension (vasoconstrictor), ocular procedures (mydriatic), rhinitis (decongestant)

Epinephrine

-β > α direct agonist -Uses: anaphylaxis, asthma, open-angle glaucoma; α effects predominate at high doses. Significantly stronger effect at β2-receptor than norepinephrine.

Mechanism of action of cephalosporins

-β-lactam drugs that inhibit cell wall synthesis but are less susceptible to penicillinases. Bactericidal. -Organisms typically not covered by cephalosporins are LAME: Listeria, Atypicals (Chlamydia, Mycoplasma), MRSA, and Enterococci. Exception: ceftaroline covers MRSA.

Isoprterenol

-β1 = β2 direct agonist -Uses: electrophysiologic evaluation of tachyarrhythmias. Can worsen ischemia

B. Decreased urinary frequency

7. After administering oxybutynin (Ditropan) to a patient with spina bifida, the nurse is assessing the patient for therapeutic effects. What is the nurse assessing for in the patient? A. Decreased muscle twitches B. Decreased urinary frequency C. Increased bowel movements D. Increased heart rate

b. Osmosis and osmolality

A nurse in the medical ICU has orders to infuse a hypertonic solution into her patient with low blood pressure. This solution will increase the number of dissolved particles in the patient's blood, creating pressure for fluids in the tissues to shift into the capillaries and increase the blood volume. Which term or terms is/are associated with this process? a. Hydrostatic pressure b. Osmosis and osmolality c. Diffusion d. Active transport

B St. John's wort

A patient currently prescribed duloxetine (Cymbalta) comes to the health clinic complaining of restlessness, agitation, diaphoresis, and tremors. The nurse suspects serotonin syndrome and questions the patient regarding concurrent use of which substance? A ibuprofen B St. John's wort C vitamin E supplements D glucosamine chondroitin

b, d, e

A patient tells the nurse that she wants to start taking the herbal product goldenseal to improve her health. The nurse will assess for which potential cautions or contraindications to goldenseal? (Select all that apply.) a. Sinus infections b. Taking a proton-pump inhibitor c. Nasal congestion d. Pregnancy e. Hypertension f. Hypothyroidism

A. Candidiasis

A patient who is prescribed the drug miconazole (Monistat), a topical antifungal, has most likely been diagnosed with which condition? A. Candidiasis B. Pruritus C. Eczema D. Folliculitis

Ramipril

ACE inhibitor (-pril) -HA, dizziness, hypotension, loss of taste, oral ulcers, cough, orofacial angioedema, scalded mouth syndrome -NSAIDs potentially decrease effectiveness, caution with position change, watch for hyperkalemia

Drugs that cause dry cough

ACE inhibitors

D. preventing aldosterone secretion.

ACE inhibitors and angiotensin receptor blockers both work to decrease blood pressure by A. preventing the formation of angiotensin II. B. enhancing sodium and water resorption. C. increasing the breakdown of bradykinin. D. preventing aldosterone secretion.

Atomoxetine

ADD/ADHD - GI, anorexia, dizziness, mood swings, no abuse - may cause seizures, nervousness, insomnia, dizziness, HA, dyskinesia, tachycardia, anorexia

LIsdexamfetamine

ADD/ADHD - GI, anorexia, dizziness, mood swings, no abuse - may cause seizures, nervousness, insomnia, dizziness, HA, dyskinesia, tachycardia, anorexia

Methylphenidate

ADD/ADHD - GI, anorexia, dizziness, mood swings, no abuse - may cause seizures, nervousness, insomnia, dizziness, HA, dyskinesia, tachycardia, anorexia

Drugs that cause nephrotoxicity/ototoxicity

Aminoglycosides, vancomycin, loop diuretics, cisplatin. Cisplatin toxicity may respond to amifostine.

Digitalis toxicity antidote

Anti-dig Fab fragments

C. 5:00 PM

Assuming the patient eats breakfast at 8:30 AM, lunch at noon, and dinner at 6:00 PM, he or she is at highest risk of hypoglycemia following an 8:00 AM dose of NPH insulin at what time? A. 10:00 AM B. 2:00 PM C. 5:00 PM D. 8:00 PM

RESPIRATORY DRUGS

Asthma is Reversible airway obstruction -Need short acting B2-agonist produces bronchodilation -albuterol (ProAir, Proventil) -metaproterenol inhaled corticosteroids: fluticasone (Flovent)

OSTEOPOROSIS DRUGS

Bisphosphonates are most common used -alendronate (Fosamax) -ibandronate (Boniva) *Use of these drugs is associated with osteonecrosis of the mandible Specifically, with IV administration

Local Anesthetic Mechanism of Action:

Block peripheral nerve conduction by decreasing permeability of ion channels to -BOARD ALERT: Local anesthetic agents 'inhibit the influx of sodium ions' -Affect the myelinated fibers only at the nodes of Ranvier

Propylthiouracil, methimazole mechanism

Block thyroid peroxidase, inhibiting the oxidation of iodide and the organification (coupling) of iodine -> inhibition of thyroid hormone synthesis. Propylthiouracil also blocks 5′-deiodinase -> decreased peripheral conversion of T4 to T3.

Chloroquine MoA

Blocks detoxification of heme into hemozoin. Heme accumulates and is toxic to plasmodia

Foscarnet clinical use

CMV retinitis in immunocompromised patients when ganciclovir fails; acyclovir-resistant HSV.

Ganciclovir clinical use

CMV, especially in immunocompromised patients. Valganciclovir, a prodrug of ganciclovir, has better oral bioavailability.

Yes, but only one at a time.

Can more than one drug fit into a receptor site?

Antidepressant: bupropion (Wellbutrin)

Can precipitate seizures o Also used for smoking cessation**

Drugs that cause SIADH

Carbamazepine, Cyclophosphamide, SSRIs

SSRIs toxicity

Fewer than TCAs. GI distress, SIADH, sexual dysfunction (anorgasmia, decreased libido).

Drugs that cause myopathy

Fibrates, niacin, colchicine, hydroxychloroquine, interferon-α, penicillamine, statins, glucocorticoids

Methotrexate (MTX) mechanism

Folic acid analog that competitively inhibits dihydrofolate reductase decreasing dTMP and decreasing DNA synthesis.

Fentanyl mechanism

Full agonist at μ receptor

List the typical (neuroleptics) antipsychotics

Haloperidol, trifluoperazine, fluphenazine, thioridazine, chlorpromazine (haloperidol + "-azines")

a. increase the risk of drug-drug interactions.

Highly protein-bound drugs a. increase the risk of drug-drug interactions. b. typically provide a short duration of action. c. must be administered with 8 ounces of water. d. have a decreased effect in patients with a low albumin level.

Levothyroxine (T4), triiodothyronine (T3) mechanism

Hormone replacement

B. They irreversibly bind to H+/K+ATPase.

How will the nurse describe the action of proton pump inhibitors? A. They compete with histamine for binding sites on the parietal cells. B. They irreversibly bind to H+/K+ATPase. C. They cause a decrease in stomach pH. D. They cannot be used to treat erosive esophagitis.

Cocaine withdrawal

Hypersomnolence, malaise, severe psychological craving, depression/suicidality.

What are the major functions of the Histamine-2 receptor?

Increase gastric acid secretion

What are the major functions of the βeta-1 receptor?

Increase heart rate, increase contractility, increase renin release, increase lipolysis

Probenecid

Inhibits reabsorption of uric acid in proximal convoluted tubule (also inhibits secretion of penicillin). Can precipitate uric acid calculi.

Ketoconazole

Inhibits steroid synthesis (inhibits 17,20-desmolase)

Ribavirin MoA

Inhibits synthesis of guanine nucleotides by competitively inhibiting inosine monophosphate dehydrogenase.

Duration of a drug

Length of time a drug produces its therapeutic effect

Foscarnet mechanism of resistance

Mutated DNA polymerase.

Ganciclovir mechanism of resistance

Mutated viral kinase.

Acyclovir, famciclovir, valacyclovir mechanism of resistance

Mutated viral thymidine kinase.

aspirin (ASA)

NONOPIOID ANALGESICS -Considered a nonsteroidal anti-inflammatory drug (NSAID) 4A'S Analgesic Antipyretic Anti-inflammatory Antiplatelet

Tetracyclines dentally Used to treat

NUP/NUG and aggressive perio -due to their excretion into the gingival crevicular fluid where they have an Anti-Collagenase effect

Salicylates toxicity antidote

NaHCO3 (alkalinize urine), dialysis

Opioids toxicity antidote

Naloxone, naltrexone

Xerostomia, drowsiness, constipation, respiratory depression

Name 4 common side effects of narcotic analgesics such as hydrocodone?

Vicodin, Lorcet, Lortab

Name three Brand names for hydrocodone with acetaminophen:

DIABETES DRUGS Type 2 - Non-Insulin Dependent (NIDDM)

Obesity is a major risk factor BOARD ALERT - hyperglycemia can result from the cells becoming insulin resistant

Acyclovir, famciclovir, valacyclovir toxicity

Obstructive crystalline nephropathy and acute renal failure if not adequately hydrated. Mutated viral thymidine kinase.

Potentiation

One drug HELPS another drug produce its desired effect without producing the effect itself.

Amoxicillin

Otitis media/ Sinusitis Dental Considerations: Oral candidiasis w/prolonged use

Adverse effect of aminoglycosides during pregnancy

Ototoxicity

D. "Sit or lie down after you take a nitroglycerin tablet to prevent dizziness."

Patient teaching regarding sublingual nitroglycerin should include which statement? A. "You can take up to five doses every 3 minutes for chest pain." B. "Chew the tablet for the quickest effect." C. "Keep the tablets locked in a safe place until you need them." D. "Sit or lie down after you take a nitroglycerin tablet to prevent dizziness."

D. Nausea and vomiting and abdominal cramping

Patient teaching regarding the administration of vitamin C would include which indication(s) of toxicity? A. Excessive bleeding tendencies B. Constipation C. Seizure activity D. Nausea and vomiting and abdominal cramping

CONGESTIVE HEART FAILURE DRUGS

Patients may be on the following drugs Diuretics ACE Inhibitors ARBS B-Blockers -Digitalis glycosides are also used to treat CHF -Not used much any longer due to narrow therapeutic index -digoxin (Lanoxin) not safe If a patient is on this, most likely they have CHF

A. Pramlintide slows gastric emptying.

Pramlintide (Symlin) is added to the treatment plan for a patient with type 1 diabetes. What information about the action of this medication does the nurse include in the patient teaching? A. Pramlintide slows gastric emptying. B. Pramlintide increases glucagon excretion. C. Pramlintide works to prevent side effects of insulin. D. Pramlintide is an oral drug administered 15 minutes before meals.

Stage 3 - advanced infection

Predominately anaerobic bacteria (mean clean) Metronidazole or clindamycin

Heparine toxicity antidote

Protamine sulfate

Sulfonylureas toxicity

Risk of hypoglycemia in renal failure. First generation: disulfiram- like effects. Second generation: hypoglycemia.

Bupropion

SNRI (aminoketone antidepressant) --> NRI and DRI - seizures, agitation, insomnia, dry mouth - can cause anxiety (stimulatory--increase dopamine), low risk for sexual dysfunction - seizure risk above 300 mg

Drugs that cause photosensitivity

Sulfonamides, Amiodarone, Tetracyclines, 5-FU

Opioids (e.g., morphine, heroin, methadone) withdrawal

Sweating, dilated pupils, piloerection ("cold turkey"), fever, rhinorrhea, yawning, nausea, stomach cramps, diarrhea ("flu-like" symptoms). Treatment: long-term support, methadone, buprenorphine.

d. Recommend taking acetaminophen (Tylenol) before the injection

The client with hepatitis is being treated with interferon alfa (Remeron), a biological response modifier. Which information should the clinic nurse discuss with the client? a. Tell the client that the skin will become yellow while taking this medication b. Discuss that the client may experience som abnormal bruising and bleeding c. Explain that if flulike symptoms occur, the client should stop taking the medication d. Recommend taking acetaminophen (Tylenol) before the injection

C. Pulmonary

The most severe adverse effects of amiodarone are evidenced in which body system? A. Renal B. Hepatic C. Pulmonary D. Hematologic

c. "Your baby's gut is sterile, and this will help the blood to clot."

The mother of a newborn African American infant asks the nursery nurse, "Why did you give my baby a vitamin K injection?" Which statement is the nurse's best response? a. "Vitamin K will help your infant's ability to fight off infection." b. "It will help protect your baby from getting sickle cell anemia." c. "Your baby's gut is sterile, and this will help the blood to clot." d. " This will help prevent your baby from becoming jaundiced."

B. AIDS

The nurse has an order for a patient to receive prednisone for contact dermatitis. What condition in the patient would alert the nurse to question the order? A. Asthma B. AIDS C. COPD D. Multiple sclerosis

a. Administer an isotonic dextrose solution for 1 to 2 hours after discontinuing the PN.

The nurse is caring for a patient who has an order to discontinue the administration of parenteral nutrition. What will the nurse do to prevent the occurrence of rebound hypoglycemia in the patient? a. Administer an isotonic dextrose solution for 1 to 2 hours after discontinuing the PN. b. Administer a hypertonic dextrose solution for 1 to 2 hours after discontinuing the PN. c. Administer 3 ampules of dextrose 50% immediately prior to discontinuing the PN. d. Administer 3 ampules of dextrose 50% 1 hour after discontinuing the PN.

a. A terminally ill patient who has opioid-induced constipation

The nurse is preparing to administer methylnaltrexone (Relistor), a peripherally acting opioid antagonist. This drug is appropriate for which patient? a. A terminally ill patient who has opioid-induced constipation b. A patient with diarrhea c. A patient who will be having colon surgery in the morning d. A patient who is scheduled for a colonoscopy

a. Acetylsalicylic acid (ASA), a salicylate, to a client receiving warfarin (Coumadin).

The nurse is preparing to administer the following medications. Which medication should the nurse question administering? a. Acetylsalicylic acid (ASA), a salicylate, to a client receiving warfarin (Coumadin). b. Ibuprofen (Motrin), an NSAID, to a client receiving furosemide (Lasix). c. Nabumetone (Relafen), a COX-2 inhibitor, to a client receiving digoxin (Lanoxin). d. Ketorolac (Toradol), and NSAID, intramuscularly to a client on a morphine PCA.

a. insulin

The nurse is reviewing a patient's medication list and notes that sitagliptin (Januvia) is ordered. The nurse will question an additional order for which drug or drug class? a. insulin b. glitazone c. sulfonylurea d. metformin (Glucophage)

a, d, e

The nurse is reviewing adverse effects of antithyroid medications with a patient who has been taking PTU. What adverse effects does the nurse include in the teaching? (Select all that apply.) A. Liver toxicity B. Polyuria C. Kidney damage D. Bone marrow toxicity E. Joint pain

A. Give it as a rapid intravenous push.

The nurse is reviewing emergency protocols and administration of adenosine (Adenocard). What is a vitally important task to remember when administering adenosine? A. Give it as a rapid intravenous push. B. Give it at the highest port in the IV tubing. C. Offer it with food or milk. D. Prepare to set up for an intravenous drip infusion.

c Every 72 hours

The nurse teaches a patient prescribed the fentanyl (Duragesic) transdermal delivery system to change the patch at what interval? a When pain recurs b Every 24 hours c Every 72 hours d Once a week

C. A 32-year-old pregnant woman with a urinary tract infection

The nurse will question an order for misoprostol (Cytotec) in which patient? A. A 21-year-old man with Zollinger-Ellison syndrome B. A 64-year-old man with hypertension C. A 32-year-old pregnant woman with a urinary tract infection D. A 45-year-old woman with GERD

A. neutralize gastric acid.

The nurse will teach patients that antacids are effective in treatment of hyperacidity because they A. neutralize gastric acid. B. decrease stomach motility. C. decrease gastric pH. D. decrease duodenal pH.

c With medications that are known to be irritating, painful, and/or staining to tissues

The nurse will use a Z-track technique when giving an intramuscular injection in which situation? a When there is insufficient muscle mass in the landmarked area b When massaging the area after medication administration is contraindicated c With medications that are known to be irritating, painful, and/or staining to tissues d With any injection that is given into the ventrogluteal muscle

c Drug properties

The nurse working in a prenatal clinic recognizes that the safety or potential harm of drug therapy during pregnancy relates to which factor? a Maternal blood type b Fetal sex c Drug properties d Diet of the mother

A. spironolactone (Aldactone)

The nurse would expect to administer which diuretic to treat a patient diagnosed with primary hyperaldosteronism? A. spironolactone (Aldactone) B. hydrochlorothiazide (HydroDIURIL) C. acetazolamide (Diamox) D. furosemide (Lasix)

D. "You cannot mix this insulin with regular insulin and thus will have to take two injections."

The nurse would include which statement when teaching a patient about insulin glargine? A. "You should inject this insulin just before meals because it is very fast acting." B. "The duration of action for this insulin is approximately 8 to10 hours, so you will need to take it twice a day." C. "You can mix this insulin with NPH insulin to enhance its effects." D. "You cannot mix this insulin with regular insulin and thus will have to take two injections."

D. Irritability

The nurse would suspect excessive thyroid replacement in a patient taking levothyroxine (Synthroid) when the patient is exhibiting which adverse effect? A. Depression B. Intolerance to cold C. Weight gain D. Irritability

Hydrochlorothiazide

Thiazide diuretic - low K+, slight xerostomia, oral ulcerations - NSAIDs potentially decrease diuretic effectiveness

Primary (essential) hypertension therapy

Thiazide diuretics, ACE inhibitors, angiotensin II receptor blockers (ARBs), dihydropyridine Ca2+ channel blockers.

β-blockers as glaucoma drugs (drugs, mechanism, side-effects)

Timolol, betaxolol, carteolol -Mechanism: decreased aqueous humor synthesis -Side effects: No pupillary or vision changes

A. Inhibiting absorption of dietary and biliary cholesterol in the small intestine

What is the mechanism of action of ezetimibe (Zetia)? A. Inhibiting absorption of dietary and biliary cholesterol in the small intestine B. Inhibiting the biosynthesis of cholesterol in the liver C. Binding to bile in the intestinal tract, inhibiting its absorption and thus causing the liver to produce bile from cholesterol D. Decreasing the adhesion of cholesterol on the arterial walls

d. Fat-soluble vitamins

When administering mineral oil, the nurse recognizes that it can interfere with the absorption of which substance? a. Electrolytes b. Minerals c. Water-soluble vitamins d. Fat-soluble vitamins

D. ondansetron (Zofran)

Which drug works by blocking serotonin in the gastrointestinal tract, vomiting center, and chemoreceptor trigger zone (CTZ)? A. metoclopramide (Reglan) B. meclizine (Antivert) C. droperidol (Inapsine) D. ondansetron (Zofran)

B Complete blood count

Which laboratory test should be monitored frequently to assess for a potential life-threatening adverse reaction to clozapine (Clozaril)? A Renal panel B Complete blood count C Liver function tests D Immunoglobulin levels

a Potential for patient harm is higher with these medications.

Why are specific medications classified as "high-alert" medications? a Potential for patient harm is higher with these medications. b Medications always cause certain adverse effects. c States require that these medications be on the high-alert list. d Only RNs are allowed to administer these medications.

Potency

amount of drug needed to produce a therapeutic effect (wanted effect) -log dose curve along x-axis

Toxic reaction-

an extension of the phramacologic effect resulting from a drugs effect on the target organs. (the amount of the desired effect is excessive)

Drug allergy:

an immunologic response to a drug resulting in a reaction such as a rash or anaphylaxis. SPECIAL NOTE THIS IS NEITHER PREDICTABLE NOR DOSE RELATED.

what is acetaminophen's pharmacological effects?

analgesic and antipyretic. it is not a anti inflammatory. can be poisonous to the liver.

what are the drug pharmacologic effects for NSAIDS?

analgesic, antipyretic and antiinflammatory.

Antianxiety Agents Contraindicated in pregnancy

benzodiazepines nitrous oxide-oxygen

Nadolol

beta-blocker (++, non-selective beta-1 and beta-2 - Limit epi to 0.04mg with ++ agents - NSAIDs may reduce effectiveness, may cause xerostomia

When the two formulations produce similar concentrations of the drug in blood and tissues, they are termed

biologically equivalent.

ANTIANGINAL DRUGS

chest pain -Nitroglycerin -administered sublingual Adverse Reactions -Hypotension -Dangerously low BP if given when patient has taken a PDE5 inhibitor (erectile dysfunction drug) in the last 24 hours (Viagra)

Nitrous Oxide Sedation BOARD ALERT- place patient on 100% oxygen for 5 minutes following sedation to prevent

diffusion hypoxia (rapid outward flow of CO2- decreasing ventilation causing headache)

cholinergic agents are classified by two ways?

direct acting (acts on receptor) or indirect acting (causes release of neurotransmitter)

q-

every

qd

every day

how does Cholinergic effect the GI?

excites the smooth muscle. increases activity.

Tetracycline, non-steroidal anti inflammatory agents, benzodiazepines, and metronidazole are safe for pregnancy's?

false

Taking NSAID with phenytoin (Dilantin) can

increase phenytoin levels

Patients with liver disease or alcoholism may have decreased ability to metabolize drugs; leads to

increased levels of drug in the blood, which increases the risk for toxicity.

Drugs with solubility in lipids =

increased potency = 1 / MAC (minimal alveolar concentation) of inhaled anesthetic

Synergism

interaction of two or more drugs such that the total effect is greater than the sum of the individual effects

Efficacy

maximum intensity of effect produced by a drug Shown by height (y) of log dose curve

penicillin VK is preferred over

penicillin G (pen G is inactivated by Gastric acids)

Patients who should not recieve cholinergic drugs?

people who are suffering from : peptic ulcer, myasthenia gravis, severe cardiac disease, gi obstruction, hyperthyroidism.

The effect of a drug exerts on biologic systems can be related to-

quantitatively to the dose of the drug given.

methamphetamine: sympathomimetic/ adrenergic

rampant caries and burned mucosal surfaces **Oral mucosal irritation is a result of the method of drug administration...not the drug itself.

acetaminophen (APAP) Side effects

rare -Patients with liver disease should avoid -Overdose can result in Hepatotoxicity (liver toxicity)

Only true contraindication to vasoconstrictors is

sulfite allergy - due to the preservative that is present

What type of administration is considered safest, least expensive, and most convenient?

suppository....kidding, oral administration.

Anti-infective Agents Contraindicated in pregnancy

tetracyclines metronidazole

the body of a prescription requires?

the RX symbol. name and dose size or concentration of the drug. amount to be dispensed. directions to the patient.

The Ph of tissues at the site of administration and dissociation characteristics of the drug will determine?

the amount of drug in the ionized and nonionized state.

Amount of drug available to produce systemic effect is reduced by

the first-pass effect.

when the Ph at the site increases?

the hydrogen ion concentration falls. This means the ionized form cannot easily penetrate tissues.

when the PH at the site decreases?

the hydrogen ion concentration will rise. this results in an increase in the un-ionized form, what can more easily penetrate tissues.

duration;

the length of a drugs effect.

aspirin is a zero-order kinetics, which means?

the rate of metabolism remains constant over time. the same amount of drug metabolized per unit of time regardless of does.

What is Stevens-Johnson syndrome and which epilepsy drugs can cause it?

-SJ syndrome is a prodrome of malaise and fever followed by a rapid onset of erythematous/purpuric macules (oral, ocular, genital). Skin lesions progress to epidermal necrosis and sloughing -Drugs that can cause this horrific motha ****a: lamotragine, carbamazepine, phenytoin, ethosuximide (Stephen Johnson's LAMe CAR SUX PHENis)

Doxazosin

-Selective α1-blocker -Uses: urinary symptoms of BPH; -Hypertension -Toxicity: 1st-dose orthostatic hypotension, dizziness, headache

Tamsulosin

-Selective α1-blocker -Uses: urinary symptoms of BPH; -Toxicity: 1st-dose orthostatic hypotension, dizziness, headache

b. Hepatic necrosis

A 38-year-old man has come into the urgent care center with severe hip pain after falling from a ladder at work. He says he has taken several pain pills over the past few hours but cannot remember how many he has taken. He hands the nurse an empty bottle of acetaminophen (Tylenol). The nurse is aware that the most serious toxic effect of acute acetaminophen overdose is which condition? a. Tachycardia b. Hepatic necrosis c. Central nervous system depression d. Nephropathy

d. Stimulation of salivation

A patient who has been diagnosed with Sjögren's syndrome will be given cevimeline for the treatment of xerostomia. The nurse will monitor for what therapeutic effect? a. Reduction of salivation b. Improvement of fine-motor control c. Reduction of gastrointestinal peristalsis d. Stimulation of salivation

Amphetamine mixtures

ADD/ADHD - GI, anorexia, dizziness, mood swings, no abuse - may cause seizures, nervousness, insomnia, dizziness, HA, dyskinesia, tachycardia, anorexia - dry mouth, dyskinetic movements, increased BP, pulse

Losartan

ARB (-sartan)

Valsartan

ARB (-sartan)

Terazosin

Alpha blocker - dizziness - HA - NSAIDs reduce effectiveness, caution with position change

Mirtazapine

Alpha-2 receptor antagonist - drowsiness, dizziness, weight gain - NO BRUXISM - minimal dry mouth, minimal SSRI type side effects

Carbamazepine

Anticonvulsant - drowsiness, ataxia, severe blood dycrasias - increase APAP liver toxicity, decrease APAP effect - low stress environment (consider BZDP), seizure control history often

B. Intramuscular injection using the Z-track method

The nurse will plan to inject iron dextran by which technique? A. Subcutaneous injection with a ½-inch, 25-gauge needle B. Intramuscular injection using the Z-track method C. Intradermal injection with a sunburst technique of administration D. Intramuscular injection with a ½-inch, 18-gauge needle

a. Obtain an order for a spacer device.

When evaluating a patient's use of a metered-dose inhaler (MDI), the nurse notes that the patient is unable to coordinate the activation of the inhaler with her breathing. What intervention is most appropriate at this time? a. Obtain an order for a spacer device. b. Notify the doctor that the patient is unable to use the MDI. c. Obtain an order for a peak flow meter. d. Ask the prescriber if the medication can be given orally.

B. Increased protein synthesis

Which is a pharmacodynamic effect of exogenous androgens? A. Decreased erythropoiesis B. Increased protein synthesis C. Increased nitrogen excretion D. Decreased sperm production

C. St. John's wort

Which over-the-counter/herbal product, when taken with theophylline, can decrease theophylline's serum drug levels? A. Caffeine B. Diphenhydramine C. St. John's wort D. Echinacea

b acetylcysteine (Mucomyst)

While admitting a patient for treatment of an acetaminophen overdose, the nurse prepares to administer which medication to prevent toxicity? a naloxone (Narcan) b acetylcysteine (Mucomyst) c methylprednisolone (Solu-Medrol) d vitamin K

Zolpidem

Anti-anxiety - headache, sedation, myalgia, nausea - xerostomia, CNS depressants additive

Goal of antianginal therapy

Goal is reduction of myocardial O2 consumption (MVO2) by decreasing 1 or more of the determinants of MVO2: end-diastolic volume, BP, HR, contractility.

Clopidogrel (Plavix)

Anti-platelet effect - Dizziness, GI upset - Levels increased by NSAIDs

Phenytoin

Anticonvulsant - Gingival overgrowth, drowsiness, ataxia - low stress environment (consider BZDP), seizure control history often

Lamotrigine

Anticonvulsant - dizziness, ataxia, HA, diplopia, rash - low stress environment (consider BZDP), seizure control history often

Donepezil

-Anticholinesterse - increases ACh -Alzheimer disease

Amphetamines withdrawal

Anhedonia, appetite, hypersomnolence, existential crisis.

thiazide diuretic

What pharmacologic category does hydrochlorothazide belong to?

Amiloride

K+ sparing diuretic

Selegiline transdermal

MAOI - for Parkinson's Disease - orthostatic hypotension, tachycardia, HA, restlessness, insomnia, dizziness, overstimulation including anxiety, agitation, and manic symptoms, dry mouth, paresthesias, diarrhea - limit epi dose to .04mg and aspirate - record baseline and post tx BP - hypertensive crisis with tyramine rich foods

Side effect of thioridazine

ReTinal deposits

a, b, c

Side effects to expect from anticholinergic (parasympatholytic) drugs, such as atropine, include (Select all that apply.) A. dilated pupils. B. urinary retention. C. dry mouth. D. diarrhea E. increased sweating.

History of recurrent UTIs

TMP-SMX

Pharmacology

The study of drugs and there effects on organisms.

c Therapeutic index

What is the ratio between a drug's therapeutic effects and toxic effects called? a Tolerance b Cumulative effect c Therapeutic index d Affinity

Treatment strategy for gestational DM

Dietary modification, exercise, insulin replacement if lifestyle changes fail

Drug of choice for status epilepticus which CAN be caused by local anesthetic overdose

diazepam (Valium)

Duration of action of anesthetics (shortest)

mepivacaine

Duration of action of anesthetics (Shortest to longest)

mepivacaine (shortest) lidocaine prilocaine articaine bupivacaine (longest)

ANTIDEPRESSANT DRUGS Used to treat symptoms of

moderate to severe depression **not a cure

Stage 1 - Initial stages of infection

o Gram-positive cocci o Drug of choice Penicillin / amoxicillin •Penicillin allergy→ erythromycin or clindamycin

Cell bodies in CNS give rise to preganglionic fibers of the ___

parasympathetic division

naloxone (Narcan)

parenteral opioid antagonist -Drug of choice for treating opiod overdose -Will reverse respiratory depression from OD -Should be in emergency kit

Tetracyclines are contraindicated in

pregnant / nursing mothers and early childhood due to tooth discoloration -**cross placental barrier and are excreted in breast milk

The closing should include?

prescribers's signatures. DEA number. and refill instructions.

how are anticholinergic drugs used in dentistry?

reduce the flow and volume of exocrine secretions to decrease salivation and create a dry field.

Sans. - cell bodies that give origin to the preganglionic fibers of the sans span from....

the thoracic T1 to the lumber L2 portion of the spinal cord

onset

the time required for the drug to begin to have its effect.

Major route of fluoride excretion is in

the urine.

In the case of tablets and capsules, the word "dispense" is ofter replaced with #. however when writing prescriptions for opioids or other controlled substances what also should be written in parentheses next to the number?

the written out form of the number to prevent people from adding 1's or 0's.

Drugs elicit pharmacological effects after they have been distributed to

their sites of action.

Giving more of a drug will not increase efficacy - increases risk for?

toxicity

Narrow therapeutic index (small TI

toxicity more likely

ANTINEOPLASTIC DRUGS (CANCER DRUGS)

treat initial stages aggressively -Used at much lower doses, can treat Psoriasis, rheumatoid arthritis, lupus

what does old aspirin smell like?

vinegar due to the acetic acid breakdown.

Antibiotics are affective against BACTERIAL INFECTIONS...not

viral infections.

Therapeutic effect

wanted effect desirable action of the drug *Predictable *Dose related *Target organs

Common medications that cause gingival bleeding:

warfarin (Coumadin) clopidogrel (Plavix) aspirin

administering more drug will _______ the efficacy.

will not increase the efficacy. but can often increase the probability of an adverse reaction.

Levothyroxine (T4), triiodothyronine (T3) clinical use

Hypothyroidism, myxedema. Off label use as weight loss supplements

Interferons clinical use

IFN-α: chronic hepatitis B and C, Kaposi sarcoma, hairy cell leukemia, condyloma acuminatum, renal cell carcinoma, malignant melanoma. IFN-β: multiple sclerosis. IFN-γ: chronic granulomatous disease.

Prescription required for schedule

II, III, and IV drugs

Bevacizumab clinical use

Colorectal cancer, renal cell carcinoma

Pramipexole

Anti-parkinson's: Dopaminergic Agent - hallucinations, nausea, dizziness, sedation, sudden sleep attacks

Carbidopa/Levodopa

Anti-parkinson's: Dopaminergic Agent - movement disorders, GI, alliterated taste, excess salivation, bruxism - can increase effects of EPI

Ropinirole

Anti-parkinson's: Dopaminergic Agent - syncope, nausea, dizziness, sedation

Gabapentin

Anticonvulsant - dizziness, ataxia, fatigue, nystagmus - low stress environment (consider BZDP), seizure control history often

Oxcarbazepine

Anticonvulsant - drowsiness and ataxia - low stress environment (consider BZDP), seizure control history often

Montelukast, zafirlukast

-Antileukotrienes -Block leukotriene receptors (CysLT1). -Especially good for aspirin-induced asthma.

Effects of β-blockers

-Angina pectoris—decrease heart rate and contractility, resulting in decrease O2 consumption -MI—β-blockers (metoprolol, carvedilol, and bisoprolol) mortality -SVT (metoprolol, esmolol)—decrease AV conduction velocity (class II antiarrhythmic) -Hypertension—decrease cardiac output, decrease renin secretion (due to β1-receptor blockade on JGA cells) -HF—decrease mortality in chronic HF -Glaucoma (timolol)—decrease secretion of aqueous humor

Adenosine

-Antiarrhythmic -Increase K+ out of cells hyperpolarizing the cell and increasing intracellular Ca2+. -Drug of choice in diagnosing/abolishing supraventricular tachycardia. Very short acting (~ 15 sec). Effects blunted by theophylline and caffeine (both are adenosine receptor antagonists).

Edrophonium

-Anticholinesterse - increases ACh -Historically used to diagnose myasthenia gravis (MG is now diagnosed by anti-AChR Ab test.

Pyridostigmine

-Anticholinesterse - increases ACh -Increases muscle strength -Used in myasthenia gravis (long acting) -Does not penetrate CNS

B Schedule II

A nurse working with patients who are diagnosed with ADHD is aware such patients often take CNS stimulant drugs. These medications are potent with a high potential for abuse and dependence. Based on this potential, how are these medications classified? A Schedule I B Schedule II C Schedule III D Schedule IV

c. Withhold the dose, and notify the prescriber.

A patient about to receive a morning dose of digoxin has an apical pulse of 53 beats/minute. What will the nurse do next? a. Administer the dose. b. Check the radial pulse for 1 full minute. c. Withhold the dose, and notify the prescriber. d. Administer the dose, and notify the prescriber.

b Cough suppressant

A patient admitted to the hospital with a diagnosis of pneumonia asks the nurse why she is receiving codeine when she does not have any pain. The nurse's response is based on knowledge that codeine also has what effect? a Stimulation of the immune system b Cough suppressant c Expectorant d Bronchodilation

d. Uncontrolled angle-closure glaucoma

A patient has a prescription for oxybutynin (Ditropan), an anticholinergic drug. When reviewing the patient's medical history, which condition, if present, would be considered a contraindication to therapy with this drug? a. Diarrhea b. Hypertension c. Neurogenic bladder d. Uncontrolled angle-closure glaucoma

Parenteral Route

Bypasses the GI tract —Injection •Intravenous most rapid •Intramuscular •Subcutaneous insulin •Intradermal TB test —Inhalation Inhalers for asthma —Topical Systemic side effects are rare •Steroids, Contraindicated if surface tissue is ulcerated,

3, 2, 1

The physician orders ipratropium bromide (Atrovent), albuterol (Proventil), and beclomethasone (Beclovent) inhalers for your patient. Put the drugs in the correct order in which they will be administered. 1. beclomethasone (Beclovent) 2. ipratropium bromide (Atrovent) 3. albuterol (Proventil)

Mg hydroxide

Effective in torsades de pointes and digoxin toxicity

Loop diuretics (drugs, mechanism, use, toxicity)

Furosemide, bumetanide, torsemide -Mechanism: sulfonamide loop diuretics. Inhibit cotransport system (Na+/K+/2Cl−) of thick ascending limb of loop of Henle. Abolish hypertonicity of medulla, preventing concentration of urine. Stimulate PGE release (vasodilatory effect on afferent arteriole); inhibited by NSAIDs. Increase Ca2+ excretion. Loops Lose Ca2+. -Use: edematous states (HF, cirrhosis, nephrotic syndrome, pulmonary edema), hypertension, hypercalcemia. -Toxicity: ototoxicity, hypokalemia, dehydration, allergy (sulfa), nephritis (interstitial), gout.

Brand Name: Zocor

Generic Name: Simvastatin

Brand Name: Ambien

Generic Name: Zolpidem

Oxycodone, hydrocodone, morphine, methadone

Give four examples of drugs found in Schedule II:

tramadol (Ultram), diazepam (Valium)

Give two examples of a Schedule IV drug:

Heroin, LSD (Ecstacy)

Give two examples of drugs found in Schedule I:

Digoxin clinical use

HF (increased contractility), atrial fibrillation (decrease conduction at AV node and depression of the SA node)

Lethal Dose 50 divided by Effective Dose 50

How is Therapeutic Index calculated?

Antagonist

If a drug binds to a receptor site and decreases the function of the cell, it is called an ______.

Agonist

If a drug binds to a receptor site and increases the function of the cell, it is called an _______.

a. Potassium level 2.8 mEq/L

In assessing a patient before administration of a cardiac glycoside, the nurse knows that which lab result can increase the toxicity of the drug? a. Potassium level 2.8 mEq/L b. Potassium level 4.9 mEq/L c. Sodium level 140 mEq/L d. Calcium level 10 mg/dL

B. Deficient fluid volume related to nausea and vomiting

In developing a plan of care for a patient receiving an antihistamine antiemetic drug, which nursing diagnosis would be of highest priority? A. Deficient knowledge regarding medication administration B. Deficient fluid volume related to nausea and vomiting C. Risk for injury related to side effects of medication D. Anxiety related to nausea and vomiting

Azoles clinical use

Local and less serious systemic mycoses. Fluconazole for chronic suppression of cryptococcal meningitis in AIDS patients and candidal infections of all types. Itraconazole for Blastomyces, Coccidioides, Histoplasma. Clotrimazole and miconazole for topical fungal infections.

Angiotensin II receptor blockers (ARBs) (drugs, mechanism, use, toxicity)

Losartan, candesartan, valsartan -Mechanism: selectively block binding of angiotensin II to AT1 receptor. Effects similar to ACE inhibitors, but ARBs do not increase bradykinin. -Use: hypertension, HF, proteinuria, or diabetic nephropathy with intolerance to ACE inhibitors (e.g., cough, angioedema). -Toxicity: hyperkalemia, decreased renal function, hypotension; teratogen.

Anorexiants

MoA-stimulant effect on hypothalamic and limbic region of the brain. USE-suppress appetite, wt loss, obesity AR-headache, dry mouth, hypokalemia, erectile dysfunction, dysgeusia, hypoglycemia, tremors, suicidal ideation, euphoria, dysphoria CI-under age 12, heart failure, glaucoma, within 14days of MAOI therapy -------caution in diabetes, psychosis, renal impairment EX-Naltrexone

ANTICONVULSANT DRUGS

Most have a narrow therapeutic index -increase the metabolism of other drugs -CNS depression Examples: -phenytoin (Dilantin)] - Gingival enlargement in 50% of chronic users - anterior facial gingiva -carbamazepine (Tegretol) -phenobarbital (Luminal) -thosuximide (Zarontin) -benzodiazepines diazepam (Valium)

Barbiturates (drugs, mechanism, use, toxicity)

Phenobarbital, pentobarbital, thiopental, secobarbital -Mechanism: facilitate GABA-A action by increasing duration of Cl− channel opening, thus decreasing neuron firing (barbidurates increase duration). Contraindicated in porphyria. -Use:sedative for anxiety, seizures, insomnia, induction of anesthesia (thiopental). -Toxicity: respiratory and cardiovascular depression (can be fatal); CNS depression (can be exacerbated by EtOH use); dependence; drug interactions (induces cytochrome P-450). Overdose treatment is supportive (assist respiration and maintain BP).

B. Conversion of recent-onset atrial fibrillation and flutter

The patient has been prescribed ibutilide (Corvert), a class III antiarrhythmic drug. The nurse is aware that this drug has been prescribed for which reason? A. Treatment of PSVT B. Conversion of recent-onset atrial fibrillation and flutter C. Conversion of life-threatening ventricular arrhythmias D. Treatment of dysrhythmias in patients with acute renal failure

Greater

The _____ the Therapeutic Index, the safer the drug.

a, b

The client is diagnosed with low back pain and is prescribed the muscle relaxant cyclobenzaprine (Flexeril). Which instructions should the clinic nurse teach the client? Select all that apply. a. The medication can cause drowsiness that will make driving unsafe. b. Suck on hard candy if the client experiences dry mouth. c. Divide the dose of medication between early morning and bedtime. d. Drink a full glass of water with each dose of medication. e. Take the medication just before leaving home for work each day.

d. Tell the client to not take any more of the Celebrex.

The client with osteoarthritis who is taking the COX-2 inhibitor celecoxib (Celebrex), a non-steroidal anti-inflammatory drug (NSAID), calls the clinic and reports having black, tarry stools. Which intervention should the clinic nurse implement? a. Ask if the client is taking any kind of iron prepartation. b. Instruct the client to bring a stool specimen to the clinic. c. Explain that this is a side effect of the medication. d. Tell the client to not take any more of the Celebrex.

A. decreases mobility of leukocytes.

The nurse assesses for therapeutic effects of colchicine based on knowledge that colchicine A. decreases mobility of leukocytes. B. increases process of phagocytosis. C. increases uric acid metabolism. D. increases production of lactic acid.

B phentolamine (Regitine)

The nurse assesses the intravenous infusion site of a patient receiving dopamine and finds it is infiltrated. The nurse will prepare which medication to treat this infiltration? A lidocaine (Xylocaine) B phentolamine (Regitine) C nitroprusside (Nipride) D naloxone (Narcan)

b Drug half-life is lengthened.

The physiologic changes that normally occur in the older adult have which implication for drug response in this patient? a Drug metabolism is faster. b Drug half-life is lengthened. c Drug elimination is faster. d Protein binding is more efficient.

in 1984, congress passed the Drug Price Competition and Patent Term restoration act what did what?

allows generic drugs to receive expedited approval. The FDA still requires that the active ingredient of the generic product enter the bloodstream at the same rate as the trade name product.

Choroquine toxicty

Retinopathy; pruritus (especially in dark-skinned individuals).

H2 blocker mechanism

Reversible block of histamine H2 receptors decreasing H+ secretion by parietal cells

Depolarizing neuromuscular blocking drugs

Succinylcholine—strong ACh receptor agonist; produces sustained depolarization and prevents muscle contraction

bupivacaine (Marcaine)

Amide -Greatly prolonged duration of action -Indicated for postoperative pain and lengthy dental procedures*

articaine (Septocaine)

Amide anesthetic with ester linkage in chemical structure (hybrid molecule) -Partially metabolized in the liver (5-10%) -partially in the blood (90-95%) GOOD 4 peeps w/ Liver problems. Drug of choice

Ramelteon

Anti-anxiety - xerostomia, CNS depressants additive -dizziness, headache, sonmolence (strong desire to sleep)

Ezetimibe (Zetia)

Antihyperlipidemic - cholesterol absorption inhibitors - GI, HA, flatulence - taste distrubance, gag reflex increased

Haloperidol

Antipsychotic--Butyrophenone - movement disorders, orthostatic hypotension, tradeoff dyskinesia - little or no interaction with EPI, CNS depressants potentiate drug, xerstomia can be severe

Clozapine

Antipsychotic--second generation atypical - 3+ for xerostomia, drowsiness, dizziness, salivation, dry mouth, aplastic anemia - most commonly used

Quetiapine

Antipsychotic--second generation atypical - HA, drowsiness, dizziness

ACE inhibitors (drugs, mechanism, use, toxicity)

Captopril, enalapril, lisinopril, ramipril -Mechanism: inhibit ACE -> decrease AT II -> decrease GFR by preventing constriction of efferent arterioles. Levels of renin increase as a result of loss of feedback inhibition. Inhibition of ACE also prevents inactivation of bradykinin, a potent vasodilator. -Use: hypertension, HF, proteinuria, diabetic nephropathy. Prevent unfavorable heart remodeling as a result of chronic hypertension. In diabetic nephropathy, intraglomerular pressure, slowing GBM thickening -Toxicity: cough, Angioedema (contraindicated in C1 esterase inhibitor deficiency), Teratogen (fetal renal malformations), increased Creatinine (decreased GFR), Hyperkalemia, and Hypotension. Avoid in bilateral renal artery stenosis, because ACE inhibitors will further decrease GFR causing renal failure.

most common opioid used in dentistry

Codeine (Tylenol #3)

adrenergic Vasopressors

Contract smooth muscle vasoconstriction -nasal decongestants by constricting swollen vessels of the nasal mucous membranes -epinephrine (used in local anesthetic to keep medication at area of administration) -dopamine :shock and hypotension -ephedrine :cold medications -pseudoephedrine :cold medications

Adrenergic

Sympathetic

Drugs that cause hot flashes

Tamoxifen, clomiphene

C. Phlebitis

The most common and possibly devastating effect of peripheral total parenteral nutrition (PPN) is which condition? A. Poor nutrition B. Heart failure C. Phlebitis D. Electrolyte imbalance

a upon the patient's admission to the health care setting.

The nurse is aware that the most appropriate time to begin patient education and begin the teaching-learning process is a upon the patient's admission to the health care setting. b once the medical diagnosis is established. c when there are written orders for teaching. d at the time discharge planning is begun.

D This is a normal occurrence related to entacapone (Comtan).

The nurse is caring for a patient with Parkinson's disease. The patient has been taking entacapone (Comtan) for the past week to treat an on-off phenomenon. The patient expresses concern over brown-orange urine. What information will the nurse provide? A The patient may be developing renal failure. B The patient may be developing hepatic failure. C Brown-orange urine signifies a lack of fluid intake. D This is a normal occurrence related to entacapone (Comtan).

A. Fresh frozen plasma

The nurse is caring for a patient with an acute renal insufficiency and thrombocytopenia. Along with platelet transfusions, the nurse would expect to administer which substance to increase deficient clotting factors in this patient? A. Fresh frozen plasma B. Albumin C. Plasma protein factors D. Whole blood

C. It should be administered with food to diminish the risk of gastric irritation.

The nurse is discussing with a patient the time of day for taking prednisone. What information would the nurse include in the teaching based on knowledge of glucocorticoids? A. It is usually administered early in the evening to coincide with the natural secretion pattern of the adrenal cortex. B. It is usually administered on a strict, unchanging schedule in order to prevent adverse reactions. C. It should be administered with food to diminish the risk of gastric irritation. D. It should be administered with the patient's morning coffee to enhance its effects.

c Psychomotor domain

The nurse is educating a patient diagnosed with type 1 diabetes. When the nurse has the patient demonstrate how to perform self-injections of insulin, which domain of learning is the nurse assessing? a Cognitive domain b Affective domain c Psychomotor domain d Physical domain

B. Dried beans

The nurse is educating a patient on diet. Which food will the nurse encourage the patient to include in a diet as a source of folate? A. Cheese B. Dried beans C. Steak D. Apples

B. Use of sunglasses is encouraged to decrease sunlight sensitivity.

The nurse is educating a patient who was administered atropine sulfate solution for an eye exam. The nurse includes which information in the education? A. The patient will have to wait 72 hours to wear contacts. B. Use of sunglasses is encouraged to decrease sunlight sensitivity. C. The patient may have small pupils for 12 hours after administration. D. A common side effect is dry eyes.

a. at a therapeutic level.

The nurse is monitoring drug levels for a patient who is receiving theophylline. The most recent theophylline level was 13 mcg/mL, and the nurse evaluates this level to be a. at a therapeutic level. b. below the therapeutic level. c. above the therapeutic level. d. at a toxic level.

C COMT inhibitors

The nurse is providing care for a patient who has been diagnosed with Parkinson's disease. The patient is also in early stage liver failure. What class of medications, if prescribed, would the nurse question? A Dopamine modulator B Anticholinergics C COMT inhibitors D Ergot derivative

C. Renal failure

The nurse is providing care to a patient newly admitted for trauma from a motor vehicle accident. The provider has ordered IV fluids with potassium. What information noted in the patient's history would alert the nurse to question the order for this IV fluid? A. Hypertension B. Cirrhosis of the liver C. Renal failure D. Multiple sclerosis

A. Coronary artery disease

The nurse is providing care to a patient prescribed a nonselective adrenergic agonist bronchodilator. Which medical diagnosis on this patient's chart would alert the nurse to question this order? A. Coronary artery disease B. Chronic obstructive pulmonary disease (COPD) C. Hyperkalemia D. Premature labor

B. This combination promotes diuresis but decreases the risk of hypokalemia.

The nurse is providing education to a patient on why spironolactone (Aldactone) and furosemide (Lasix) are prescribed together. What information does the nurse provide to the patient? A. Moderate doses of two different types of diuretics are more effective than a large dose of one type. B. This combination promotes diuresis but decreases the risk of hypokalemia. C. This combination prevents dehydration and hypovolemia. D. Using two drugs increases osmolality of plasma and the glomerular filtration rate.

A. Patient is allergic to sulfa drugs.

The nurse is providing education to a patient who was diagnosed with bilateral conjunctivitis (pink eye). The patient is prescribed sulfacetamide (Bleph-10) for the diagnosis. What information in the patient history has the highest priority and should be reported immediately to the health care provider? A. Patient is allergic to sulfa drugs. B. Patient does not like eyedrops. C. Patient reports never having pink eye before. D. Patient wears contacts.

C. Intranasal steroids

The nurse is providing education to a patient with a history of chronic nasal congestion secondary to allergic rhinitis. Which class of medications would the nurse anticipate the provider would recommend for the patient to use on a long-term basis? A. Anititussives B. Antihistamines C. Intranasal steroids D. Expectorants

B. Pulmonary edema

The nurse is receiving shift report on her patients. One patient, per report, had a new IV bag of normal saline (NS) hung, which is to be infused at a rate of 100 mL per hour. The nurse discovers that the bag is almost empty and the pump was mistakenly set at 1000 mL per hour. What is the most immediate concern for any patient in this situation? A. Edema of hands and feet B. Pulmonary edema C. Increased urination D. Phlebitis

d. Parasympathetic nervous system

The nurse is reviewing the use of anticholinergic drugs. Anticholinergic drugs block the effects of which nervous system? a. Sympathetic nervous system b. Central nervous system c. Somatic nervous system d. Parasympathetic nervous system

B. Constipation

The nurse knows a common expected adverse effect of iron supplementation is which effect? A. Fatigue B. Constipation C. Heartburn D. Flatus

b, c, e,

The nurse knows that the medication reconciliation process involves which three steps? (Select all that apply.) a. Reporting b. Reconciliation c. Verification d. Administration e. Clarification

A Dystonia

The nurse monitors a patient taking an antipsychotic medication for extrapyramidal side effects. What is the nurse assessing for in the patient? A Dystonia B Orthostatic hypotension C Dry mouth and constipation D Neuroleptic malignant syndrome

a, b, d, e

The nurse needs to be aware that which factors will affect the absorption of orally administered medications? (Select all that apply.) a. Presence of food in the stomach b. pH of the stomach c. Patient position upon intake of medication d. Form of drug preparation e. Time of day

C Bipolar disorder

The nurse notes lithium on a patient's drug history upon admission. Which condition would the nurse suspect that this patient has been diagnosed with? A Obsessive-compulsive disorder B Absence seizures C Bipolar disorder D Paranoid schizophrenia

a. 0800

The nurse notes that a medication was scheduled to be administered at 0900. A medication error has occurred if the medication was administered at which time? a. 0800 b. 0830 c. 0900 d. 0930

b Ginseng

The nurse obtains a medication history from a patient diagnosed with primary hypertension. A nursing diagnosis of "Deficient knowledge related to medication therapy" would be identified when the patient states that he or she takes which over-the-counter medication and/or supplement? a Garlic b Ginseng c Valerian root d St. John's wort

b. A woman with a history of thrombophlebitis

The nurse recognizes that use of estrogen drugs is contraindicated in which patient? a. A patient who has atrophic vaginitis b. A woman with a history of thrombophlebitis c. A patient who has inoperable prostate cancer d. A woman who has just given birth and wants to prevent postpartum lactation

A. mupirocin (Bactroban)

The nurse will anticipate application of which medication to treat a patient diagnosed with impetigo? A. mupirocin (Bactroban) B. benzoyl peroxide C. isotretinoin (Amnesteem) D. retinoic acid (Renova)

C. Increased risk of bleeding

The nurse will assess a patient receiving gemfibrozil (Lopid) and warfarin (Coumadin) for which adverse effect? A. Increased risk of clotting B. Increased risk of vitamin K toxicity C. Increased risk of bleeding D. Deep vein thrombosis

C. nimodipine (Nimotop)

The nurse would plan to administer which calcium channel blocker to a patient with cerebral artery spasms following a subarachnoid hemorrhage? A. amlodipine (Norvasc) B. diltiazem (Cardizem) C. nimodipine (Nimotop) D. verapamil (Calan)

A. Uncontrolled diabetes mellitus

The nurse would question an order for steroids in a patient with which condition? A. Uncontrolled diabetes mellitus B. Rheumatoid arthritis C. Septic shock D. Exacerbation of chronic obstructive pulmonary disease (COPD)

A. Deep vein thrombosis

The nurse would question the order for estrogen replacement therapy in a patient with a history of which condition? A. Deep vein thrombosis B. Vaginal bleeding C. Weight loss D. Dysmenorrhe

B. Aortic regurgitation

The nurse would question the use of milrinone (Primacor) in a patient with which disorder? A. Systolic heart failure B. Aortic regurgitation C. Acute renal failure D. Mitral valve prolapse

C. Carbonated beverages

The nurse would teach a patient prescribed simethicone (Gas-X) to avoid which substances? A. Antacids B. Dairy products C. Carbonated beverages D. Histamine2-receptor antagonists

C "Fosphenytoin is converted to phenytoin once it is in your bloodstream. Since you are NPO, fosphenytoin is easier on your veins than phenytoin."

The patient asks the nurse why she is receiving a different drug than her usual phenytoin (Dilantin). The patient is NPO (nothing by mouth) secondary to illness and is receiving intravenous fosphenytoin (Cerebyx). Which is the nurse's most accurate response? A "Your serum phenytoin levels were not therapeutic, so your health care provider has changed your medication to a more effective drug." B "Phenytoin is not effective while you are NPO, so your health care provider has changed your medication to a more effective drug." C "Fosphenytoin is converted to phenytoin once it is in your bloodstream. Since you are NPO, fosphenytoin is easier on your veins than phenytoin." D "Since you are NPO, you cannot take phenytoin orally. Phenytoin does not come in an intravenous form. You will be transferred back to phenytoin after you recover from this illness."

B. "Dalteparin is a low-molecular-weight heparin that has a more predictable anticoagulant effect."

The patient asks what the difference is between dalteparin (Fragmin) and heparin. What is the nurse's best response? A. "There is really no difference, but dalteparin is preferred because it is less expensive." B. "Dalteparin is a low-molecular-weight heparin that has a more predictable anticoagulant effect." C. "I'm not really sure why some physicians choose dalteparin and some heparin, so you should ask your doctor." D. "The only difference is that heparin dosing is based on the patient's weight."

A. Below the therapeutic level

The patient has a serum digoxin level drawn and it comes back 0.4 ng/mL. How does the nurse interpret this lab value result? A. Below the therapeutic level B. A therapeutic level C. Above the therapeutic level D. A toxic level

C. Competing at histamine receptor sites

The patient has been prescribed an ocular antihistamine, olopatadine (Patanol), for treatment of allergic conjunctivitis. The nurse knows the medication works though which action? A. Vasoconstriction of blood vessels to reduce edema B. Isotonic action to lubricate and reduce symptoms C. Competing at histamine receptor sites D. Inhibiting release of mast cells

C 12 mcg/mL

The patient receiving phenytoin (Dilantin) has a serum drug level drawn. Which level will the nurse note as therapeutic? A 8 mcg/mL B 30 mcg/mL C 12 mcg/mL D 6 mcg/mL

C "This is called the 'on-off phenomenon.' Your health care provider can change your medication regimen slightly to help diminish this effect."

The patient with Parkinson's disease who has been positively responding to carbidopa-levodopa (Sinemet) suddenly develops a relapse of symptoms. Which explanation by the nurse is appropriate? A "You have obviously developed resistance to your current medication and will have to be switched to another drug." B "This is an atypical response. Unfortunately, there are no other options of drug therapy to give you." C "This is called the 'on-off phenomenon.' Your health care provider can change your medication regimen slightly to help diminish this effect." D "You just need to keep taking your medication and these effects will go away."

b Malignant hyperthermia

The patient's chart notes the administration of dantrolene (Dantrium) immediately postoperatively. What does the nurse expect the patient has experienced? a Delirium tremens b Malignant hyperthermia c Tonic-clonic seizure d Respiratory arrest

d. It needs to be taken every day on a continuous schedule, even if symptoms improve.

The prescriber has changed the patient's medication regimen to include the leukotriene receptor antagonist montelukast (Singulair) to treat asthma. The nurse will emphasize which point about this medication? a. The patient needs to keep it close by at all times to treat acute asthma attacks. b. When the asthma symptoms improve, the dosage schedule can be tapered and eventually discontinued. c. The proper technique for inhalation must be followed. d. It needs to be taken every day on a continuous schedule, even if symptoms improve.

C Alzheimer's disease

The provider has ordered donepezil (Aricept) for the patient, and the patient states "I have no idea why I take this medication." What is the most common diagnosis associated with the administration of donepezil (Aricept)? A Parkinson's disease B Bladder retention C Alzheimer's disease D Urinary retention

a. It prevented the sale of drugs that had not been thoroughly tested before marketing.

The student nurse is preparing for the first pharmacology exam. In planning for this exam, what is most important for the student nurse to remember about the Food, Drug, and Cosmetic Act? a. It prevented the sale of drugs that had not been thoroughly tested before marketing. b. It prohibited the sale of drugs labeled with false therapeutic claims that could defraud the public. c. It gave the government the power to control the labeling content of medications. d. It helped standardize the quality of drugs and cosmetics.

Monoamine oxidase (MAO) inhibitors (drugs, mechanism, use, toxicity)

Tranylcypromine, Phenelzine, Isocarboxazid, Selegiline (selective MAO-B inhibitor) -Mechanism: nonselective MAO inhibition increases levels of amine neurotransmitters (norepinephrine, 5-HT, dopamine). -Use: atypical depression, anxiety. -Toxicity: hypertensive crisis (most notably with ingestion of tyramine, which is found in many foods such as wine and cheese); CNS stimulation. -Contraindicated with SSRIs, TCAs, St. John's wort, meperidine, dextromethorphan (to prevent serotonin syndrome).

Esomeprazole

Treatment of PUD and GERD, hypersecretory conditions Dental Considerations: Xerostomia

Chloroquine clinical use

Treatment of plasmodial species other than P. falciparum (frequency of resistance in P. falciparum is too high). Resistance due to membrane pump that intracellular concentration of drug. Treat P. falciparum with artemether/lumefantrine or atovaquone/proguanil. For life-threatening malaria, use quinidine in U.S. (quinine elsewhere) or artesunate.

High potency typical antipsychotics

Trifluoperazine, fluphenazine, haloperidol "Try to Fly through the Halo"

A person is suffering Anemia due to penicillin. what type of reaction is this?

Type 2.(cytotoxic/cytolytic) complement dependent reaction involving either immunoglobiulin G or M antibodies. The antigen-antibody complex is fixed to a circulating red blood cell, resulting in lysis.

Glitazones/ thiazolidinediones clinical use

Used as monotherapy in type 2 DM or combined with other agents

Effective dose 50

What is the dose that produces a desired response in 50% of the subjects?

Hypertension

What is the drug hydrochlorothiazide used for?

C To balance cholinergic and dopaminergic activity in the brain

What is the goal of pharmacologic therapy in treating Parkinson's disease? A To increase the amount of acetylcholine at the presynaptic neurons B To decrease the amount of dopamine available in the substantia nigra C To balance cholinergic and dopaminergic activity in the brain D To block dopamine receptors in both presynaptic and postsynaptic neurons

gingival hyperplasia

What is the most common side effect for taking Calcium Channel Blockers that effects the oral cavity?

Hypertension and angina pectoris

What is the most common usage for Calcium channel blockers?

D. They decrease inflammation.

What is the role of corticosteroids in the treatment of acute respiratory disorders? A. They stimulate the immune system. B. They directly dilate the bronchi. C. They increase gas exchange in the alveoli. D. They decrease inflammation.

c. Pharmacodynamics

What is the study of the physiochemical properties of drugs and how they influence the body called? a. Pharmacokinetics b. Pharmacotherapeutics c. Pharmacodynamics d. Pharmacology

Efficacy

What is the term which has to do with the "Effectiveness" of a drug, efficacy or potency?

d Health Insurance Portability and Accountability Act

What legislation, which was passed in 1996, ensures the privacy of patient information is protected? a Federal Food, Drug, and Cosmetic Act b Durham-Humphrey Amendment c Medicare Prescription Drug, Improvement, and Modernization Act d Health Insurance Portability and Accountability Act

a U.S. Food and Drug Administration (FDA)

What organization announced new regulations requiring bar codes for all prescription and over-the-counter medications? a U.S. Food and Drug Administration (FDA) b Drug Enforcement Agency (DEA) c Federal Bureau of Investigation (FBI) d Department of Health and Human Services (DHHS)

Loop Diuretic

What pharmacologic category does the drug furosemide belong to?

a Assessment

What phase of the nursing process would be a priority for the nurse when providing education on a new medication to a patient and family? a Assessment b Outcome planning c Evaluation d Goal identification

B. Dry, nonproductive cough

What side effect of angiotensin-converting enzyme (ACE) inhibitors most often results in the provider changing the treatment plan to an angiotensin receptor blocker? A. Orthostatic hypotension B. Dry, nonproductive cough C. Fatigue D. Hypokalemia

pharmacodynamics

What the drug does to the body and its function (Mechanism of Action)

Beta lactam

What type of antibiotic is amoxicillin?

a, d, e

What vitamins are most likely to lead to toxic levels, hypervitaminosis, when administered in megadoses? (Select all that apply.) A. Vitamin A B. Vitamin B complex C. Vitamin C D. Vitamin D E. Vitamin K

b. Document that the medication has been administered

What will the nurse do immediately after giving a PRN pain medication? a. Ask if the patient's pain has been relieved b. Document that the medication has been administered c. Recheck the patient's vital signs d. Notify the prescriber

b. Assess the patient's ability to swallow

When administering an oral medication to a client, the nurse should: a. Administer the medication with grapefruit juice b. Assess the patient's ability to swallow c. Crush enteric coated medications if the patient has difficulty swallowing them whole d. Avoid giving the medication to a patient with a nasogastric tube

a. 0.5 mg

When administering digoxin immune Fab (Digibind) to a patient with severe digoxin toxicity, the nurse knows that each vial can bind with how much digoxin? a. 0.5 mg b. 5.5 mg c. 5 mg d. 15 mg

D. Orange juice

When administering ferrous sulfate (iron) to a patient, the nurse plans to give this medication with which fluid to increase absorption of the iron? A. 8 ounces of water B. 4 ounces of milk C. Hot tea D. Orange juice

A. Depressed deep tendon reflexes

When administering intravenous magnesium sulfate, the nurse will monitor for which signs and symptoms of hypermagnesemia? A. Depressed deep tendon reflexes B. Hyperthermia C. Diaphoresis D. Tachycardia

b Drop the prescribed number of drops into the conjunctival sac.

When administering ophthalmic eyedrops, the nurse will perform which action? a Place the eyedrops directly on the cornea. b Drop the prescribed number of drops into the conjunctival sac. c Hold the eyedropper 4 to 5 cm above the eye to avoid contamination. d Document that a smaller dose was absorbed if the patient blinks during administration.

B. Apply the ointment to a nonhairy part of the upper torso.

When applying nitroglycerin ointment, the nurse should perform which action? A. Use the fingers to spread the ointment evenly over a 3-inch area. B. Apply the ointment to a nonhairy part of the upper torso. C. Massage the ointment into the skin. D. Apply two thick lines of ointment over the prescribed measured area on the nitroglycerin paper.

a, c, d

When assessing a patient for adverse effects related to morphine sulfate, which effects would the nurse expect to find? (Select all that apply.) a. Decreased peristalsis b. Diarrhea c. Delayed gastric emptying d. Urinary retention

a Respiratory rate

When assessing for the most serious adverse reaction to a narcotic analgesic, what does the nurse monitor for in the patient? a Respiratory rate b Heart rate c Blood pressure d Mental status

d 3 to 4 finger widths below the antecubital space.

When planning to administer an intradermal medication, the nurse recognizes that the preferred site of injection on the forearm is a just above the wrist. b just below the antecubital space. c 2 to 4 finger widths above the wrist. d 3 to 4 finger widths below the antecubital space.

C. 30 minutes before meals and at bedtime.

When planning to administer metoclopramide (Reglan), the nurse is aware that this drug must be given A. with food to decrease gastrointestinal upset. B. every 6 hours around the clock. C. 30 minutes before meals and at bedtime. D. with a full glass of water first thing in the morning.

A. Diarrhea

When preparing an adult formula gastrostomy tube feeding for a patient, the nurse is aware that which is the most common adverse effect? A. Diarrhea B. Gastroesophageal reflux C. Allergic reaction D. Flatulence

B. 4 mg/min

When preparing to administer furosemide (Lasix) intravenously to a patient with renal dysfunction, the nurse plans implementation based on knowledge that the medication should be administered no faster than which rate? A. 2 mg/min B. 4 mg/min C. 6 mg/min D. 8 mg/min

D abrupt medication withdrawal may lead to a rebound hypertensive crisis.

When teaching a patient about beta blockers such as atenolol (Tenormin) and metoprolol (Lopressor), it is important to inform the patient that A these medications may be taken with antacids to minimize gastrointestinal distress. B hot baths and showers will help enhance the therapeutic effects and are encouraged. C alcohol intake is encouraged for its vasodilating effects. D abrupt medication withdrawal may lead to a rebound hypertensive crisis.

A Carbidopa decreases levodopa's conversion in the periphery, increasing the levodopa available to cross the blood-brain barrier.

When teaching a patient about carbidopa-levodopa (Sinemet), what information will the nurse include in the teaching? A Carbidopa decreases levodopa's conversion in the periphery, increasing the levodopa available to cross the blood-brain barrier. B Carbidopa increases levodopa's conversion in the periphery, enhancing the amount of dopamine available to the brain. C Giving both drugs together minimizes side effects. D Carbidopa crosses the blood-brain barrier to increase the metabolism of levodopa to dopamine in the brain.

B. Black tarry stools

When teaching a patient about potential side effects of NSAID therapy, the nurse will teach the patient to promptly report which effect? A. Mild indigestion B. Black tarry stools C. Nonproductive cough D. Nausea

B. Leg cramps

When teaching a patient about signs and symptoms of hypokalemia, the nurse will instruct the patient to notify the health care provider if which occurs? A. Diaphoresis B. Leg cramps C. Constipation D. Blurred vision

c. "Regular, consistent dosing is important for successful treatment."

When teaching a patient about taking a newly prescribed antiepileptic drug (AED) at home, the nurse will include which instruction? a. "Driving is allowed after 2 weeks of therapy." b. "If seizures recur, take a double dose of the medication." c. "Regular, consistent dosing is important for successful treatment." d. "Antacids can be taken with the AED to reduce gastrointestinal adverse effects."

b Methylphenidate (Ritalin) is a C-II narcotic that cannot be refilled and can only be filled with a written prescription.

When teaching a patient about the legalities regarding a prescription for methylphenidate (Ritalin), which statement is most accurate? a Methylphenidate (Ritalin) is a C-I narcotic that can only be prescribed according to an approved protocol. b Methylphenidate (Ritalin) is a C-II narcotic that cannot be refilled and can only be filled with a written prescription. c Methylphenidate (Ritalin) is a C-III narcotic for which a prescription will expire in 6 months. d Methylphenidate (Ritalin) is a C-IV narcotic that is only allowed to be refilled five times per prescription.

a. "Apply the drops into the conjunctival sac instead of directly onto the eye."

When teaching a patient about the proper application of timolol (Timoptic) eyedrops, the nurse will include which instruction? a. "Apply the drops into the conjunctival sac instead of directly onto the eye." b. "Tilt your head forward before applying the eyedrops." c. "Apply the drops directly to the eyeball (cornea) for the best effect." d. "Blot your eye with a tissue immediately after applying the drops."

C. ketorolac (Toradol)

Which NSAID would the nurse anticipate administering parenterally for the treatment of acute postoperative pain? A. allopurinol (Zyloprim) B. indomethacin (Indocin) C. ketorolac (Toradol) D. diclofenac (Cataflam)

d. Massaging the site after injection

Which action, when performed by the nurse, will increase the absorption of a medication administered intramuscularly? a. Applying cold packs to the injection site b. Lowering the extremity below the level of the heart c. Administering the medication via the Z-track method d. Massaging the site after injection

a, c, d

Which actions describe the beneficial effects produced by sulfonylurea oral hypoglycemics? (Select all that apply.) A. Stimulate insulin secretion from beta cells B. Increase hepatic glucose production C. Enhance action of insulin in various tissues D. Inhibit breakdown of insulin by liver

b, c, d, e, f

Which actions will the nurse perform to ensure the medication dose for a pediatric patient is correct? (Select all that apply.) a. Use the patient's weight in pounds. b. Use a current drug reference to determine usual dosage per 24 hours. c. Determine the dose parameters by multiplying the weight by the minimum and maximum daily doses of the drug (the safe range). d. Determine the total amount of the drug to administer per dose and per day. e. Compare the drug dosage prescribed with the calculated safe range. f. If the drug dosage prescribed varies from recommended reference range, notify the provider.

b, c, d

Which are common side effects of nitroglycerin? (Select all that apply.) A. Blurred vision B. Flushing C. Headache D. Hypotension

C. regular insulin (Humulin R)

Which insulin can be administered by continuous intravenous infusion? A. insulin glargine (Lantus) B. insulin aspart (Novolog) C. regular insulin (Humulin R) D. insulin detemir (Levemir)

B. Risk for injury related to thrombocytopenia

Which nursing diagnosis is appropriate for a patient receiving famotidine (Pepcid)? A. Risk for infection related to immunosuppression B. Risk for injury related to thrombocytopenia C. Impaired urinary elimination related to retention D. Ineffective peripheral tissue perfusion related to hypertension

c Risk for injury

Which nursing diagnosis is appropriate for a patient who has received a sedative-hypnotic drug? a Ineffective peripheral tissue perfusion b Fluid volume excess c Risk for injury d Risk for infection

Lipid soluble, Non-Ionized, Neutral or slightly alkaline

Which of the following drugs generally are easily absorbed into cells?

Class IA sodium channel blocker clinical use

Both atrial and ventricular arrhythmias, especially re-entrant and ectopic SVT and VT

D. Inhibiting HMG-CoA reductase, the enzyme responsible for the biosynthesis of cholesterol in the liver

By which action does atorvastatin (Lipitor) decrease lipid levels? A. Inhibiting absorption of dietary cholesterol in the small intestine B. Binding to bile in the intestinal tract, forming an insoluble complex that is excreted in the feces C. Stimulating the biliary system to increase excretion of dietary cholesterol D. Inhibiting HMG-CoA reductase, the enzyme responsible for the biosynthesis of cholesterol in the liver

c Uracil (U)

Deoxyribonucleic acid (DNA), the primary molecule in the body that serves to transfer genes from parents to offspring, is composed of four different organic bases. Which is not one of the four organic bases of DNA? a Adenine (A) b Guanine (G) c Uracil (U) d Cytosine (C)

SNRIs use

Depression. Venlafaxine is also used in generalized anxiety disorder, panic disorder, PTSD. Duloxetine is also indicated for diabetic peripheral neuropathy

For a pill taken orally, there are four stages.

Disruption: initial disruption of coating or shell. disintegration; contents must break apart dispersion; particles must spread dissolution; drug must be dissolved in GI fluid.

Agonist

Drug produces an effect and has affinity for a receptor on cell membrane —gives action

What is the shortest acting β-blocker

Esmolol

Pramlintide action

-Amylin analog -Decrease gastric emptying -Decrease glucagon

Macrolides

-Azithromycin, clarithromycin, erythromycin -Inhibit protein synthesis by blocking translocation ("macroslides"); bind to the 23S rRNA of the 50S ribosomal subunit. Bacteriostatic. -Atypical pneumonias (Mycoplasma, Chlamydia, Legionella), STIs (Chlamydia), gram-positive cocci (streptococcal infections in patients allergic to penicillin), and B. pertussis. Toxicity: MACRO: Gastrointestinal Motility issues, Arrhythmia caused by prolonged QT interval, acute Cholestatic hepatitis, Rash, eOsinophilia. Increases serum concentration of theophyllines, oral anticoagulants. Clarithromycin and erythromycin inhibit cytochrome P-450. Resistance: methylation of 23S rRNA-binding site prevents binding of drug.

Benzodiazepines intoxication

-Greater safety margin that barbiturates; ataxia, minor respiratory depression. -Treatment: flumazenil (benzodiazepine receptor antagonist, but rarely used as it can precipitate seizures).

Acyclovir, famciclovir, valacyclovir clinical use

-HSV and VZV. Weak activity against EBV. No activity against CMV. Used for HSV- induced mucocutaneous and genital lesions as well as for encephalitis. Prophylaxis in immunocompromised patients. No effect on latent forms of HSV and VZV. Valacyclovir, a prodrug of acyclovir, has better oral bioavailability. -For herpes zoster, use famciclovir.

THYROID DRUGS

-Hypothyroidism levothyroxine (Synthroid) Hashimoto's -Hyperthyroidism due to Grave's Disease goiter

Naloxone (mechanism, use, adverse effects)

-Mechanism: antagonist at all opioid receptor sites -Use: treatment of acute opioid overdose; low dose treatment for adverse effects of opioid agonist delivered IV or epidural -Adverse effects: can precipitate withdrawal syndrome in individuals already receiving full agonists (particularly in dependent users) -CI: pregnancy→ crosses placenta, can precipitate withdrawal in fetus

Lithium mechanism

-Mechanism: not established; possibly related to inhibition of phosphoinositol cascade.

Idiosyncratic reaction

-Not a side effect or allergic reaction - just random reaction -Sometimes genetic reaction

Class IC sodium channel blockers toxicity

-Proarrhythmic, especially post-MI (DO NOT USE POST MI!) -Contraindicated in structural and ischemic heart disease

a. The somatropin dosage may be adjusted.

A 16-year-old boy who is taking somatropin comes into the office because he had an asthma attack during a race at school. Because of this new development, the nurse expects which intervention to occur next? a. The somatropin dosage may be adjusted. b. His growth will be documented and monitored for changes. c. He will need to stop participating in school physical education classes. d. The somatropin must be discontinued immediately.

a, b, d, f

A cholinergic drug is prescribed for a patient. The nurse checks the patient's medical history, knowing that this drug is contraindicated in which disorders? (Select all that apply.) a. Bradycardia b. Chronic obstructive pulmonary disease c. Bladder atony d. Gastrointestinal obstruction e. Alzheimer's disease f. Hypotension

A. "This is a typical adverse effect and may lessen over time. If this continues, we need to schedule an appointment with your provider."

A patient is prescribed epinephrine eye drops for treatment of chronic open-angle glaucoma. The patient complains of burning with instillation of the drops. How would the nurse respond to the patient? A. "This is a typical adverse effect and may lessen over time. If this continues, we need to schedule an appointment with your provider." B. "That should not occur. Come to the office now." C. "This only occurs if the drop is not placed in the correct area of the eye." D. "This is normal and will continue as long as you use the drops."

a, b, d, e

A patient is prescribed selegiline, an MAO-B inhibitor, as adjunctive therapy in treatment of Parkinson's disease. What potential adverse effects would the nurse include in education for this patient? (Select all that apply.) A Headache B Insomnia C Weight gain D Depression E Blood pressure changes

d. It slows down the formation of thyroid hormone.

A patient who is taking propylthiouracil (PTU) for hyperthyroidism wants to know how this medicine works. Which explanation by the nurse is accurate? a. It destroys overactive cells in the thyroid gland. b. It blocks the action of thyroid hormone. c. It inactivates already existing thyroid hormone in the bloodstream. d. It slows down the formation of thyroid hormone.

C. "These are the most common side effects of nitroglycerin. They should subside with continued use of nitroglycerin."

A patient who is taking sublingual nitroglycerin is complaining of flushing and headaches. What is the nurse's best response? A. "This is a normal response to your chest pain." B. "Stop taking the nitroglycerin because you are probably allergic to it." C. "These are the most common side effects of nitroglycerin. They should subside with continued use of nitroglycerin." D. "These symptoms are not related to your sublingual nitroglycerin. You should notify your doctor for diagnostic testing."

a. The patient will need to restrict fat intake to less than 30% to help reduce these adverse effects.

A patient who started taking orlistat (Xenical) 1 month ago calls the clinic to report some "embarrassing" adverse effects. She tells the nurse that she has had episodes of "not being able to control my bowel movements." Which statement is true about this situation? a. The patient will need to restrict fat intake to less than 30% to help reduce these adverse effects. b. These are expected adverse effects that will eventually diminish. c. The patient will need to stop this drug immediately if these adverse effects are occurring. d. The patient will need to increase her fat intake to prevent these adverse effects.

c. Adrenocortical insufficiency

A patient's medication order indicates that he is to receive a dose of cosyntropin (Cortrosyn). The nurse is aware that this drug is used to diagnose which condition? a. Diabetes insipidus b. Pituitary dwarfism c. Adrenocortical insufficiency d. Myasthenia gravis

d Phase IV

A pharmaceutical company is voluntarily conducting a postmarketing study to obtain further proof of the therapeutic effects of a new drug. What phase of drug study is this considered? a Phase I b Phase II c Phase III d Phase IV

True/false there are 6 schedules of controlled substances.

False, 5.

Rosuvastatin

Familial Hypercholesterolemia/ Hyperlipidemia/ Hypertriglyceridemia/ Prevention of CVD Dental considerations: Muscle weakness/ Myopathy

Amphotericin B toxicity

Fever/chills ("shake and bake"), hypotension, nephrotoxicity, arrhythmias, anemia, IV phlebitis ("amphoterrible"). Hydration nephrotoxicity. Liposomal amphotericin toxicity.

SSRIs

Fluoxetine, paroxetine, sertraline, citalopram.

c. Monitoring the patient continuously for therapeutic as well as adverse effects

For which activity is the nurse responsible during the evaluation phase of drug administration? a. Preparing and administering prescribed medications safely b. Planning measurable outcomes for the patient related to drug therapy c. Monitoring the patient continuously for therapeutic as well as adverse effects d. Gathering data in a drug and dietary history

a, c, d

For which potential adverse effects would the nurse monitor patients prescribed amiodarone? (Select all that apply.) A. Bluish skin discoloration B. Diarrhea C. Hypothyroidism D. Photosensitivity E. Hypertension

Meperidine mechanism

Full agonist at μ receptor with antimuscarinic effects

Methadone mechanism

Full agonist at μ receptor; NMDA receptor antagonist and MAOI

Brand Name: Amoxil

Generic Name: Amoxicillin

Brand Name: Lipitor

Generic Name: Atorvastatin

Brand Name: Zithromax

Generic Name: Azithromycin

Brand Name: HCTZ

Generic Name: Hydrochlorothiazide

Vicodin®

Generic Name: Hydrocodone/ APAP

Brand Name: Synthroid®

Generic Name: Levothyroxine

Brand Name: Prinivil

Generic Name: Lisinopril

Adverse effect of chloramphenicol during pregnancy

Gray baby syndrome (vomiting, ashen gray color of the skin, limp body tone, hypotension, cyanosis of lips and skin, hypothermia, cardiovascular collapse, within 2-9 days of birth-especially premature)

Simeprevir MoA

HCV protease inhibitor; prevents viral replication

Drugs that cause adrenocortical insufficiency

HPA suppression 2° to glucocorticoid withdrawal

Dihydropyridine calcium channel blockers clinical use

HTN, angina (including Prinzmetal), Raynaud phenomenon. **NOT nimodipine which is used for subarachnoid hemorrhage to prevent cerebral vasospasm)

Non-dihydropyridine calcium channel blockers clinical use

HTN, angina, atrial fib/flutter

HMG-CoA reductase side effects/problems

Hepatotoxicity ( LFTs), myopathy (esp. when used with fibrates or niacin)

B. "I will need to keep the nitroglycerin stored in the bottle it comes in."

Which statement indicates that the patient understands discharge teaching about nitroglycerin? A. "I will need to refill my prescription once a year in order to maintain potency." B. "I will need to keep the nitroglycerin stored in the bottle it comes in." C. "I will take a nitroglycerin tablet every 15 minutes until my chest pain is gone." D. "I should take a nitroglycerin tablet 2 hours before I want to engage in activity that will cause chest pain."

C "It is okay to take this drug with a small glass of wine to help relax me."

Which statement made by a patient demonstrates a lack of understanding of patient teaching regarding phenothiazine drug therapy? A "I need to change positions slowly to prevent dizziness." B "I will need to wear sunscreen and protective clothing when outdoors." C "It is okay to take this drug with a small glass of wine to help relax me." D "I should call my doctor if I notice any uncontrollable movements of my tongue."

C. "I will rinse the eyedropper with tap water after each use."

Which statement, made by a patient, indicates the need for further patient teaching regarding proper administration of eyedrops? A. "I will put pressure on the inside corner of my eye after I administer the drops." B. "I will be careful not to touch my eye with the dropper." C. "I will rinse the eyedropper with tap water after each use." D. "I will turn my head slightly toward the outside of the eye I am putting the drops in."

A. Serum electrolytes

Which would be a priority assessment prior to administering eplerenone (Inspra)? A. Serum electrolytes B. Level of consciousness C. Patient's knowledge level D. Thiocyanate levels

A Rebound seizure activity

While completing discharge activity for a patient prescribed an antiepileptic drug, the nurse instructs the patient that what potential complication could occur if he or she abruptly stopped taking the antiepileptic drug? A Rebound seizure activity B Acute withdrawal syndrome C Hypotension D Confusion and delirium

B. Massages the site after administration of the medication

While observing a patient self-administer enoxaparin (Lovenox), the nurse identifies the need for further teaching when the patient completes which action? A. Does not aspirate prior to injecting the medication B. Massages the site after administration of the medication C. Administers the medication greater than 2 inches away from the umbilicus D. Injects the medication and then waits 10 seconds before withdrawing the needle

A Absence seizures

While obtaining a patient history, the nurse notes that the patient has been prescribed ethosuximide (Zarontin). Which condition will the nurse suspect that the patient experiences? A Absence seizures B Tonic-clonic seizures C Panic attacks D Partial seizures

c Anesthetic-induced complications

While taking a patient's history prior to surgery for a cardiac problem, the patient informs the nurse he is addicted to cocaine. The nurse alerts the anesthesiologist of the finding because the nurse knows the use of cocaine can cause which effect when a patient is under anesthesia? a Immediate withdrawal symptoms b Blood-clotting problems c Anesthetic-induced complications d Complications during recovery

C Maximally reducing seizure activity while minimizing side effects of medication therapy.

While teaching a patient newly diagnosed with a seizure disorder, what does the nurse state as the goal of pharmacologic therapy of this medication? A Eradicating all seizure activity and then weaning off medication once the patient is seizure free for 3 months. B Reducing seizure occurrence to one per week. C Maximally reducing seizure activity while minimizing side effects of medication therapy. D Maximizing drug dosages to control seizure activity.

Serotonin syndrome

With any drug that increases 5-HT (e.g., MAO inhibitors, SNRIs, TCAs) hyperthermia, confusion, myoclonus, cardiovascular instability, flushing, diarrhea, seizures. -Treatment: cyproheptadine (5-HT2 receptor antagonist).

c. Hyperchloremia

You are called to your patient's room by a family member who voices concern about the patient's status. On assessment, you find the patient tachypnic, lethargic, weak, and exhibiting a diminished cognitive ability. You also find 3+ pitting edema. What electrolyte imbalance would you suspect? a. Hypercalcemia b. Hyponatremia c. Hyperchloremia d. Hypophosphatemia

a. Diminished deep tendon reflexes

You are caring for a patient admitted with a diagnosis of renal failure. When you review your patient's laboratory reports, you note that the patient's magnesium levels are high. What would be important for you to assess? a. Diminished deep tendon reflexes b. Tachycardia c. Cool, clammy skin d. Increased serum magnesium

b. Lactated Ringer's

You are caring for a patient in liver failure who is exhibiting signs and symptoms of hypovolemic shock. You anticipate that the physician will order the administration of a crystalloid for the management of this patient. Which crystalloid fluid is most commonly used to treat hypovolemic shock? a. Albumin b. Lactated Ringer's c. 3% NaCl d. Dextran

b. Lower doses are required.

You are caring for a patient who is on thrombolytic therapy. The patient has a comorbidity of renal insufficiency. What do you know about the amount of heparin required to treat this patient? a. You do not need to use an infusion pump for this patient. b. Lower doses are required. c. You cannot use heparin to treat this patient. d. Higher doses are required.

c. Whatever fluid that is most readily available in the ICU, due to the nature of the emergency

You are working in the ICU and have just been notified that you are receiving a patient from the Obstetrics unit who is in hypovolemic shock due to massive blood loss during delivery. You know that the best choice for fluid replacement for this patient is what? a. 5% Albumin because it is inexpensive and is always readily available b. Dextran because it increases intravascular volume and counteracts coagulopathy c. Whatever fluid that is most readily available in the ICU, due to the nature of the emergency d. Lactated Ringer's solution because it increases volume, buffers acidosis, and is the best choice for patients with liver failure

NSAIDs (drugs, mechanism, use, toxicity)

Ibuprofen, naproxen, indomethacin, ketorolac, diclofenac. -Mechanism: reversibly inhibit cyclooxygenase (both COX-1 and COX-2). Block prostaglandin synthesis. -Use: antipyretic, analgesic, anti-inflammatory. Indomethacin is used to close a PDA. -Toxicity: interstitial nephritis, gastric ulcer (prostaglandins protect gastric mucosa), renal ischemia (prostaglandins vasodilate afferent arteriole).

c Impaired gas exchange related to respiratory depression

In developing a plan of care for a patient receiving morphine sulfate, which nursing diagnosis is a priority? a Acute pain b Risk for injury related to central nervous system side effects c Impaired gas exchange related to respiratory depression d Constipation related to gastrointestinal side effects

c the chemoreceptor trigger zone.

In monitoring a patient for adverse effects related to morphine sulfate, the nurse assesses for stimulation of a autonomic control over circulation. b the cough reflex center. c the chemoreceptor trigger zone. d respiratory rate.

Urine NaCl changes with diuretic therapy

Increase with all diuretics except acetazolamide. Serum NaCl may decrease as a result

Nicotine withdrawal

Irritability, anxiety, craving. Treatment: nicotine patch, gum, or lozenges; bupropion/ varenicline

Marijuana withdrawal

Irritability, depression, insomnia, nausea, anorexia. Most symptoms peak in 48 hours and last for 5-7 days. Generally detectable in urine for up to 1 month.

Drugs that cause seizures

Isoniazid (vitamin B6 deficiency), Bupropion, Imipenem/cilastatin, Enflurane

a. Never, it rapidly enters red blood cells, causing them to rupture.

Isotonic IV fluids are fluids with a total osmolality close to that of the ECF. Most IV fluids contain either dextrose or electrolytes in water. When would you infuse electrolyte-free water intravenously? a. Never, it rapidly enters red blood cells, causing them to rupture. b. When the patient is severely dehydrated c. When the patient is in an excess of an electrolyte, i.e. hypercalcemia d. When the patient is in a deficit of an electrolyte, i.e. hypocalcemia

A. acne.

Isotretinoin (Amnesteem) and tretinoin (Retin-A) are common dermatologic drugs used to treat A. acne. B. male pattern baldness. C. keloid scarring. D. fungal infections.

D. Erectile dysfunction medications

It is most important to instruct a patient prescribed nitroglycerin to avoid which substance(s)? A. Antacids B. Grapefruit juice C. Thiazide diuretics D. Erectile dysfunction medications

Glucocorticoids mechanism

Metabolic, catabolic, anti-inflammatory, and immunosuppressive effects mediated by interactions with glucocorticoid response elements, inhibition of phospholipase A2, and inhibition of transcription factors such as NF-κB.

Drugs that cause interstitial nephritis

Methicillin, NSAIDs, furosemide

Drugs that cause direct Coombs- positive hemolytic anemia

Methyldopa, penicillin

Amphetamine & -Like (MoA, use, adverse effects, contraindications, example)

MoA-modulation of seratonergic pathways occurs by affecting changes in dopamine transport. USE-hyperactivity caused by ADHD, increase attention-span, and control narcolepsy AR-tachycardia, HTN, seizures, growth suppression, transient wt loss in children, blurred vision, irritability, tremors, dry mouth, euphoria, headache, n/v, dizziness. CI-glaucoma, tourettes, anxiety, hyperthyroidism, psychosis, depression INTERACTS-decrease anti-htn effects, barbituates, increase oral anti-coag effects, anti-convulsants, and TCA's, Increase hypertensive crisis with MAOI's, caffeine may increase effects EX-Methylphenidate (Ritalin), Modafinil (Provigil)

Neuromuscular blocking drugs

Muscle paralysis in surgery or mechanical ventilation. Selective for motor (vs. autonomic) nicotinic receptor.

5-fluorouracil (5-FU) toxicity

Myelosuppression, which is not reversible with leucovorin (folinic acid)

Atypical antipsychotics mechanism

Not completely understood. Varied effects on 5-HT2, dopamine, and α- and H1-receptors

Monoamine Oxidase Inhibitors (MAOIS)

Not used much anymore due to many interactions with foods, wines, other drugs o Can cause hypertensive crisis and death

c Obtain a family history

Nurses working in the general population are required to have a basic understanding of the role of the nurse in genetics. Which is an appropriate role for a general nurse related to genetics? a Provide genetic testing options b Suggest genetic counseling c Obtain a family history d Develop a genetic testing policy

C. Increase fluid intake to decrease viscosity of secretions.

Patient teaching regarding expectorants should instruct the patient to perform which action? A. Restrict fluids to decrease mucus production. B. Take the medication once a day only, usually at bedtime. C. Increase fluid intake to decrease viscosity of secretions. D. Increase fiber and fluid intake to prevent constipation.

C. "This medication works by preventing the inflammation that causes your asthma attack."

Patient teaching regarding the use of antileukotriene drugs such as zafirlukast (Accolate) would include which statement? A. "Take the medication as soon as you begin wheezing." B. "It will take about 3 weeks before you notice a therapeutic effect." C. "This medication works by preventing the inflammation that causes your asthma attack." D. "Increase fiber and fluid in your diet to prevent the common side effect of constipation."

Indirect cholinomimetics as glaucoma drugs

Physostigmine, echothiophate -Mechanism: increase outflow of aqueous humor via contraction of ciliary muscle and opening of trabecular meshwork -Side effects: miosis and cyclospasm (contraction of ciliary muscle)

Ezetimibe mechanism of action

Prevents cholesterol absorption at the small intestine brush border

Sans: Ne is the transmitter released by the postganglionic nerves. this is called :

adrenergic

PCP withdrawal

Depression, anxiety, irritability, restlessness, anergia, disturbances of thought and sleep.

ADRENOCORTICOSTEROID DRUGS

(-SONE, -LONE) -palliative - NOT curative -Prevent the release of substances in the body that cause inflammation. -suppress the immune system Useful in treating many conditions: -Inflammation -Allergic reactions -Autoimmune disorders -Addison's disease: take steroids -Asthma Examples: cortisone, hydrocortisone, prednisone, triamcinolone

Metabolism

(aka biotransformation) -changing a drug so that it can be excreted -Excreted product is metabolite - more ionized and less lipid soluble than a parent compound (cant absorb so excretes) -Most common location is the liver Metabolism mechanisms: -Active drug to inactive metabolite - (most common ) -Inactive drug to active metabolite - inactive prodrug (valcyclovir ) -Active drug to active metabolite - long half life (valium)

Allergic reaction

*NOT predictable *NOT dose related -Mild reactions treated with antihistamine and possibly steroids -Life-threatening (anaphylaxis) treated with epinephrine injection -4 types

Short acting insulin (regular) mechanism

-Binds insulin receptor (tyrosine kinase activity) -Liver: increase glucose stored as glycogen -Muscle: increase glycogen, protein synthesis; increase K+ uptake -Fat: increase TG storage

ANTIANXIETY Barbiturates

-(-tal) -(Bad BARBie is Tall and Bad.. will be in jail a Long time) Long acting & narrow therapeutic index (narrow margin of safety) -Replaced by benzodiazepines for anxiety -for anticonvulsant -Produce CNS depression -NO analgesic effect BOARD ALERT

Chloramphenicol

-Blocks peptidyltransferase at 50S ribosomal subunit. -Bacteriostatic. -Use: Meningitis (Haemophilus influenzae, Neisseria meningitidis, Streptococcus pneumoniae) and Rocky Mountain spotted fever (Rickettsia rickettsii). Limited use owing to toxicities but often still used in developing countries because of low cost. -Toxicity: anemia (dose dependent), aplastic anemia (dose independent), gray baby syndrome (in premature infants because they lack liver UDP-glucuronyl transferase). -Resistance: plasmid-encoded acetyltransferase inactivates the drug.

Adverse effects of adrenergic agents

-CNS disturbances (anxiety, fear, tension, headache, tremor) -Cardiac arrythmias -Cerebral hemorrhage -these should not be used in patients with angina, uncontrolled hypertension and uncontrolled hyperthyroidism ie. epinephrine

Typical antipsychotic toxicity

-Highly lipid soluble and stored in body fat; thus, very slow to be removed from body. -Extrapyramidal system side effects (e.g., dyskinesias). Treatment: benztropine or diphenhydramine. -Endocrine side effects (e.g., dopamine receptor antagonism hyperprolactinemia galactorrhea). -Side effects arising from blocking muscarinic (dry mouth, constipation), α1 (hypotension), and histamine (sedation) receptors. -Can cause QT prolongation.

Overuse of calcium carbonate

-Hypercalcemia, rebound acid increase -Can chelate and decrease effectiveness of other drugs

Carbapenems

-Imipenem, meropenem, ertapenem, doripenem -Imipenem is a broad-spectrum, β-lactamase- resistant carbapenem. Always administered with cilastatin (inhibitor of renal dehydropeptidase I) to decrease inactivation of drug in renal tubules -Use: gram-positive cocci, gram-negative rods, and anaerobes. Wide spectrum, but significant side effects limit use to life-threatening infections or after other drugs have failed. Meropenem has arisk of seizures and is stable to dehydropeptidase I -Toxicity: GI distress, skin rash, and CNS toxicity (seizures) at high plasma levels

Zolpidem (mechanism, use, toxicity)

-Mechanism: selective agonist at GABA-A α1-site; shortens sleep latency, prolongs sleep time; onset w/in 30 minutes; extended release available→ 7 hours of sleep; T ½: 2-4 hours; greater in hepatic insufficiency & the elderly -Use: approved for bedtime use to sleep through the night, especially if one has lots of awakenings; can try to fall asleep & then use after not falling asleep for 30 min. (don't use in the middle of the night b/c you'll be drowsy if you don't get 7 hours); should taper off for 5 weeks -Tolerance and dependance rare

Buspirone

-Mechanism: stimulates 5-HT1A receptors. -Use: generalized anxiety disorder. Does not cause sedation, addiction, or tolerance. Takes 1-2 weeks to take effect. Does not interact with alcohol (vs. barbiturates, benzodiazepines).

Glycopyrrolate

-Muscarinic antagonist -Parental use: preoperative use to reduce airway secretions -Oral use: drooling, peptic ulcer

Amphotericin B clinical use

-Serious, systemic mycoses. Cryptococcus (amphotericin B with/without flucytosine for cryptococcal meningitis), Blastomyces, Coccidioides, Histoplasma, Candida, Mucor. -Intrathecally for fungal meningitis. -Supplement K+ and Mg2+ because of altered renal tubule permeability

Hydralazine clinical use

-Severe HTN (particularly acute), HF (with organic nitrate). -Safe to use in pregnancy. -Frequently coadministered with a β-blocker to prevent reflex tachycardia.

General mechanism of sodium channel blockers (class I antiarrhythmic)

-Slow or block conduction (especially in depolarized cells). -Decrease slope of phase 0 depolarization. -Are state dependent - selectively depress tissue that is frequently depolarized (e.g., tachycardia)

Calcium channel blockers effect on the pacemaker cell action potential curve

-Slow rise of AP (decrease conduction velocity) -Increased ERP -Increased PR interval -Prolonged repolarization (at AV node)

Sulfonamides

-Sulfamethoxazole (SMX), sulfisoxazole, sulfadiazine -Inhibit folate synthesis. Para-aminobenzoic acid (PABA) antimetabolites inhibit dihydropteroate synthase. Bacteriostatic (bactericidal when combined with trimethoprim). (Dapsone, used to treat lepromatous leprosy, is a closely related drug that also inhibits folate synthesis.) -Gram-positives, gram-negatives, Nocardia, Chlamydia. Triple sulfas or SMX for simple UTI. -Toxicity: Hypersensitivity reactions, hemolysis if G6PD deficient, nephrotoxicity (tubulointerstitial nephritis), photosensitivity, kernicterus in infants, displace other drugs from albumin (e.g., warfarin). -Resistance: Altered enzyme (bacterial dihydropteroate synthase), decreased uptake, or increased PABA synthesis.

Tetracyclines

-Tetracycline, doxycycline, minocycline -Bacteriostatic; bind to 30S and prevent attachment of aminoacyl-tRNA; limited CNS penetration. Doxycycline is fecally eliminated and can be used in patients with renal failure. Do not take tetracyclines with milk (Ca2+), antacids (Ca2+ or Mg2+), or iron-containing preparations because divalent cations inhibit drugs' absorption in the gut. -Clinical use: Borrelia burgdorferi, M. pneumoniae. Drugs' ability to accumulate intracellularly makes them very effective against Rickettsia and Chlamydia. Also used to treat acne. -Toxicity: GI distress, discoloration of teeth and inhibition of bone growth in children, photosensitivity. Contraindicated in pregnancy. -Resistance: decrease uptake or increased efflux out of bacterial cells by plasmid-encoded transport pumps.

ANTIHISTAMINE DRUGS

-ine -Block histamine release from H1 Receptors -Main side effects: xerostomia and sedation -diphenhydramine (Benadryl) -fexofenadine (Allegra) -cetirizine (Zyrtec) -loratadine (Claritin)

DIABETES DRUGS Type 1- Insulin Dependent Diabetes Mellitus (IDDM)

-lack of insulin secretion -Most common medication is insulin given by subcutaneous injection -common side effect is hypoglycemia* insulin shock confusion weakness, slurred speech Causes of insulin shock -Too much insulin given -Failure to eat with medication -Increase exercise or STRESS - can lead to dental emergency -Some dental drugs affect insulin demand - raise blood sugar - use with caution in diabetic patients -epinephrine -Glucocorticosteroids

Common examples of B-Blockers (-olol)

-metoprolol (Lopressor) -cardio-selective more specific for Bi (heart) less side effects -propranol (Inderal) -affects B1(heart) and B2(lungs) meaning more side effects

•Adrenergic Blocking Drugs

-olol drugs -a-blocking drugs - increased blood flow Used to treat Raynaud's syndrome (poor circulation) -B-blocking drugs - block the B-receptors Used mainly to treat cardiac conditions -Some off-label uses •Migraine headaches •Generalized anxiety disorder -propranolol (Inderal) -metoprolol (Lopressor, Toprol) B1 -atenolol (Tenormin)

Relative contraindications of prilocaine

-patients with methemoglobinemia - condition in which the iron in hemoglobin does not carry oxygen effectively -Sickle cell and other anemias -Patients taking acetaminophen raises methemoglobin

Albuterol

-β2 > β1 direct agonist -Acute asthma

Maximum recommended dose of epinephrine is based on the patient's health status —Healthy dose

0.2mg epi per dental appt ASA I or II

How long does it take for the patent of the original drug expires, and other companies can market the same compound under a generic name?

17 years

Pregnant patient receiving lido with 1:100,000 epi can receive

2 cartridges of anesthetic 0.04mg/0.018mg = 2.22 = 2 cartridges

SSRIs mechanism

5-HT-specific reuptake inhibitors.

b. The medication should be given 4 to 6 hours before bedtime to diminish insomnia.

A 6-year-old boy has been started on an extended-release form of methylphenidate hydrochloride (Ritalin) for the treatment of attention deficit hyperactivity disorder (ADHD). During a follow-up visit, his mother tells the nurse that she has been giving the medication at bedtime so that it will be "in his system" when he goes to school the next morning. What is the nurse's appropriate evaluation of the mother's actions? a. The medication should be taken with meals for optimal absorption. b. The medication should be given 4 to 6 hours before bedtime to diminish insomnia. c. She is giving him the medication dosage appropriately. d. The medication should not be taken until he is at school.

A. gemfibrozil (Lopid)

A patient with elevated triglyceride levels unresponsive to HMG-CoA reductase inhibitors will most likely be prescribed which drug? A. gemfibrozil (Lopid) B. cholestyramine (Questran) C. colestipol (Colestid) D. simvastatin (Zocor)

B. The patient must use at least two forms of contraception while taking this medication.

A teenage female patient is going to be prescribed isotretinoin (Amnesteem) for acne. Which information is most important for the nurse to include in the education of the patient? A. The drug is supplied in oral form. B. The patient must use at least two forms of contraception while taking this medication. C. The drug is used in treatment of severe acne. D. Providers who prescribe this medication must be registered with the FDA

b. Pregnancy test

A woman visits a health center requesting oral contraceptives. Which laboratory test is most important for the nurse to assess before the patient begins oral contraceptive therapy? a. Vaginal cultures b. Pregnancy test c. Serum potassium level d. Complete blood count

Lisinopril

ACE inhibitor (-pril) -HA, dizziness, hypotension, loss of taste, oral ulcers, cough, orofacial angioedema, scalded mouth syndrome -NSAIDs potentially decrease effectiveness, caution with position change, watch for hyperkalemia

b. Chronic obstructive pulmonary disease

Abatacept (Orencia) is prescribed for a patient with severe rheumatoid arthritis. The nurse checks the patient's medical history, knowing that this medication would need to be used cautiously if which condition is present? a. Hypertension b. Chronic obstructive pulmonary disease c. Diabetes mellitus d. Coronary artery disease

Heparin mechanism

Activator of antithrombin; decrease thrombin and decrease factor Xa. Short half-life.

Class IB sodium channel blockers clinical use

Acute ventricular arrhythmias (especially post- MI), digitalis-induced arrhythmias. (IB is Best post-MI)

Glucocorticoids clinical use

Addison disease, inflammation, immunosuppression, asthma

OPIOID NARCOTIC ANALGESICS Drug Interactions Additive

Additive effects with other CNS depressants - causing respiratory depression Alcohol, Antianxiety medications

No drug is free from producing what?

Adverse effects.

c propofol (Diprivan)

An intubated, mechanically ventilated patient in the intensive care unit is becoming increasingly restless and anxious. The nurse expects to administer which intravenous anesthetic drug? a isoflurane (Forane) b nitrous oxide c propofol (Diprivan) d halothane (Fluothane)

Sirolimus (Rapamycin) toxicity

Anemia, thrombocytopenia, leukopenia, insulin resistance, hyperlipidemia; not nephrotoxic (kidney "sir-vives")

Clinical use of epoetin alfa (erythropoietin)

Anemias (especially in renal failure)

Metoprolol

Angina/ Heart failure/ Hypertension Dental considerations: Enhances response to epinephrine

-sartan

Angiotensin-II receptor blocker

Clonazepam

Anticonvulsant - Drowsiness, ataxia, behavior disorders - low stress environment (consider BZDP), seizure control history often

Sodium Valproate

Anticonvulsant - Possible bleeding, GI, HA, ataxia, drowsiness, tremor, thrombocytopenia - low stress environment (consider BZDP), seizure control history often

Gemfibrozil

Antihyperlipidemic - fibrate - GI, abnormal taste

Cephalosporins

B-lactam antibiotics (like penicillin) - bactericidal cephalexin (Keflex) ear infection

Glucocorticoids (the ridiculist)

Beclomethasone, dexamethasone, fludrocortisone (mineralocorticoid and glucocorticoid activity), hydrocortisone, methylprednisolone, prednisone, triamcinolone. Basically anything ending in -sone/lone

D. protamine sulfate

Before emergency surgery, the nurse would anticipate administering which medication to a patient receiving heparin? A. vitamin K B. vitamin E C. phenytoin D. protamine sulfate

Calcium channel blockers (class IV antiarrhythmics) toxicity

Constipation, flushing, edema, CV effects (HF, AV block, sinus node depression)

Barbiturates withdrawal

Delirium, life-threatening cardiovascular collapse.

Overuse of magnesium hydroxide

Diarrhea, hyporeflexia, HoTN, cardiac arrest

Adverse effect of tetracyclines during pregnancy

Discolored teeth, inhibition of bone growth

B. oxytocin (Pitocin)

During a postpartum assessment, the nurse notes a boggy uterus and increased vaginal bleeding. Based upon this assessment and standing physician orders, the nurse prepares to administer which medication? A. prostaglandin E B. oxytocin (Pitocin) C. terbutaline (Brethine) D. clomiphene (Clomid)

c "Most drugs produce sedation at low doses and sleep, the hypnotic effect, at higher doses."

During patient teaching, the nurse explains the difference between a sedative and hypnotic with which statement? a "Sedatives are much stronger than hypnotic drugs and should only be used for short periods of time." b "Sedative drugs induce sleep, whereas hypnotic drugs induce a state of hypnosis." c "Most drugs produce sedation at low doses and sleep, the hypnotic effect, at higher doses." d "There really is no difference; the terms are used interchangeably."

vasoconstrictors: Two available in US

Epinephrine -1:100,000 → 0.018mg epi/cartridge -1:200,000 (lowest conc.) → 0.009mg epi/cartridge Levonordefrin -1:20,000 || Carti

C Increased heart rate and contractility

Epinephrine, as an adrenergic (sympathomimetic) drug, produces which therapeutic effect? A Urinary retention B Bronchial constriction C Increased heart rate and contractility D Decreased intestinal motility

A vasoconstriction.

Ergot alkaloids such as ergotamine tartrate (Ergostat) exert a therapeutic effect by causing A vasoconstriction. B vasodilation. C blockade of the beta2 receptors. D simulation of the alpha receptors.

Drugs that cause acute cholestatic hepatitis, jaundice

Erythromycin

Two groups of anesthetics →

Esters and Amides

Sympathetic Response (SANS)

Fight or flight- STRESS -Increased blood pressure (vasoconstriction) -Increased heart rate -Increased oxygen to muscles -Decreased peristalsis -Dilates bronchioles -Dilates pupils (myDriasis) -Decreased salivary flow *Adrenergic agents /sympathomimetics

Drugs that cause tendonitis, tendon rupture, and cartilage damage

Fluoroquinolones

Brand Name: Glucophage

Generic Name: Metformin

Brand Name: Lopressor

Generic Name: Metoprolol

Brand Name: Singulair

Generic Name: Montelukast

Brand Name: Crestor

Generic Name: Rosuvastatin

Brand Name: Zoloft

Generic Name: Sertraline

Acyclovir, famciclovir, valacyclovir MoA

Guanosine analogs. Monophosphorylated by HSV/VZV thymidine kinase and not phosphorylated in uninfected cells few adverse effects. Triphosphate formed by cellular enzymes. Preferentially inhibit viral DNA polymerase by chain termination.

Inhaled anesthetics (drugs, mechanism, use, toxicity)

Halothane, enflurane, isoflurane, sevoflurane, methoxyflurane, N2O -Mechanism: unknown. -Use: myocardial depression, respiratory depression, nausea/emesis, cerebral blood flow ( cerebral metabolic demand). -Toxicity: hepatotoxicity (halothane), nephrotoxicity (methoxyflurane), proconvulsant (enflurane), expansion of trapped gas in a body cavity (N2O). *Can cause malignant hyperthermia—rare, life-threatening hereditary condition in which inhaled anesthetics (except N2O) and succinylcholine induce fever and severe muscle contractions. Treatment: dantrolene.

Disopyramide toxicity

Heart failure

Ribavirin toxicity

Hemolytic anemia; severe teratogen.

Drugs that cause thrombocytopenia

Heparin

Pramlintide toxicity

Hypoglycemia, nausea, diarrhea.

What are the major functions of the Histamine-1 receptor?

Increase nasal and bronchial mucus production, increase vascular permeability, contraction of bronchioles, pruritis, pain

Glitazones/ thiazolidinediones mechanism

Increase sensitivity in peripheral tissue. Binds to PPAR-γ nuclear transcription regulator.

Bismuth, sucralfate use

Increased ulcer healing, travelers' diarrhea

Barbiturates intoxication

Low safety margin, marked respiratory depression. Treatment: symptom management (e.g., assist respiration, BP).

Amides

Metabolized in the LIVER ••lidocaine (Xylocaine) -Safe in pregnancy (Category B) and during lactation -Available in combination with epinephrine (not plain in US) ••mepivacaine (Carbocaine) -Available with levonordefrin (neo-cobefrin) ••prilocaine (Citanest) -Associated with methemoglobinemia

List the key β-blockers (class II antiarrhythmics)

Metoprolol, propranolol, esmolol, atenolol, timolol, carvedilol

b Amnesia

Midazolam (Versed) has been ordered for a patient to be administered by injection 30 minutes prior to a colonoscopy. The nurse informs the patient that one of the most common side effects of this medication is which effect? a Decreased heart rate b Amnesia c Constipation d Dry mouth

Cyclosporine toxicity

Nephrotoxicity, hypertension, hyperlipidemia, neurotoxicity, gingival hyperplasia, hirsutism.

Adverse Effects of B-blockers

Non-specific B-blockers will block B2 receptors (lungs)- causing shortness of breath in asthmatics (Ex. propranolol) not for asthmatics

a, b, d

Nursing care for a patient receiving alteplase (Activase) would include which action? (Select all that apply.) A. Record vital signs and report changes. B. Observe for signs and symptoms of bleeding. C. Monitor liver enzymes. D. Assess for cardiac dysrhythmias. E. Administer injections intramuscularly to prevent bleeding.

Drugs that cause thrombotic complications

OCPs, hormone replacement therapy

Adverse Reactions ACE Inhibitors (-pril)

Orthostatic hypotension ***Dry, hacking cough**

C. Achlorhydria

Proton pump inhibitors have the ability to almost totally inhibit gastric acid secretion. Due to this possibility, the use of the medication can lead to what problem? A. Gastric ulcer formation B. Gastroesophageal reflux disease (GERD) C. Achlorhydria D. Diverticulosis

a, b, c

Potassium-sparing diuretics may cause which adverse reactions? (Select all that apply.) A. Hyperkalemia B. Dizziness C. Headache D. Hypermagnesemia E. Muscle cramps

-prost

Prostaglandin analog

List the class IA sodium channel blockers

Quinidine, procainamide, disopyramide,

Ezetimibe side effects/problems

Rare increase in LFTs, diarrhea

What is the treatment for M. tuberculosis?

Rifampin, isoniazid, pyrazinamide, ethambutol (RIPE)

Drugs that cause hepatitis

Rifampin, isoniazid, pyrazinamide, statins, fibrates

Neuroleptic malignant syndrome (NMS)

Rigidity, myoglobinuria, autonomic instability, hyperpyrexia. Treatment: dantrolene, D2 agonists (e.g., bromocriptine).

Fluoxetine (Prozac)

SSRI - HA, insomnia, irritability -BRUXISM (TX WITH buspirone or switch to mirtazapine)

Sertraline (Zoloft)

SSRI - decrease diazepam clearance -BRUXISM (TX WITH buspirone or switch to mirtazapine)

Citalopram (Celexa)

SSRI - nausea, dry mouth, sedation, insomnia - much less dry mouth than TCA -BRUXISM (TX WITH buspirone or switch to mirtazapine)

Paroxetine (paxil)

SSRI - nausea, sedation, dry mouth, dizziness -BRUXISM (TX WITH buspirone or switch to mirtazapine)

Escitalopram (Lexapro)

SSRI - reduced side effects, well tolerated -BRUXISM (TX WITH buspirone or switch to mirtazapine)

Nitroprusside

Short acting; increase cGMP via direct release of NO. Can cause cyanide toxicity (releases cyanide)

K+-sparing diuretics (drugs, mechanism, use, toxicity)

Spironolactone and eplerenone; Triamterene, and Amiloride. -Mechanism: spironolactone and eplerenone are competitive aldosterone receptor antagonists in cortical collecting tubule. Triamterene and amiloride act at the same part of the tubule by blocking Na+ channels in the cortical collecting tubule. -Use: hyperaldosteronism, K+ depletion, HF. -Toxicity: hyperkalemia (can lead to arrhythmias), endocrine effects with spironolactone (e.g., gynecomastia, antiandrogen effects).

ADA

The Council on Dental Therapeutics is part of what larger organization?

d. Monitor the client's serum potassium level.

The client diagnosed with pernicious anemia is prescribed cyanocobalamin (Cyanabin), Vitamin B12. Which intervention should the nurse implement? a. Administer the subcutaneous injection into the lower outer aspect of the abdomen. b. Double-check the dose with another registered nurse. c. Instruct the client to sip the medication through a straw. d. Monitor the client's serum potassium level.

A "Ropinirole is a dopamine agonist that has fewer side effects than carbidopa-levodopa."

The patient asks the nurse to explain the difference between carbidopa-levodopa (Sinemet) and ropinirole (Requip). How will the nurse respond? A "Ropinirole is a dopamine agonist that has fewer side effects than carbidopa-levodopa." B "Carbidopa-levodopa is less effective than ropinirole in treating the symptoms of Parkinson's disease." C "Both drugs have the same pharmacodynamic and side effect profiles." D "Carbidopa-levodopa acts as a dopamine agonist, whereas ropinirole directly replaces dopamine."

Liver

The primary site of drug metabolism.

Long acting insulin clinical use

Type 1 DM, type 2 DM, GDM (basal glucose control)

Short acting insulin clinical use

Type 1 DM, type 2 DM, GDM, DKA (IV), hyperkalemia (+ glucose), stress hyperglycemia.

Nimodipine clinical use

Used in subarachnoid hemorrhage to prevent cerebral vasospasm)

Nitrate mechanism of action

Vasodilate by increasing NO in vascular smooth muscle thereby increasing cGMP and smooth muscle relaxation. Dilate veins >> arteries. Reduces preload

Active Transport and Passive Transport

What are two ways drugs are transported into a cell?

List the calcium channel blockers (class IV antiarrhythmics)

Verapamil, diltiazem

Foscarnet MoA

Viral DNA/RNA polymerase inhibitor and HIV reverse transcriptase inhibitor. Binds to pyrophosphate-binding site of enzyme. Does not require activation by viral kinase. Foscarnet = pyrofosphate analog.

Warfarin toxicity antidote

Vitamin K (delayed effect), fresh frozen plasma (immediate)

Somatic Nervous system

We can control our _______.

Glitazones/ thiazolidinediones toxicity

Weight gain, edema, hepatotoxicity, HF, increased risk if fractures

Treatment of hypertension, angina pectoris

What are beta blockers most commonly used for?

hypertension

What are loop diuretics such as furosemide commonly used for?

Prinivil, Zestril

What are the common Trade Names for lisinopril?

Moxatag, Amoxil, Trimox

What are three common Trade names for Amoxicillin:

D risperidone (Risperdal)

What atypical antipsychotic medication would the nurse anticipate a provider prescribing for treatment of refractory schizophrenia? A trazodone (Desyrel) B phenelzine (Nardil) C amoxapine (Asendin) D risperidone (Risperdal)

A. B cells

What cells are the functional cells of the humoral immune system that mature into immunoglobulins? A. B cells B. Granulocytes C. T cells D. Red blood cells

Relief of severe pain

What is hydrocodone commonly used for?

d. Having direct action on the cough center

When giving dextromethorphan, the nurse understands that this drug suppresses the cough reflex by which mechanism of action? a. Decreasing the viscosity of the bronchial secretions b. Causing depression of the central nervous system c. Anesthetizing the stretch receptors d. Having direct action on the cough center

a. Remove the old ointment before new ointment is applied.

When the nurse is administering topical nitroglycerin ointment, which technique is correct? a. Remove the old ointment before new ointment is applied. b. Apply the ointment on the skin on the forearm. c. Massage the ointment gently into the skin, and then cover the area with plastic wrap. d. Apply the ointment only in the case of a mild angina episode.

C. Magnesium-containing antacids

Which type of antacids will the nurse most likely question in an order for a patient with chronic renal failure? A. Aluminum-containing antacids B. Calcium-containing antacids C. Magnesium-containing antacids D. Sodium-containing antacids

Side effect-

a dose related reaction that is not part of the desired therapeutic outcome. The drug acts on nontraget organs to produce an undesirable effect

what is schedule 3 handling procedures?

abuse potential is moderate. Examples Codeine mixtures. Prescriptions may be telephoned; no more than five prescriptions in 6 months.

nonopioids-

also called non narcotic, periheral, mild and antipyretic analgesics

aspirin's mechanism of action?

analgesic (pain), antipyretic (fever reducer), antiinflammatory, and antiplatelet (anti aggregated) is due to its ability to inhibit prostaglandin synthesis.

prn

as needed

Carvedilol

beta-blocker (+++ beta and alpha-1) - +++ agents used for HF so disease limits epi to 0.04mg - NSAIDs may reduce effectiveness, may cause xerostomia

Propranolol

beta-blocker (++, non-selective beta-1 and beta-2 - Limit epi to 0.04mg with ++ agents - NSAIDs may reduce effectiveness, may cause xerostomia

Atenolol

beta-blocker (+, selective beta-1) - Can use up to 0.2mg epic with + agents. - NSAIDs may reduce effectiveness, may cause xerostomia

Bisoprolol

beta-blocker (+, selective beta-1) - Can use up to 0.2mg epic with + agents. - NSAIDs may reduce effectiveness, may cause xerostomia

Nitrous oxide/ oxygen sedation can reduce the orofacial muscle tone in

cerebral palsy patients during dental treatment

acteylcholine is a neurotransmitter. nerves that release this are called?

cholinergic

Opioids are NOT used for

chronic pain (like TMJ)

Adrenergic Agents-Epinephrine is contraindicated in

cocaine or methamphetamine abuser.

Enamel erosion on the lingual surfaces of the maxillary anterior teeth-

common in pts with GERD

Pans is concerned with:

conservation of body processes. both digestion and intestinal tract motility are greatly influenced by the pans

myosis

constriction of pupils

Relative contraindications to vasoconstrictors: Nonselective B-blockers

hypertension -propranolol (Inderal)

Relative contraindications to vasoconstrictors: Tricyclic antidepressants (TCA)

hypertension amitriptyline (Elavil)

albuterol can cause

insomnia. all adrenergic drugs can - fight or flight response

effects of local reactions?

irritation pain, and tissue necrosis at the site of injection.

The median effect dose?ED 50

is the dose required to produce a specified intensity of effect in 50% of the animals

what is schedule 5 handling procedures?

least, some codeine-containing cough syrups. Can be bought OTC in some states.

most drugs are metabolized in the?

liver

ANTIHYPERLIPIDEMIC DRUGS

lower level of cholesterol in the blood by reducing the production of cholesterol by the liver - (they inhibit HMG-COA reductase the enzyme that aids in cholesterol synthesis) -atorvastatin (Lipitor) -lovastatin (Mevacor) -rosuvastatin (Crestor) -simvastatin (Zocor) -Statins =cholesterol

Therapeutic index

measurement of the safety of a drug

Kidney (renal) excretion is

most common. Also, from lungs, saliva, breast milk, Gingival crevicular fluid.

because the postsynaptic tissue responds to muscarine, it is called?

muscarinic found in poisonous mushrooms, and decaying fish

Lipid soluble drugs are not excreted in

must be metabolized by liver into water soluble form to be excreted in urine.

Heading on a prescription pad requires?

name, address, and telephone number of the prescribe. same of patient. date of prescription.

Are the efficacy and the potency of a drug related?

nope

Common examples of CCBS (Always Very Nice Day - except with ging enlargement!)

o (a)mlodipine (Norvasc) o (v)erapamil (Isoptin, Calan) o (n)ifedipine (Procardia, Adalat)-high probability of gingival enlargement!! o (d)iltiazem (Cardizem, Dilacor)

Enteral Route

o Drug placed directly in GI tract - :- orally or rectally o Safe, cheap, convenient o Large absorbing area -Requires patient cooperation -Orally delivered drugs must pass through hepatic portal circulation which can inactive some drugs - Called the first-pass effect. P-450 enzymes in the liver are responsible. more drugs needed then

can be used for premedication if there is a penicillin allergy

o azithromycin & clarithromycin

Opioid Agonists

o morphine prototype o oxycodone (Oxycontin, Percocet) o hydrocodone (Vicodin, Norco) •codeine (Tylenol #2, #3, #4 and cough syrup) o meperidine (Demerol) o hydromorphone (Dilaudid) •fentanyl, sufentanil, alfentanil

Antagonism

occurs when a combination of two agents produces less effect than either agent alone

where have people searched for active components in drugs?

plants, animals, minerals, and soil. today organic synthetic chemistry researchers are responsible primarily for developing new drugs.

How do cholinergic effect the eyes?

produces miosis (contraction) and cycloplegia (loss of accommodation) useful in the treatment of glaucoma.

BOARD ALERT: Epi should NOT be used on patients taking specific B-Blockers such as? (Q may be in testlet or case study)

propranolol. can result in greatly increased vasopressor response causing HYPERTENSION. Avoid using epinephrine-containing product or use cardiac dose.

what is the most common type of excretion of drugs?

renal excretion

Pans. postganglionic fibers, originate in ganglia, they are usually _____ and ________ on the innervated tissue

short and Terminate

the space between the preganglionic and the post ganglionic fibers is called the ?

synapse

tab

tablet

PKa

the percent the drug is 50/50 nonionized to ionized.

When the two formulations prove to have an equal therapeutic effect in a clinical trial,

therapeutically equivalent.

sig

write

what is schedule 2 handling procedures?

written prescription with providers;signature only; no refills.

Terbutaline, ritodrine

β2-agonists that relax the uterus; used to contraction frequency in women during labor.

Nystatin clinical use

"Swish and swallow" for oral candidiasis (thrush); topical for diaper rash or vaginal candidiasis.

ANTIANXIETY DRUGS Benzodiazepines

(-pam, -lam) • Fast acting (Benz fast car picknup Pam and Lam) -NO analgesic effect- BOARD ALERT -Additive with other CNS depressants

Toxic reaction

(aka overdose reaction) *Expected response - exaggeration of the therapeutic effect *Dose related *Target organs

Cholinergic Drugs

(parasympathomimetics - MIMIC the PANS) -Bradycardia, -Increased motility -Miosis - pupil constriction, decrease intraocular pressure -Large doses produce SLUD (toxic effect - exaggerated therapeutic effect) **increase salivation**

Adrenergic Drugs

(sympathomimetics MIMIC the SANS) -a-receptors- vasoconstriction use Epinephrin -B1 receptors - 1 HEART Increased rate and strength of contraction of heart -B2 receptors- 2 LUNGS Relaxation of smooth muscles of bronchioles - bronchodilation

ANTIHYPERTENSIVE Beta Blockers (B -Adrenergic Blocking Agents)

- think letter blocks OLOL (baby-beta) -Block stimulation of ß receptors decreasing cardiac output - lowering BP -Effectiveness reduced by NSAIDS -Adverse Reactions Orthostatic hypotension

Esters

-*Metabolized in the plasma / BLOOD -High potential for allergic reactions -BOARD ALERT - Benzocaine is an ester and available as a topical -procaine (Novocain) - no longer used in dentistry -propoxycaine (Ravocaine) -tetracaine (Pontocaine)

Evolution of EPS side effects

-4 hr acute dystonia (muscle spasm, stiffness, oculogyric crisis) -4 day akathisia (restlessness) -4 wk bradykinesia (parkinsonism) -4 mo tardive dyskinesia

Flutamide

-A nonsteroidal competitive inhibitor at androgen receptors. -Used for prostate carcinoma.

ANTIHYPERTENSIVE DRUGS ACE Inhibitors

-A-pril -Prevent conversion of angiotensin I to angiotensin II → by blocking the ACE (Angiotensin Converting Enzyme) o Angiotensin II causes vasoconstriction... so when blocked, the result is vasodilation, resulting in lowered blood pressure. -Effectiveness reduced by NSAIDS

Rivastigmine

-Anticholinesterse - increases ACh -Alzheimer disease

Physostigmine

-Anticholinesterse - increases ACh -Used in anticholinergic toxicity -Crosses the blood-brain barrier (CNS)

Neostigmine

-Anticholinesterse - increases ACh -Used in postoperative and neurogenic ileus and urinary retention, myasthenia gravis, and postoperative reversal of neuromuscular junction blockade

Dextromethorphan

-Antitussive (antagonizes NMDA glutamate receptors). -Synthetic codeine analog. Has mild opioid effect when used in excess. -Naloxone can be given for overdose. Mild abuse potential. -May cause serotonin syndrome if combined with other serotonergic agents

Protease inhibitor mechanism

-Assembly of virions depends on HIV-1 protease (pol gene), which cleaves the polypeptide products of HIV mRNA into their functional parts. Thus, protease inhibitors prevent maturation of new viruses. -Ritonavir can "boost" other drug concentrations by inhibiting cytochrome P-450. -All protease inhibitors end in -navir. Navir (never) tease a protease.

Meperidine adverse effects

-CSA Schedule 2 -Drug-drug interactions w/ MAOIs & SSRIs (due to weak inhibition of serotonin reuptake and/or inhibition of hepatic CYP enzymes)→ Serotonin Syndrome, Coma, Death -Use w/ MAOIs (or 2 weeks after) is CI

Adverse effects of morphine

-CSA Schedule 2 drug -Behavioral restlessness, tremulousness, hyperactivity, respiratory depression, N/V, ↑intracranial pressure, postural HoTN accentuated by hypovolemia, constipation, urinary retention, itching around nose, urticaria -CI: concomitant use of full w/ partial agonists (or antagonists), ↓pulmonary function, head injury, pregnancy, impaired hepatic or renal function, endocrine dysfunction -Drug Interactions: sedative hypnotics, antipsychotics, MOAIs

Methotrexate (MTX) use

-Cancers: leukemias (ALL), lymphomas, choriocarcinoma, sarcomas. -Non-neoplastic: ectopic pregnancy, medical abortion (with misoprostol), rheumatoid arthritis, psoriasis, IBD, vasculitis.

Drugs for tonic-clonic seizure

-Carbamazepine, valproate, and phenytoin are 1st choice -Phenobarbital, lamotragine, topiramate, and levetiracetam can all be used 2nd line

Nonselective α- and β-antagonists

-Carvedilol, labetalol

Teratogenic effect

-Cause congenital malformations during pregnancy - rapidly dividing cells most susceptive to the effects -FDA defined categories A, B, C, D, X A = least X- most -Dental drugs of concern: Tetracycline, NSAIDS, Benzodiazepines, Metronidazole

2nd generation cephalosporins

-Cefoxitin, cefaclor, cefuroxime -Use: gram-positive cocci, Haemophilus influenzae, Enterobacter aerogenes, Neisseria spp., Proteus mirabilis, E. coli, Klebsiella pneumoniae, Serratia marcescens.

5th generation cephalosporins

-Ceftaroline -Use: broad gram-positive and gram-negative organism coverage, including MRSA; does not cover Pseudomonas.

Digoxin toxicity

-Cholinergic—nausea, vomiting, diarrhea, blurry yellow vision (think van Gogh), arrhythmias, AV block. -Can lead to hyperkalemia, which indicates poor prognosis.

H2 blocker toxicity

-Cimetidine is a potent inhibitor of cytochrome P-450 (multiple drug interactions); it also has antiandrogenic effects (prolactin release, gynecomastia, impotence, decreased libido in males); can cross blood-brain barrier (confusion, dizziness, headaches) and placenta. -Both cimetidine and ranitidine increase renal excretion of creatinine. Other H2 blockers are relatively free of these effects.

List the H2 blockers

-Cimetidine, ranitidine, famotidine, nizatidine. -Take H2 blockers before you "dine". Think "table for 2" to remember H2.

Fluoroquinolones

-Ciprofloxacin, norfloxacin, levofloxacin, ofloxacin, moxifloxacin, gemifloxacin, enoxacin. -Inhibit prokaryotic enzymes topoisomerase II (DNA gyrase) and topoisomerase IV. Bactericidal. Must not be taken with antacids. -Gram-negative rods of urinary and GI tracts (including Pseudomonas), Neisseria, some gram-positive organisms. Toxicity: GI upset, superinfections, skin rashes, headache, dizziness. Less commonly, can cause leg cramps and myalgias. -Contraindicated in pregnant women, nursing mothers, and children < 18 years old due to possible damage to cartilage. Some may prolong QT interval. May cause tendonitis or tendon rupture in people > 60 years old and in patients taking prednisone. -Resistance: chromosome-encoded mutation in DNA gyrase, plasmid-mediated resistance, efflux pumps.

Bethanechol

-Direct cholinergic agonist -Activates bowel and bladder smooth muscle -Used in postoperative and neurogenic ileus -Resistant to AChE

Carbachol

-Direct cholinergic agonist -Carbon copy of acetylcholine -Constricts pupils and relieves intraocular pressure in glaucoma

Pilocarpine

-Direct cholinergic agonist -Contracts ciliary muscle of eye (open angle glaucoma), contracts pupillary sphincter (closed angle glaucoma) -Potent stimulator of sweat, tears and saliva -AChE resistant

Methacholine

-Direct cholinergic agonist -Stimulates muscarinic receptors in airways when inhaled -Used as a challenge test for diagnosis of asthma

Hypertension with heart failure therapy

-Diuretics, ACE inhibitors/ARBs, β-blockers (compensated HF), aldosterone antagonists. -β-blockers must be used cautiously in decompensated HF and are contraindicated in cardiogenic shock.

Oral contraception (synthetic progestins, estrogen)

-Estrogen and progestins inhibit LH/FSH and thus prevent estrogen surge. No estrogen surge, then no LH surge then no ovulation. -Progestins cause thickening of cervical mucus, thereby limiting access of sperm to uterus. Progestins also inhibit endometrial proliferation -> endometrium is less suitable to the implantation of an embryo. -Contraindications: smokers > 35 years old (increased risk of cardiovascular events), patients with history of thromboembolism and stroke or history of estrogen-dependent tumor.

Metformin mechanism

-Exact mechanism unknown. -Decrease gluconeogenesis, increase glycolysis, increase peripheral glucose uptake (increase insulin sensitivity).

Atypical antipsychotic toxicity

-Fewer extrapyramidal and anticholinergic side effects than traditional antipsychotics. -Olanzapine/clozapine may cause significant weight gain. --Clozapine may cause agranulocytosis (requires weekly WBC monitoring) and seizure. -Risperidone may increase prolactin (causing lactation and gynecomastia) decreasing GnRH, LH, and FSH (causing irregular menstruation and fertility issues). -All may prolong QT interval.

Metronidazole

-Forms toxic free radical metabolites in the bacterial cell that damage DNA. Bactericidal, antiprotozoal. -Treats Giardia, Entamoeba, Trichomonas, Gardnerella vaginalis, Anaerobes (Bacteroides, C. difficile). Used with a proton pump inhibitor and clarithromycin for "triple therapy" against H. Pylori. -Treats anaerobic infection below the diaphragm vs. clindamycin (anaerobic infections above diaphragm). Toxicity: Disulfiram-like reaction (severe flushing, tachycardia, hypotension) with alcohol; headache, metallic taste.

Aminoglycosides

-Gentamicin, neomycin, amikacin, tobramycin, streptomycin -Bactericidal; irreversible inhibition of initiation complex through binding of the 30S subunit. Can cause misreading of mRNA. Also block translocation. Require O2 for uptake; therefore ineffective against anaerobes. -Severe gram-negative rod infections. Synergistic with β-lactam antibiotics. -Neomycin for bowel surgery. -Toxicity: Nephrotoxicity, Neuromuscular blockade, Ototoxicity (especially when used with loop diuretics). Teratogen. -Resistance: Bacterial transferase enzymes inactivate the drug by acetylation, phosphorylation, or adenylation.

Pegloticase

-Gout -Recombinant uricase that catalyze metabolism of uric acid into allantoin (a more water soluble product)

Allopurinol

-Gout -Inhibits xanthine oxidase after being converted to alloxanthine, decrease conversion of xanthine to uric acid. Also used in lymphoma and leukemia to prevent tumor lysis-associated urate nephropathy. Increase concentrations of azathioprine and 6-MP (both normally metabolized by xanthine oxidase).

HIV therapy

-Highly active antiretroviral therapy (HAART): often initiated at the time of HIV diagnosis. -Strongest indication for patients presenting with AIDS-defining illness, low CD4+ cell counts (< 500 cells/mm3), or high viral load. -Regimen consists of 3 drugs to prevent resistance: 2 NRTIs and 1 of the following: NNRTI or protease inhibitor or integrase inhibitor.

Glucocorticoids toxicity

-Iatrogenic Cushing syndrome (hypertension, weight gain, moon facies, truncal obesity, buffalo hump, thinning of skin, striae, osteoporosis, hyperglycemia, amenorrhea, immunosuppression), adrenocortical atrophy, peptic ulcers, steroid diabetes, steroid psychosis. -Adrenal insufficiency when drug stopped abruptly after chronic use.

Toxicity of β-blockers

-Impotence, cardiovascular adverse effects (bradycardia, AV block, HF), CNS adverse effects (seizures, sedation, sleep alterations), dyslipidemia (metoprolol), and asthma/COPD exacerbations -Avoid in cocaine users due to risk of unopposed α-adrenergic receptor agonist activity -Despite theoretical concern of masking hypoglycemia in diabetics, benefits likely outweigh risks; not contraindicated

PPI toxicity

-Increased risk of C. difficile infection, pneumonia. -Decreased serum Mg2+ with long term use

Ephedrine

-Indirect general sympathetic agonist -Releases stored catecholamines -Nasal decongestion, urinary incontinence, hypotension.

Cocaine

-Indirect general sympathetic agonist -Reuptake inhibitor -Causes vasoconstriction and local anesthesia. -Never give β-blockers if cocaine intoxication is suspected (can lead to unopposed α1 activation and extreme hypertension).

Amphetamine

-Indirect general sympathetic agonist -reuptake inhibitor; also releases stored catecholamines -Narcolepsy, obesity, ADHD.

α-glucosidase inhibitors action

-Inhibit intestinal brush-border α-glucosidases. -Delayed carbohydrate hydrolysis and glucose absorption leading to decreased postprandial hyperglycemia.

Trimethoprim

-Inhibits bacterial dihydrofolate reductase. Bacteriostatic. -Used in combination with sulfonamides (trimethoprim-sulfamethoxazole [TMP- SMX]), causing sequential block of folate synthesis. Combination used for UTIs, Shigella, Salmonella, Pneumocystis jirovecii pneumonia treatment and prophylaxis, toxoplasmosis prophylaxis. -Toxicity: megaloblastic anemia, leukopenia, granulocytopenia. (May alleviate with supplemental folinic acid). TMP Treats Marrow Poorly.

Azathioprine toxicity

-Leukopenia, anemia, thrombocytopenia. -6-MP degraded by xanthine oxidase; toxicity by allopurinol.

Metabolism Factors

-Liver function- hepatitis, alcoholism -First pass effect liver inactivates some drugs -Other drugs -Half-life First order kinetics :constant percentage of drug is removed -Half-life Zero order kinetics :Constant amount of drug removed from body per unit time (alcohol and aspirin)

Ondansetron (mechanism, clinical use and toxicity)

-Mechanism: 5-HT3 antagonist; decrease vagal stimulation. Powerful central-acting antiemetic. -Use: Control vomiting postoperatively and in patients undergoing cancer chemotherapy. -Toxicity: Headache, constipation, QT interval prolongation.

Flumazenil (mechanism, use, toxicity)

-Mechanism: : competitive BZ receptor antagonist that can block the effects of BZ and any drug that binds to the BZ site, including the Z drugs. Does not block effects of other sedative hypnotics: alcohol, barbiturates or buspirone -Use: antidote to excessive BZ intake -Toxicity: none

Sulfasalazine (mechanism, clinical use and toxicity)

-Mechanism: A combination of sulfapyridine (antibacterial) and 5-aminosalicylic acid (anti-inflammatory). Activated by colonic bacteria. -Use: Ulcerative colitis, Crohn disease (colitis component). -Toxicity: Malaise, nausea, sulfonamide toxicity, reversible oligospermia.

Phenytoin (mechanism, use, side effects)

-Mechanism: Increase Na+ channel inactivation, zero order kinetics -Use: all seizure types except for absence; first line tonic-clonic; first line prophylaxis for status epilepticus -Adverse effects: gingival hyperplasia, hirsutism, peripheral neuropathy, megaloblastic anemia, teratogenesis (fetal hydantoin syndrome), SLE-like syndrome, induction of cytochrome P-450, lymphadenopathy, *Stevens- Johnson syndrome*, osteopenia

Sumatriptan

-Mechanism: Inhibit trigeminal nerve activation; prevent vasoactive peptide release; induce vasoconstriction. -Use: acute migraine, cluster headache attack -Toxicity: Coronary vasospasm (contraindicated in patients with CAD or Prinzmetal angina), mild paresthesia.

Testosterone, methyltestosterone (mechanism, use, toxicity)

-Mechanism: agonists at androgen receptors. -Use: treats hypogonadism and promotes development of 2° sex characteristics; stimulation of anabolism to promote recovery after burn or injury. -Toxicity: causes masculinization in females; decreased intratesticular testosterone in males by inhibiting release of LH (via negative feedback) causing gonadal atrophy. Premature closure of epiphyseal plates. Increased LDL, decreased HDL.

Naltrexone (mechanism, use, adverse effects)

-Mechanism: antagonist at all opioid receptors -Use: alcohol dependence treatment; maintenance treatment to prevent relapse in opioid dependent patients. -Adverse effects: multiple effects including CNS, hepatic & injection site effects; can precipitate withdrawal syndrome in individuals already receiving full agonists (particularly in dependent users) -CI: narcotic dependence or current use of opioid analgesics; compromised liver function

Raloxifene (mechanism, use, toxicity)

-Mechanism: antagonist at breast, uterus; agonist at bone; -Use: primarily to treat osteoporosis -Increased risk of thromboembolic events but no increased risk of endometrial cancer (vs. tamoxifen);

Clomiphene (mechanism, use, toxicity)

-Mechanism: antagonist at estrogen receptors in hypothalamus. Prevents normal feedback inhibition and release of LH and FSH from pituitary, which stimulates ovulation. -Use: to treat infertility due to anovulation (e.g., PCOS). ----Toxicity: may cause hot flashes, ovarian enlargement, multiple simultaneous pregnancies, visual disturbances.

Infliximab, adalimumab

-Mechanism: anti-TNF-α monoclonal antibody. -Use: inflammatory bowel disease, rheumatoid arthritis, ankylosing spondylitis, psoriasis

Estrogens (ethinyl estradiol, DES, mestranol) (mechanism, use, toxicity)

-Mechanism: bind estrogen receptors. -Use: hypogonadism or ovarian failure, menstrual abnormalities, hormone replacement therapy in postmenopausal women; use in men with androgen-dependent prostate cancer. -Toxicity: increased risk of endometrial cancer, bleeding in postmenopausal women, clear cell adenocarcinoma of vagina in females exposed to DES in utero, increased risk of thrombi. -Contraindications—ER ⊕ breast cancer, history of DVTs

Tricyclic antidepressants mechanism, use, and toxicity

-Mechanism: block reuptake of norepinephrine and 5-HT. -Use: major depression, OCD (clomipramine), peripheral neuropathy, chronic pain, migraine prophylaxis. -Toxicity: sedation, α1-blocking effects including postural hypotension, and atropine-like (anticholinergic) side effects (tachycardia, urinary retention, dry mouth). 3° TCAs (amitriptyline) have more anticholinergic effects than 2° TCAs (nortriptyline). Can prolong QT interval. -Tri-C's: Convulsions, Coma, Cardiotoxicity (arrhythmias); also respiratory depression, hyperpyrexia. Confusion and hallucinations in elderly due to anticholinergic side effects (use nortriptyline). Treatment: NaHCO3 to prevent arrhythmia.

Topiramate (mechanism, use, side effects)

-Mechanism: blocks Na+ channels, increases GABA action -Use: simple, complex, and tonic-clonic seizures; also used for migraine prevention -Adverse effects: sedation, mental dulling, kidney stones, weight loss

Ethosuximide (mechanism, use, side effects)

-Mechanism: blocks thalamic T-type Ca2+ channels -Use: Absence seizures (Sux to have Silent (absence) Seizures) -Side effects: *Stevens-Johnson syndrome*, GI, fatigue, headache, urticaria,

Levodopa (L-dopa)/carbidopa

-Mechanism: carbidopa blocks peripheral conversion of L-DOPA to dopamine by inhibiting DOPA decarboxylase. Also reduces side effects of peripheral L-dopa conversion into dopamine (e.g., nausea, vomiting). -Use: Parkinson disease therapy

Acetazolamide (mechanism, use, toxicity)

-Mechanism: carbonic anhydrase inhibitor. Causes self- limited NaHCO3 diuresis and decreased total body HCO3− stores. -Use: glaucoma, urinary alkalinization, metabolic alkalosis, altitude sickness, pseudotumor cerebri. -Toxicity: hyperchloremic metabolic acidosis, paresthesias, NH3 toxicity, sulfa allergy.

Cyclophosphamide, ifosfamide (mechanism, use, toxicity)

-Mechanism: cross-link DNA at guanine N-7. Require bioactivation by liver. -Use: solid tumors, leukemia, lymphomas. -Toxicity: myelosuppression; hemorrhagic cystitis, partially prevented with mesna (thiol group of mesna binds toxic metabolites).

Minoxidil

-Mechanism: direct arteriolar vasodilator -Use: androgenetic alopecia; severe refractory hypertension

Aliskiren (mechanism, use, toxicity)

-Mechanism: direct renin inhibitor, blocks conversion of angiotensinogen to angiotensin I. -Use: hypertension -Toxicity: hyperkalemia, decreased renal function, hypotension. -Contraindicated in diabetics taking ACE inhibitors or ARBs.

Acetazolamide as a glaucoma drug

-Mechanism: diuretic; decrease aqueous humor synthesis via inhibition of carbonic anhydrase -No pupillary or vision changes

Doxorubicin, daunorubicin (mechanism, use, toxicity)

-Mechanism: generate free radicals. Intercalate in DNA causing breaks in DNA decreasing replication. -Use: solid tumors, leukemias, lymphomas. -Toxicity: cardiotoxicity (dilated cardiomyopathy), myelosuppression, alopecia. Toxic to tissues following extravasation. Dexrazoxane (iron chelating agent), used to prevent cardiotoxicity.

Benzodiazepines as anti-epileptic (diazepam, lorazepam)(mechanism, use, side effects)

-Mechanism: increase GABA-A action -Use: first line for acute status epilepticus. Also for eclampsia seizures (1st line is MgSO4) -Adverse effects: sedation, tolerance, dependence, respiratory depression

Carbamazepine (mechanism, use, side effects)

-Mechanism: increase Na+ channel inactivation -Use: first line for simple, complex, and tonic-clonic seizures. 1st line for trigeminal neuralgia -Side effects: blood dyscrasias (agranulocytosis, aplastic anemia), liver toxicity, teratogenesis, induction of cytochrome P-450, SIADH, *Stevens-Johnson syndrome*

Valproic acid/valproate (mechanism, use, side effects)

-Mechanism: increase Na+ channel inactivation, increase GABA concentration by inhibiting GABA transaminase; action at the T-type Ca2+ channels -Use: all seizure types except for status epilepticus; first line for tonic-clonic; also used for myoclonic seizures; bipolar disorder -Adverse effect: GI, distress, rare but fatal hepatotoxicity (measure LFTs), neural tube defects (e.g., spina bifida), tremor, weight gain, contraindicated in pregnancy

Amantadine

-Mechanism: increase dopamine release and decrease dopamine reuptake) -Use: Parkinson disease; also used as an antiviral against influenza A and rubella -Toxicity: ataxia, livedo reticularis

l-dopa (levodopa)/carbidopa

-Mechanism: increase level of dopamine in brain. Unlike dopamine, l-dopa can cross blood-brain barrier and is converted by dopa decarboxylase in the CNS to dopamine. Carbidopa, a peripheral DOPA decarboxylase inhibitor, is given with l-dopa to the bioavailability of l-dopa in the brain and to limit peripheral side effects. -Use: Parkinson disease. -Toxicity: arrhythmias from peripheral formation of catecholamines. Long-term use can lead to dyskinesia following administration ("on-off" phenomenon), akinesia between doses.

Phenobarbital (mechanism, use, side effects)

-Mechanism: increased GABA-A receptor action (keeps receptor open longer) -Use: simple, complex, and tonic-clonic seizures; 1st line in neonates -Adverse effects: sedation, tolerance, dependence, induction of cytochrome P-450, cardiorespiratory depression

Bleomycin (mechanism, use, toxicity)

-Mechanism: induces free radical formation causing breaks in DNA strands. -Use: testicular cancer, Hodgkin lymphoma. -Toxicity: Pulmonary fibrosis, skin hyperpigmentation, mucositis, minimal myelosuppression.

Sildenafil, vardenafil, tadalafil (mechanism, use, toxicity)

-Mechanism: inhibit PDE-5 increasing cGMP, smooth muscle relaxation in corpus cavernosum, increase blood flow, penile erection. "Sildenafil, vardenafil, and tadalafil fill the penis" -Use: erectile dysfunction. -Toxicity: headache, flushing, dyspepsia, cyanopsia (blue-tinted vision). Risk of life-threatening hypotension in patients taking nitrate**

Tramadol (mechanism, use, adverse effects)

-Mechanism: inhibition of serotonin & NE reuptake (main mechanism); μ receptors (partial agonist)→ analgesic effect only partially inhibited by naloxone -Use: mild to moderate pain; chronic neuropathic pain (can be insensitive to opioids) -Adverse effects: as for morphine except - ↓respiratory depression than equivalent doses of morphine, ↓constipation, precipitation of withdrawal on abrupt discontinuation (tapered withdrawal recommended) CI: As for morphine; patients w/ Hx of opioid abuse/addiction (can reinitiate dependence). Avoid using w/ MAOIs & SSRIs (drug-drug interaction)→ Serotonin Syndrome

Dactinomycin/actinomycin D (mechanism, use, toxicity)

-Mechanism: intercalates in DNA. -Use: Wilms tumor, Ewing sarcoma, rhabdomyosarcoma. **Used for childhood tumors ("children act out"). -Toxicity: myelosuppression.

Aspirin (mechanism, use, toxicity)

-Mechanism: irreversibly inhibits cyclooxygenase (both COX-1 and COX-2) via acetylation, which synthesis of TXA2 and prostaglandins. bleeding time. No effect on PT, PTT. A type of NSAID. -Use: low dose (< 300 mg/day): platelet aggregation. Intermediate dose (300-2400 mg/day): antipyretic and analgesic. High dose (2400-4000 mg/day): anti-inflammatory. -Toxicity: gastric ulceration, tinnitus (CN VIII). Chronic use can lead to acute renal failure, interstitial nephritis, GI bleeding. Risk of Reye syndrome in children treated with aspirin for viral infection. Causes respiratory alkalosis early, but transitions to mixed metabolic acidosis-respiratory alkalosis.

Ramelteon (mechanism, use, toxicity)

-Mechanism: melatonin agonist (MT1 & 2); shortens sleep latency -Use: may be used in the elderly, ICU & those who travel -No bad side effects, tolerance or dependency

Rituximab (mechanism, use, toxicity)

-Mechanism: monoclonal antibody against CD20, which is found on most B-cell neoplasms. -Use: non-Hodgkin lymphoma, CLL, IBD, rheumatoid arthritis. Toxicity: increased risk of progressive multifocal leukoencephalopathy.

Mannitol (mechanism, use, toxicity)

-Mechanism: osmotic diuretic; increase tubular fluid osmolarity - increase urine flow, decreasing intracranial/intraocular pressure. -Use: drug overdose, elevated intracranial/intraocular pressure. -Toxicity: pulmonary edema, dehydration. Contraindicated in anuria, HF.

Buspirone (mechanism, use, toxicity)

-Mechanism: partial agonist at 5-HT 1A, acts as agonist when 5-HT is low and an antagonist when 5-HT is high; onset of action takes 1-2 weeks; doesn't produce sedation, confusion, mental clouding; doesn't enhance alcohol or other CNS depressants -Use: reduce cognitive aspects of worry and poor concentration; helps in depression with anxiety; useful with CBT; anxiety in ADHD; alleviates aches, cramps, fatigue, pain, and sexual dysfunction due to GAD -Toxicity: no abuse liability -CI: MAOIs

Codeine (mechanism, use, adverse effects)

-Mechanism: partial agonist at μ receptor; biotransformed into a number of active metabolites: morphine (<10%) -Use: relief of moderate pain; antitussive in selected patients (at doses much less than needed for analgesia) -Adverse effects: same as morphine

Hydrocodone (mechanism, use, adverse effects)

-Mechanism: partial agonist at μ receptor; partially biotransformed to hydromorphone -Use: in combination w/ acetaminophen for relief of mild-moderate pain (MC prescribed generic drug); antitussive in selected patients (doses are ↓than needed for analgesia)→ in combination w/ Guaifenesin or other drugs -Adverse effects: same as morphine

Ethacrynic acid (mechanism, use, toxicity)

-Mechanism: phenoxyacetic acid derivative (not a sulfonamide). Essentially same action as furosemide. -Use: diuresis in patients allergic to sulfa drugs. -Toxicity: similar to furosemide; can cause hyperuricemia; never use to treat gout

Cilostazol, dipyridamole (mechanism, use, toxicity)

-Mechanism: phosphodiesterase III inhibitor; increase cAMP in platelets, resulting in inhibition of platelet aggregation; vasodilators. -Use: intermittent claudication, coronary vasodilation, prevention of stroke or TIAs (combined with aspirin), angina prophylaxis. -Toxicity: nausea, headache, facial flushing, hypotension, abdominal pain.

Dantrolene

-Mechanism: prevents release of Ca2+ from the sarcoplasmic reticulum of skeletal muscle. -Use: malignant hyperthermia and neuroleptic malignant syndrome (a toxicity of antipsychotic drugs).

Trazodone

-Mechanism: primarily blocks 5-HT2 and α1-adrenergic receptors. -Used primarily for insomnia, as high doses are needed for antidepressant effects. -Toxicity: sedation, nausea, priapism, postural hypotension. Called trazobone due to male-specific side effects.

Gabapentin (mechanism, use, side effects)

-Mechanism: primarily inhibits high- voltage-activated Ca2+ channels; designed as GABA analog -Use: partial (focal) seizures - simple and complex. Also used for peripheral neuropathy, postherpetic neuralgia -Adverse effects: sedation and ataxia

Acetaminophen (mechanism, use, toxicity)

-Mechanism: reversibly inhibits cyclooxygenase, mostly in CNS. Inactivated peripherally. -Use: antipyretic, analgesic, but not anti-inflammatory. Used instead of aspirin to avoid Reye syndrome in children with viral infection. -Toxicity: overdose produces hepatic necrosis; acetaminophen metabolite (NAPQI) depletes glutathione and forms toxic tissue byproducts in liver. N-acetylcysteine is antidote—regenerates glutathione.

Celecoxib (mechanism, use, toxicity)

-Mechanism: reversibly inhibits specifically the cyclooxygenase (COX) isoform 2, which is found in inflammatory cells and vascular endothelium and mediates inflammation and pain; spares COX-1, which helps maintain gastric mucosa. Thus, does not have the corrosive effects of other NSAIDs on the GI lining. Spares platelet function as TXA2 production is dependent on COX-1. -Use: rheumatoid arthritis, osteoarthritis. -Toxicity: increased risk of thrombosis. Sulfa allergy.

Tamoxifen, raloxifene (mechanism, use, toxicity)

-Mechanism: selective estrogen receptor modulators (SERMs)—receptor antagonists in breast and agonists in bone. Block the binding of estrogen to ER ⊕ cells. -Use: breast cancer treatment (tamoxifen only) and prevention. Raloxifene also useful to prevent osteoporosis. -Toxicity: tamoxifen—partial agonist in endometrium, which increases the risk of endometrial cancer; "hot flashes." Raloxifene—no increased in endometrial carcinoma because it is an estrogen receptor antagonist in endometrial tissue.

Prednisone, prednisolone (mechanism, use, toxicity)

-Mechanism: various; bind intracytoplasmic receptor; alter gene transcription. -Use: most commonly used glucocorticoids in cancer chemotherapy. Used in CLL, non-Hodgkin lymphoma (part of combination chemotherapy regimen). Also used as immunosuppressants (e.g., in autoimmune diseases). -Toxicity: Cushing-like symptoms; weight gain, central obesity, muscle breakdown, cataracts, acne, osteoporosis, hypertension, peptic ulcers, hyperglycemia, psychosis.

Vincristine, vinblastine (mechanism, use, toxicity)

-Mechanism: vinca alkaloids that bind β-tubulin and inhibit its polymerization into microtubules preventing mitotic spindle formation (M-phase arrest). -Use: Solid tumors, leukemias, Hodgkin (vinblastine) and non-Hodgkin (vincristine) lymphomas. -Toxicity: vincristine: neurotoxicity (areflexia, peripheral neuritis), paralytic ileus. Vinblastine blasts bone marrow (suppression).

Butorphanol (mechanism, use, toxicity)

-Mechanism: κ-opioid receptor agonist and μ-opioid receptor partial agonist; produces analgesia. -Use: severe pain (e.g., migraine, labor). Causes less respiratory depression than full opioid agonists. -Toxicity: can cause opioid withdrawal symptoms if patient is also taking full opioid agonist (competition for opioid receptors). Overdose not easily reversed with naloxone.

Amphetamine

-Mechanism: ↑dopamine & NE tone by blocking their reuptake & facilitating their release -Use: ADHD; improves attention, concentration, execution, wakefulness, hyperactivity

Theophylline

-Methylxanthine -Likely causes bronchodilation by inhibiting phosphodiesterase increasing cAMP levels due to decreased cAMP hydrolysis. -Usage is limited because of narrow therapeutic index (cardiotoxicity, neurotoxicity); metabolized by cytochrome P-450. -Blocks actions of adenosine.

MAC (minimal alveolar concentration)

-Minimal Alveolar Concentration (of inhaled anesthetic) required to prevent 50% of subjects from moving in response to noxious stimulus (e.g., skin incision). -Examples: nitrous oxide (N2O) has blood and lipid solubility, and thus fast induction and low potency. Halothane, in contrast, has lipid and blood solubility, and thus high potency and slow induction.

Class IC sodium channel blockers effect on action potential curve

-Minimal effect on action potential duration -Decreases slope of phase 0

Drug interactions

-More drugs a patient takes, higher the chance - may cause - toxicity, lack of efficacy or increased efficacy

Hyoscyamine

-Muscarinic antagonist -Antispasmodics for IBS

Ipratropium

-Muscarinic antagonist -Competitively blocks muscarinic receptors, preventing bronchoconstriction. Also used for COPD. Tiotropium is long acting.

Scopalamine

-Muscarinic antagonist -Motion sickness

Solifenacin

-Muscarinic antagonist -Reduced bladder spasms and urge urinary incontinence

Tolterodine

-Muscarinic antagonist -Reduced bladder spasms and urge urinary incontinence

Ipratropium

-Muscarinic antagonist -Used in COPD and asthma

Tiotropium

-Muscarinic antagonist -Used in COPD and asthma

Benztropine

-Muscarinic antagonist -Works in CNS -Used in Parkinson disease and acute dystonia

Atropine

-Muscarinic antagonist -Used in bradycardia and for ophthalmic applications -Also used as antidote for cholinesterase inhibitor poisoning -Actions include increase pupil dilation, cycloplegia, decreased airway secretions, decreased acid secretions, decreased gut motility, decreased bladder urgency in cystitis -Toxicity: increased body temp (due to decreased sweating), rapid pulse, dry mouth, dry and flushed skin, cycloplegia, constipation, disorientation; -Can cause acute angle-closure glaucoma in elderly (due to mydriasis), urinary retention in men with prostatic hyperplasia, and hyperthermia in infants -See also homatropine and tropicamide

Methacholine

-Muscarinic receptor (M3) agonist. -Used in bronchial challenge test to help diagnose asthma.

Methotrexate (MTX) toxicity

-Myelosuppression, which is reversible with leucovorin "rescue." -Hepatotoxicity. -Mucositis (e.g., mouth ulcers). -Pulmonary fibrosis.

Nonopiod analgesic: acetaminophen (APAP)

-NO anti-inflammatory action -NOT considered an NSAID -NO effect on clotting -no GI irritation Uses 2A'S o Analgesic o Antipyretic -safe Use for pg women

Nonselective β-blockers

-Nadolol, pindolol (partial agonist), propranolol, timolol -Mostly go from N to Z

Side effects of high potency typical antipsychotics

-Neurologic side effects (e.g. Huntington disease, delirium, EPS symptoms)

Nitrous Oxide abuse can lead to:

-Neurologic symptoms similar to Parkinson's disease and dementia -Numbness and tingling in the extremities

Treatment of Huntington disease

-Neurotransmitter changes in Huntington disease: decrease GABA, decrease ACh, increase dopamine. -Treatments: 1. tetrabenazine and reserpine—inhibit vesicular monoamine transporter (VMAT); limit dopamine vesicle packaging and release; 2. haloperidol—D2 receptor antagonist.

Phenoxybenzamine

-Nonselective α-blocker -Irreversible -Used preoperatively for pheochromocytoma to prevent catecholamine (hypertensive) crisis -Toxicity: orthostatic hypotension, reflex tachycardia

Fentanyl use

-One of the Most Used Synthetic Opioids -Relief of Moderate-Severe Pain -Anesthetic Adjuvant (Preoperative Analgesia) -Postoperative or Labor Analgesia (Epidural) -Chronic Pain (Patch) -Breakthrough in Cancer pain (Oral Lozenge)

Methadone use

-Opioid abuse -Increasingly Recognized as a useful analgesic (can be used in opioid rotation)

Local effect

-Oral drugs can cause GI symptoms nausea or dyspepsia -Topical drugs can irritate the site -Injectable drugs can irritate injection site

Metformin clinical use

-Oral. First line therapy in type 2 DM, causes modest weight loss. -Can be used in patients without islet function

Dicloxacillin, nafcillin, oxacillin

-Penicillinase-resistant penicillins Same mechanism as penicillin (inhibits peptidoglycan cross-linking) -Narrow spectrum; -Penicillinase resistant because bulky R group blocks access of β-lactamase to β-lactam ring. -Use with S. aureus (except MRSA; resistant because of altered penicillin-binding protein target site). -Toxicity: Hypersensitivity reactions, interstitial nephritis.

Proton pump inhibitors (PPIS)

-Potent inhibitors of gastric acid secretion -Most powerful med for treating GERD Examples: (-prazole) (P=P) omeprazole (Prilosec) lansoprazole (Prevacid) esomeprazole (Nexium)

Current guidelines for antibiotic premedication:

-Prosthetic cardiac valves -Prosthetic material used for cardiac valve repair -History of infective endocarditis -Cardiac transplant with valve regurgitation due to abnormal valve -Congenital Heart Disorders (only very specific situations)

Penicillin G, V

-Prototype β-lactam antibiotics -G=IV or IM; V=Oral administration -Bind penicillin-binding proteins (transpeptidases). -Block transpeptidase cross-linking of peptidoglycan in cell wall. Activate autolytic enzymes. -Mostly used for gram-positive organisms (S. pneumoniae, S. pyogenes, Actinomyces). Also used for gram-negative cocci (mainly N. meningitidis) and spirochetes (namely T. pallidum). Bactericidal for gram-positive cocci, gram-positive rods, gram-negative cocci, and spirochetes. -Penicillinase in bacteria (a type of β-lactamase) cleaves β-lactam ring. -Toxicity: hypersensitivity reactions, hemolytic anemia

Endothelin receptor antagonists

-Pulmonary hypertension therapy -Include bosentan. -Competitively antagonize endothelin-1 receptors decreasing pulmonary vascular resistance. -Hepatotoxic (monitor LFTs)

Ethambutol

-Reduces carbohydrate polymerization of mycobacterium cell wall by blocking arabinosyltransferase. -Use: Mycobacterium tuberculosis. -Toxicity: optic neuropathy (red-green color blindness).

Other Common Sympathomimetics / Adrenergic Agents

-Ritalin / Adderall - used in treatment of ADHD -cocaine :The only local anesthetic that is a vasoconstrictor

Prazosin

-Selective α1-blocker -Uses: urinary symptoms of BPH; PSTD -Hypertension -Toxicity: 1st-dose orthostatic hypotension, dizziness, headache

Mirtazapine

-Selective α2-blocker -Used in depression -Toxicity: sedation, increased serum cholesterol, increased appetite

Class IC sodium channel blockers mechanism

-Significantly prolongs ERP in AV node and accessory bypass tracts. No effect on ERP in Purkinje and ventricular tissue. -Minimal effect on AP duration

Antibiotics to avoid during pregnancy

-Sulfonamides -Aminoglycosides -Fluoroquinolones -Clarithromycin -Tetracyclines -Ribavirin (antiviral) -Griseofulvin (antifungal) -Chloramphenicol (SAFe Children Take Really Good Care)

Lithium toxicity

-Tremor, hypothyroidism, polyuria (causes nephrogenic diabetes insipidus), teratogenesis. -Causes Ebstein anomaly in newborn if taken by pregnant mother. -Narrow therapeutic window requires close monitoring of serum levels. -Almost exclusively excreted by kidneys; most is reabsorbed at PCT with Na+. Thiazide use is implicated in lithium toxicity in bipolar patients.

Fibrates mechanism of action

-Upregulate LPL causing increase in TG clearance. -Activates PPAR-α to induce HDL synthesis

Bupropion (use, mechanism, toxicity)

-Use: Depression; also used for smoking cessation and ADHD. -Mechanism: increase norepinephrine and dopamine via unknown mechanism. -Toxicity: stimulant effects (tachycardia, insomnia), headache, seizures in anorexic/bulimic patients. No sexual side effects.

Mirtazapine (use, mechanism, toxicity)

-Use: depression -Mechanism: α2-antagonist (increase release of norepinephrine and 5-HT) and potent 5-HT2 and 5-HT3 receptor antagonist. -Toxicity: sedation (which may be desirable in depressed patients with insomnia), increase appetite, weight gain (which may be desirable in elderly or anorexic patients), dry mouth.

β1-selective antagonist

-acebutolol (partial agonist), atenolol, betaxolol, esmolol, metoprolol -Mostly go from A to M

Oral medications used to treat Type 2 Diabetes

-glyburide Classified as a sulfonylurea -metformin (Glucophage) Classified as a biguanide

ANTIHYPERTENSIVE DRUGS Diuretics

-ide -Enhance the excretion of sodium and water - decreasing blood pressure -Effectiveness reduced by NSAIDS Adverse Reactions o xerostomia and orthostatic hypotension Two major types: Thiazide diuretics: hydrochlorothiazide (HCTZ) Loop diuretics: furosemide (Lasix)

Anesthetics available WITHOUT a vasoconstrictor (Plain solution) (no sodium bisulfate)

-lidocaine - not available in dentistry in the US -mepivacaine -prilocaine

Common examples of ACE Inhibitors (-pril)

-lisinopril (Prinivil, Zestril) -enalapril (Vasotec)

Sirolimus (Rapamycin) MoA

-mTOR inhibitor; binds FKBP. -Blocks T-cell activation and B-cell differentiation by preventing response to IL-2.

Pathophysiology of ADHD

-may be caused by a dysregulation of seratonin, norepinephrine, and dopamine. -occurs primarily <age7. -3 to 7 times more common in boys

BOARD ALERT Common medications that cause gingival overgrowth (gingival hyperplasia):

-phenytoin (Dilantin) -Calcium Channel Blockers (Always Very Nice Day) - antihypertensives -amlodipine (Norvasc) -verapamil (Calan) -nifedipine (Procardia) -diltiazem (Cardizem) -cyclosporine (immunosuppressant for organ transplant patients)

Cholinergic Drugs Uses:

-pilocarpine (Salagen) - treat xerostomia -nicotine found in tobacco products, nicotine patches, smoking cessation chewing gum -treatment of glaucoma and Alzheimer's *increase salivation

ANTIHYPERTENSIVE Calcium Channel Blockers (CCBS)

-some end in -dipine -Effectiveness NOT reduced by NSAIDS (one of the few!!) Adverse Reactions -Orthostatic hypotension -Xerostomia -Gingival enlargement hyperplasia

Tricyclic Antidepressants (TCAS)

-suicidal tendencies -BOARD ALERT - Do not use epinephrine in patients taking TCAS- may lead to hypertensive crisis- relative contraindication - if must use, limit to cardiac dose -amitriptyline (Elavil)

Common examples of ARBSs (-sartan)

-valsartan (Diovan) -losartan (Cozaar) (Sartan says ARB!!)

α-methyldopa

-α2-agonist -Used for hypertension in pregnancy -Toxicity: Direct Coombs ⊕ hemolysis, SLE-like syndrome

Clonidine

-α2-agonist -Uses: hypertensive urgency (limited situations); does not decrease renal blood flow; ADHD, Tourette syndrome -Toxicity: CNS depression, bradycardia, hypotension, respiratory depression, miosis

Salmeterol, formoterol

-β2-agonists -Long-acting agents for prophylaxis. -Adverse effects are tremor and arrhythmia.

Albuterol

-β2-agonists -Relaxes bronchial smooth muscle (β2). -Used during acute exacerbation

B. Decrease in urinary frequency

3. The nurse monitors a patient taking tolterodine (Detrol) for which therapeutic effect? A. Decrease in gastrointestinal motility B. Decrease in urinary frequency C. Increase in heart rate D. Increase in blood pressure

b. maintenance of muscle strenght

A 62-year-old client is receiving treatment for myasthenia gravis with an acetylcholinesterase inhibitor. The nurse is assessing the client. What clinical manifestations would be noted to determine if the medication is working? a. increased salivation b. maintenance of muscle strenght c. miosis d. experiencing the side effect of GI upset

c. He will need to take measures to reduce the occurrence of heat stroke during his activities.

A 72-year-old man has a new prescription for an anticholinergic drug. He is an active man and enjoys outdoor activities, such as golfing and doing his own yard work. What will the nurse emphasize to him during the teaching session about his drug therapy? a. Increased salivation may occur during exercise and outside activities. b. Fluid volume deficits may occur as a result of an increased incidence of diarrhea. c. He will need to take measures to reduce the occurrence of heat stroke during his activities. d. Drowsiness may interfere with his outdoor activities.

d. Significant decrease in blood pressure

A 73-year-old male patient is in the clinic for a yearly physical and is asking for a prescription for sildenafil (Viagra). He has listed on his health history that he is taking a nitrate for angina. The nurse is aware that which problem may occur if sildenafil is taken with a nitrate? a. Reduced effectiveness of the sildenafil b. Increased risk of bleeding c. Significant increase in pulse rate d. Significant decrease in blood pressure

b. He needs to take extra precautions when standing up because of possible orthostatic hypotension and resulting injury from falls.

A 79-year-old patient is taking a diuretic for treatment of hypertension. This patient is very independent and wants to continue to live at home. The nurse will know that which teaching point is important for this patient? a. He should take the diuretic with his evening meal. b. He needs to take extra precautions when standing up because of possible orthostatic hypotension and resulting injury from falls. c. He should skip the diuretic dose if he plans to leave the house. d. If he feels dizzy while on this medication, he needs to stop taking it and take potassium supplements instead.

c. "Take the prescribed dose as soon as you remember it, and if it is very close to the time for the next dose, delay that next dose."

A client with a urinary tract infection is to take nitrofurantoin (Macrodantin) four times a day. The client asks the nurse, "What should I do if I forget a dose?" What should the nurse tell the client? a. "Double the amount prescribed with your next dose." b. "Take a lot of water with a double amount of your prescribed dose." c. "Take the prescribed dose as soon as you remember it, and if it is very close to the time for the next dose, delay that next dose." d. "You can wait and take the next dose when it is due."

c. Oral

A drug given by which route is altered by the first-pass effect? a. Sublingual b. Subcutaneous c. Oral d. Intravenous

b patient's level of sedation is inadequate.

A mechanically ventilated patient receiving a neuromuscular-blocking drug has tearing in the eyes and increased heart rate and blood pressure. The nurse interprets that the a patient is having an adverse reaction to the medication. b patient's level of sedation is inadequate. c patient's dose of the neuromuscular-blocking drug is insufficient. d patient's response to the drug is appropriate.

a. Hyperventilation and drowsiness

A mother brings her toddler into the emergency department and tells the nurse that she thinks the toddler has eaten an entire bottle of chewable aspirin tablets. The nurse will assess for which most common signs of salicylate intoxication in children? a. Hyperventilation and drowsiness b. Acute gastrointestinal bleeding c. Photosensitivity and nervousness d. Tinnitus and hearing loss

d "Drugs can cross from mother to infant in breast milk, so it will depend on the drug you are taking."

A mother of a 1-month-old infant calls the clinic and asks the nurse if the medication she is taking can be passed to her infant during breastfeeding. What is the appropriate response for this patient? a "You should not take any medication while breastfeeding." b "Only certain medications pass to infants while breastfeeding." c "I will leave the doctor a message to return your call." d "Drugs can cross from mother to infant in breast milk, so it will depend on the drug you are taking."

B. Debrox

A mother of a young child asks what she can use to help keep her child's ears clean from wax. What over-the-counter medication will the nurse suggest? A. Cortic B. Debrox C. Floxin D. Ciprodex

Drugs can not impart what to an organism?

A new function. They either produce the same action as an endogenous agent, or lock the action of an endogenous agent.

a, b, c

A nurse is admitting a patient with idiopathic thrombocytopenic purpura to the unit. In completing the admission assessment, the nurse must be alert for what medications that alter platelet function? (Mark all that apply.) a. Aspirin-based drugs b. Sulfa-containing medications c. NSAIDs d. Antihypertensives e. Penicillins

b. The client's subcutaneous site for redness following injection.

A nurse is caring for a client who is beginning a new prescription for etanercept (Enbrel) for rheumatoid arthritis. Based on the route of administration of etanercept, which of the following should the nurse plan to monitor? a. The client's skin for irritation following removal of transdermal patch. b. The client's subcutaneous site for redness following injection. c. The client's vein for thrombophlebitis during IV administration. d. The client's oral mucosa for ulceration after oral administration.

d. "I've been buying Tagamet to help with the indigestion I've had."

A nurse is caring for a client who is taking naproxen (Naprosyn) following an exacerbation of rheumatoid arthritis. Which of the following comments by the client requires further discussion by the nurse? a. "I signed up for a swimming class." b. "The Naprosyn goes down easier when I crush it and put it in applesauce." c. "I've lost 2 pounds since my appointment 2 weeks ago." d. "I've been buying Tagamet to help with the indigestion I've had."

a, b, e

A nurse is providing education to a group of athletes on the topic of anabolic steroids. What side effects would the nurse include in the discussion with the group of athletes? (Select all that apply.) A. Sterility B. Liver cancer C. Impotence D. Stroke E. Cardiovascular disease

b Ataxia

A nurse would monitor older adults who are prescribed a benzodiazepine for treatment of insomnia for which potential side effect? a Hallucinations b Ataxia c Alertness d Dyspnea

b. Blood pressure of 88/62 mm Hg

A patient arrives in the emergency department with severe chest pain. The patient reports that the pain has been occurring off and on for a week now. Which assessment finding would indicate the need for cautious use of nitrates and nitrites? a. Apical pulse rate of 110 beats/min b. Blood pressure of 88/62 mm Hg c. History of a myocardial infarction 2 years ago d. History of renal disease

D. "You are on a diuretic that is potassium-sparing, so there is no need for extra potassium."

A patient asks about taking potassium supplements while taking spironolactone (Aldactone). What is the nurse's best response? A. "You are correct about your concern. I will make sure that you get some right away." B. "I will call your doctor and let him know of your concern." C. "Potassium supplements are usually not necessary with this type of diuretic." D. "You are on a diuretic that is potassium-sparing, so there is no need for extra potassium."

a, b, d

A patient asks the nurse about a new drug advertised on television. The patient wants to know if Ambien would be better for her to use than her current medication, Restoril, for periodic insomnia. The nurse's response is based on knowledge that zolpidem (Ambien) (Select all that apply.) a. is less likely to cause grogginess in the morning. b. is a pregnancy category C medication. c. is contraindicated with asthma. d. should be limited to 7 to 10 days of treatment.

B. sustained use of nasal decongestants over several days may result in rebound congestion.

A patient complains of worsening nasal congestion despite the use of oxymetazoline (Afrin) nasal spray every 2 to 4 hours for the past 5 days. The nurse's response is based on knowledge that A. oxymetazoline is not an effective nasal decongestant. B. sustained use of nasal decongestants over several days may result in rebound congestion. C. oxymetazoline should be administered in an hourly regimen for severe congestion. D. the patient is probably displaying an idiosyncratic reaction to oxymetazoline.

D There is a risk of toxicity when this medication is taken with alcohol.

A patient diagnosed with depression is started on a TCA after failure to improve symptoms on an SSRI. The nurse should include which teaching point when educating the patient about the new medication? A There are no contraindications to this medication. B The medication is safe; it has been used longer than many others. C This class of medications has no other use and is only for depression. D There is a risk of toxicity when this medication is taken with alcohol.

c "Nausea and vomiting occur much less frequently than in the past related to the use of a balanced approach to anesthesia."

A patient expresses fear of nausea and vomiting postoperatively from the anesthesia. Which is the nurse's best response? a "I understand your concern; that is one of the pitfalls of surgery." b "Don't worry, because you'll be heavily sedated if that occurs." c "Nausea and vomiting occur much less frequently than in the past related to the use of a balanced approach to anesthesia." d "You will need to speak to both the surgeon and the anesthesiologist in order to identify possible allergies to drugs that may be causing this side effect."

c Acute hepatic necrosis

A patient has been admitted after overdosing on acetaminophen (Tylenol), with a total ingested dose of 14 g over a period of 1 hour. The nurse plans to monitor this patient for development of which of the following signs and symptoms related to the overdose? a Renal failure b Kidney stones c Acute hepatic necrosis d Metabolic alkalosis

a. "Limit use of this spray to 3 to 5 days."

A patient has been advised to add a nasal spray (an adrenergic decongestant) to treat a cold. The nurse will include which instruction? a. "Limit use of this spray to 3 to 5 days." b. "Avoid use of this spray if a fever develops." c. "You won't see effects for at least 1 week." d. "Continue the spray until nasal stuffiness has resolved."

b. If the chest pain is not relieved after one tablet, call 911 immediately.

A patient has been diagnosed with angina and will be given a prescription for sublingual nitroglycerin tablets. When teaching the patient how to use sublingual nitroglycerin, the nurse will include which instruction? a. Take up to five doses at 15-minute intervals for an angina attack. b. If the chest pain is not relieved after one tablet, call 911 immediately. c. Wait 1 minute between doses of sublingual tablets, up to three doses. d. If the tablet does not dissolve quickly, chew the tablet for maximal effect.

c. To decrease insulin resistance

A patient has been diagnosed with metabolic syndrome and is started on the biguanide metformin (Glucophage). The nurse knows that the purpose of the metformin, in this situation, is which of these? a. To increase the pancreatic secretion of insulin b. To decrease the pancreatic secretion of insulin c. To decrease insulin resistance d. To increase blood glucose levels

c. Carbidopa prevents the breakdown of levodopa in the periphery.

A patient has been given a prescription for levodopa-carbidopa (Sinemet) for her newly diagnosed Parkinson's disease. She asks the nurse, "Why are there two drugs in this pill?" The nurse's best response reflects which fact? a. Carbidopa is the biologic precursor of dopamine and can penetrate into the CNS. b. There are concerns about drug-food interactions with levodopa therapy that do not exist with the combination therapy. c. Carbidopa prevents the breakdown of levodopa in the periphery. d. Carbidopa allows for larger doses of levodopa to be given.

b. Take the medication with 6 to 8 ounces of water and food.

A patient has been instructed to take one enteric-coated low-dose aspirin a day as part of therapy to prevent strokes. The nurse will provide which instruction when providing patient teaching about this medication? a. Low-dose aspirin therapy rarely causes problems with bleeding. b. Take the medication with 6 to 8 ounces of water and food. c. Coated tablets may be crushed if necessary for easier swallowing. d. Aspirin needs to be taken on an empty stomach to ensure maximal absorption.

A. AIDS

A patient has been prescribed monoclonal antibody therapy for the treatment of rheumatoid arthritis. What known disorder or disease is a contraindication and would alert the nurse to question the order for this class of drugs? A. AIDS B. Hypertensive crisis C. Chronic obstructive pulmonary disease D. Nephrotic syndrome

a. Heparin is used to start anticoagulation so as to allow time for the blood levels of warfarin to reach adequate levels.

A patient has been prescribed warfarin (Coumadin) in addition to a heparin infusion. The patient asks the nurse why he has to be on two medications. The nurse's response is based on which rationale? a. Heparin is used to start anticoagulation so as to allow time for the blood levels of warfarin to reach adequate levels. b. The oral and injection forms work synergistically. c. The combination of heparin and an oral anticoagulant results in fewer adverse effects than heparin used alone. d. Oral anticoagulants are used to reach an adequate level of anticoagulation when heparin alone is unable to do so.

b. "This drug is an antidote to digoxin and will help to lower the blood levels."

A patient has been taking digoxin at home but took an accidental overdose and has developed toxicity. The patient has been admitted to the telemetry unit, where the physician has ordered digoxin immune Fab (Digifab). The patient asks the nurse why the medication is ordered. What is the nurse's best response? a. "It helps to convert the irregular heart rhythm to a more normal rhythm." b. "This drug is an antidote to digoxin and will help to lower the blood levels." c. "It will increase your heart rate." d. "This drug helps to lower your potassium levels."

a. This drug causes deprivation of REM sleep and may cause the inability to deal with normal stress.

A patient has been taking phenobarbital for 2 weeks as part of his therapy for epilepsy. He tells the nurse that he feels tense and that "the least little thing" bothers him now. Which is the correct explanation for this problem? a. This drug causes deprivation of REM sleep and may cause the inability to deal with normal stress. b. This drug causes the rapid eye movement (REM) sleep period to increase, resulting in nightmares and restlessness. c. The drug must be stopped immediately because of possible adverse effects. d. These are adverse effects that usually subside after a few weeks.

a. Immunosuppression may make it more difficult to eliminate the eye infection.

A patient has been taking the corticosteroid dexamethasone (Decadron) but has developed bacterial conjunctivitis and has a prescription for gentamicin (Garamycin) ointment. The nurse notes that which interaction is possible if the two drugs are used together? a. Immunosuppression may make it more difficult to eliminate the eye infection. b. The infection may become systemic. c. The corticosteroid may cause overgrowth of nonsusceptible organisms. d. The gentamicin effects may become more potent.

a. The incidence of dry mouth is much lower with darifenacin.

A patient has been taking tolterodine (Detrol), but today her prescriber changed her to a newer drug, darifenacin (Enablex). What advantage does darifenacin have over the tolterodine? a. The incidence of dry mouth is much lower with darifenacin. b. It helps reduce urinary retention. c. It can be used in patients with narrow-angle glaucoma. d. The newer cholinergic-blocker drugs are more effective.

d. Iron

A patient has come to the OB/GYN clinic with complaints of a heavy menstrual flow. The nurse knows that red blood cell production will be increased in the patient's body. Because of this, the nurse is aware that the patient may need to increase her daily intake of what? a. Vitamin D b. Vitamin C c. Magnesium d. Iron

d. "Take the bronchodilator inhaler first."

A patient has prescriptions for two inhalers. One inhaler is a bronchodilator, and the other is a corticosteroid. Which instruction regarding these inhalers will the nurse give to the patient? a. "Take the corticosteroid inhaler first." b. "It does not matter which inhaler you use first." c. "Take these two drugs at least 2 hours apart." d. "Take the bronchodilator inhaler first."

c. Osmotic diuretics

A patient in the neurologic intensive care unit is being treated for cerebral edema. Which class of diuretic is used to reduce intracranial pressure? a. Vasodilators b. Loop diuretics c. Osmotic diuretics d. Thiazide diuretics

c, e, f

A patient is about to receive a dose of octreotide (Sandostatin). The nurse will assess for which contraindications or cautions? (Select all that apply.) a. Diarrhea b. Carcinoid crisis c. Gallbladder disease d. Esophageal varices e. Chronic renal failure f. Type 1 diabetes mellitus

b activated charcoal

A patient is admitted to the emergency department after taking an overdose of a barbiturate 15 minutes prior to arrival. The nurse can anticipate that which drug will be prescribed? a naloxone (Narcan) b activated charcoal c flumazenil (Romazicon) d ipecac syrup

d flumazenil (Romazicon)

A patient is admitted to the emergency department with a severe overdose of a benzodiazepine. The nurse immediately prepares to administer which antidote from the emergency drug cart? a naloxone (Narcan) b naltrexone (ReVia) c nalmefene (Revex) d flumazenil (Romazicon)

C. Furosemide is effective in treating patients with renal insufficiency.

A patient is admitted to the hospital with pneumonia and has a history of chronic renal insufficiency. Why does the physician order furosemide (Lasix) 40 mg twice a day? A. Furosemide will not cause potassium loss. B. Furosemide is effective in treating patients with pulmonary congestion. C. Furosemide is effective in treating patients with renal insufficiency. D. Furosemide will increase PO2 levels.

a, b, d

A patient is being seen for acne issues. The nurse is discussing use of various topical drugs. Which drugs are used in the treatment of acne?(Select all that apply.) A. benzoyl peroxide B. clindamycin C. isotretinoin D. vitamin A acid E. norethindrone and ethinyl estradiol

B "I will not get as dizzy when I change positions after I switch medications."

A patient is being switched from amitriptyline (Elavil) to citalopram (Celexa). Which statement made by the patient reflects understanding of patient education? A "I can just stop taking my Elavil and start taking the Celexa as ordered." B "I will not get as dizzy when I change positions after I switch medications." C "The doctor is switching me to this medication because it is less expensive but just as effective." D "I will need to limit my intake of cheese when taking Celexa to prevent a rise in my blood pressure."

D. carbamide peroxide

A patient is complaining of excessive earwax, leading to diminished hearing ability. The nurse will expect to teach the patient regarding administration of which medication to correct this problem? A. hydrocortisone B. pramoxine C. acetic acid D. carbamide peroxide

a. A short-acting beta2 agonist such as albuterol (Proventil)

A patient is in an urgent-care center with an acute asthma attack. The nurse expects that which medication will be used for initial treatment? a. A short-acting beta2 agonist such as albuterol (Proventil) b. A corticosteroid such as fluticasone (Flovent) c. A long-acting beta2 agonist such as salmeterol (Serevent) d. An anticholinergic such as ipratropium (Atrovent)

d. diltiazem (Cardizem)

A patient is in the emergency department with a new onset of rapid-rate atrial fibrillation, and the nurse is preparing a continuous infusion. Which drug is most appropriate for this dysrhythmia? a. atenolol (Tenormin) b. lidocaine c. adenosine (Adenocard) d. diltiazem (Cardizem)

b, d, f

A patient is on a chemotherapy regimen in an outpatient clinic and is receiving a chemotherapy drug that is known to be highly emetogenic. The nurse will implement which interventions regarding the pharmacologic management of nausea and vomiting? (Select all that apply.) a. Observing carefully for the adverse effects of restlessness and anxiety b. Instructing the patient that the antinausea drugs may cause extreme drowsiness c. Giving antinausea drugs at the beginning of the chemotherapy infusion d. Instructing the patient to rise slowly from a sitting or lying position because of possible orthostatic hypotension e. For best therapeutic effects, medicating for nausea once the symptoms begin f. Administering antinausea drugs 30 to 60 minutes before chemotherapy is started

C. Renal insufficiency

A patient is prescribed an over-the-counter laxative magnesium hydroxide (milk of magnesia). Which disorder in the patient's history would alert the nurse that there is a need to discuss this medication with the patient and the provider? A. Hypertension B. Liver dysfunction C. Renal insufficiency D. Chronic constipation

c. Prokinetic drugs, such as metoclopramide (Reglan)

A patient is receiving a tube feeding through a gastrostomy. The nurse expects that which type of drug will be used to promote gastric emptying for this patient? a. Neuroleptic drugs, such as chlorpromazine (Thorazine) b. Serotonin blockers, such as ondansetron (Zofran) c. Prokinetic drugs, such as metoclopramide (Reglan) d. Anticholinergic drugs, such as scopolamine (Scopace)

a. Constipation

A patient is receiving an aluminum-containing antacid. The nurse will inform the patient to watch for which possible adverse effect? a. Constipation b. Abdominal cramping c. Nausea d. Diarrhea

b. The drug dose should be weaned down prior to discontinuing.

A patient is receiving dopamine, a vasoactive drug used for shock, to increase her stroke volume. What should the nurse be aware of when monitoring a vasoactive drug? a. The drug should be discontinued immediately after blood pressure increases. b. The drug dose should be weaned down prior to discontinuing. c. The drug may cause respiratory alkalosis. d. The drug reduces oxygen demands of the heart.

c. "Take the Restasis first, and then wait 15 minutes before taking the artificial tears."

A patient is receiving ocular cyclosporine (Restasis) and also has an order for an artificial tears product. The nurse includes which instructions in the teaching plan for these medications? a. "First take the artificial tears, and then take the Restasis after 5 minutes." b. "These two eye drugs cannot be given together. Let's check with your prescriber." c. "Take the Restasis first, and then wait 15 minutes before taking the artificial tears." d. "You may take these two drugs together at the same time."

a. Thiazide diuretics

A patient is started on a diuretic for antihypertensive therapy. The nurse expects that a drug in which class is likely to be used initially? a. Thiazide diuretics b. Potassium-sparing diuretics c. Osmotic diuretics d. Loop diuretics

B. "It usually takes 4 to 5 days to achieve a full therapeutic effect for warfarin, so the heparin is continued to help prevent blood clots until the warfarin is working up to speed."

A patient is started on oral anticoagulant therapy while still receiving intravenous heparin. The patient is concerned about risk for bleeding. What is the nurse's best response? A. "Your concern is valid in that you are at an increased risk for bleeding, so I will call the doctor to discontinue the heparin." B. "It usually takes 4 to 5 days to achieve a full therapeutic effect for warfarin, so the heparin is continued to help prevent blood clots until the warfarin is working up to speed." C. "Because of your valve replacement, it is especially important for you to be fully anticoagulated, and the heparin and warfarin together are more effective than one alone." D. "Because you are now up and walking, you have a higher risk of blood clots and therefore need to be on both medications."

a, d, e

A patient is taking a sulfonylurea medication for new-onset type 2 diabetes mellitus. When reviewing potential adverse effects during patient teaching, the nurse will include information about which of these effects? (Select all that apply.) a. Weight gain b. Peripheral edema c. Diarrhea d. Nausea e. Hypoglycemia

A. Evaluate the patient for other symptoms of digoxin toxicity.

A patient is taking digoxin (Lanoxin) 0.25 mg and furosemide (Lasix) 40 mg. When the nurse enters the room, the patient states, "I think I need to take a nap. Everything is starting to look a little yellow." Which action will the nurse take? A. Evaluate the patient for other symptoms of digoxin toxicity. B. Withhold the furosemide. C. Administer the medication as ordered. D. Document the findings and reassess in 1 hour.

b. Force fluids to help loosen and liquefy secretions.

A patient is taking guaifenesin (Humibid) as part of treatment for a sinus infection. Which instruction will the nurse include during patient teaching? a. Report clear-colored sputum to the prescriber. b. Force fluids to help loosen and liquefy secretions. c. Avoid driving a car or operating heavy machinery because of the sedating effects. d. Report symptoms that last longer than 2 days.

d. Increased ease of breathing

A patient is taking intravenous aminophylline for a severe exacerbation of chronic obstructive pulmonary disease. The nurse will assess for which therapeutic response? a. Increased sputum production b. Increased respiratory rate c. Increased heart rate d. Increased ease of breathing

d. "The patch allows for better absorption of the medication."

A patient is to receive testosterone therapy via a transdermal patch. He asks the nurse, "Why am I getting a patch? Can't I just take a pill?" Which response by the nurse is correct? a. "You will only have to change the patch weekly." b. "If you don't take the patch, you will have to have injections instead." c. "The patch reduces the incidence of side effects." d. "The patch allows for better absorption of the medication."

a. Impaired gas exchange related to central nervous system depression produced by general anesthesia

A patient is undergoing abdominal surgery and has been anesthetized for 3 hours. Which nursing diagnosis would be appropriate for this patient? a. Impaired gas exchange related to central nervous system depression produced by general anesthesia b. Anxiety related to the use of an anesthetic c. Decreased cardiac output related to systemic effects of local anesthesia d. Risk for injury related to increased sensorium from general anesthesia

d morphine

A patient needs to switch analgesic drugs secondary to an adverse reaction to the present regimen. The patient is concerned that he will not receive an effective dose of a new drug to control pain. The nurse responds based on knowledge that potencies of analgesics are determined using an equianalgesic table comparing doses of these drugs with what prototype? a meperidine b fentanyl c codeine d morphine

d. "Please come into the clinic for an evaluation to make sure there are no complications."

A patient on diuretic therapy calls the clinic because he's had the flu, with "terrible vomiting and diarrhea," and he has not kept anything down for 2 days. He feels weak and extremely tired. Which statement by the nurse is correct? a. "You will need an increased dosage of the diuretic because of your illness. Let me speak to the physician." b. "It's important to try to stay on your prescribed medication. Try to take it with sips of water." c. "Stop taking the diuretic for a few days, and then restart it when you feel better." d. "Please come into the clinic for an evaluation to make sure there are no complications."

A. phenytoin

A patient prescribed a tube feeding has a history of a seizure disorder. The absorption of which antiepileptic medication is reduced when given with tube feedings? A. phenytoin B. levetiracetam C. pregabalin D. lamotrigine

B. Obstructive benign prostatic hyperplasia

A patient prescribed prazosin (Minipress) does not have a history of hypertension. The nurse would assess for what disorder for which this medication is also used? A. Pulmonary emboli B. Obstructive benign prostatic hyperplasia C. Seizure disorder D. Subarachnoid hemorrhage

C. Discontinue the infusion of packed cells.

A patient receiving a unit of red blood cells suddenly develops shortness of breath, chills, and fever. Following patient assessment, what is the nurse's initial action? A. Reassure the patient that this is an expected reaction. B. Notify the physician while a peer monitors the blood transfusion. C. Discontinue the infusion of packed cells. D. Decrease the infusion rate, and reassess the patient in 15 minutes.

B. Decrease the intravenous nitroglycerin by 10 mcg/min.

A patient receiving intravenous nitroglycerin at 20 mcg/min complains of dizziness. Nursing assessment reveals a blood pressure of 85/40 mm Hg, heart rate of 110 beats/min, and respiratory rate of 16 breaths/min. What is the nurse's best action? A. Assess the patient's lung sounds. B. Decrease the intravenous nitroglycerin by 10 mcg/min. C. Increase the intravenous nitroglycerin by 10 mcg/min. D. Recheck the patient's vital signs in 1 hour.

d increasing fluid and fiber in the diet.

A patient receiving narcotic analgesics for chronic pain can minimize the gastrointestinal (GI) side effects by a taking Lomotil with each dose. b eating foods high in lactobacilli. c taking the medication on an empty stomach. d increasing fluid and fiber in the diet.

B. "Propylthiouracil inhibits the formation of new thyroid hormone, thus gradually returning your metabolism to normal."

A patient receiving propylthiouracil (PTU) asks the nurse how this medication will help relieve his symptoms. What is the nurse's best response? A. "Propylthiouracil inactivates any circulating thyroid hormone, thus decreasing signs and symptoms of hyperthyroidism." B. "Propylthiouracil inhibits the formation of new thyroid hormone, thus gradually returning your metabolism to normal." C. "Propylthiouracil helps your thyroid gland use iodine and synthesize hormones better." D. "Propylthiouracil stimulates the pituitary gland to secrete thyroid-stimulating hormone (TSH), which inhibits the production of hormones by the thyroid gland."

b. Take an aspirin tablet 30 minutes before taking the drug.

A patient reports having adverse effects with nicotinic acid (niacin). The nurse can suggest performing which action to minimize these undesirable effects? a. Take the drug with large amounts of fiber. b. Take an aspirin tablet 30 minutes before taking the drug. c. Take the drug on an empty stomach. d. Take the medication every other day until the effects subside.

A. "Use an alternative method of birth control this month during antibiotic use."

A patient taking oral contraceptives is being treated for a urinary tract infection with antibiotics. Which information should the nurse include as education related to the oral contraceptives? A. "Use an alternative method of birth control this month during antibiotic use." B. "The urinary tract infection can be passed to your partner, so he should be screened." C. "Be sure to complete the full course of antibiotics." D. "There is no interaction of oral contraceptives and antibiotics."

C the patient is suffering from rebound congestion related to excessive use of the Afrin nasal spray.

A patient using Afrin nasal spray complains of worsening cold symptoms and tells the nurse, "I don't understand why this is not working. I am using it almost every 3 hours!" The nurse's response is based on knowledge that A Afrin nasal spray is not an effective nasal decongestant. B the medication needs to be used every 30 minutes for maximum effectiveness. C the patient is suffering from rebound congestion related to excessive use of the Afrin nasal spray. D adrenergic decongestants should only be used prophylactically, not to treat acute congestion.

b. dronabinol (Marinol), a tetrahydrocannabinoid

A patient who has AIDS has lost weight and is easily fatigued because of his malnourished state. The nurse anticipates an order for which antinausea drug to stimulate his appetite? a. metoclopramide (Reglan), a prokinetic drug b. dronabinol (Marinol), a tetrahydrocannabinoid c. ondansetron (Zofran), a serotonin blocker d. aprepitant (Emend), a substance P/NK1 receptor antagonist

d. prevent thrombus formation.

A patient who has a history of coronary artery disease has been instructed to take one 81-mg aspirin tablet a day. The nurse is aware that the purpose of this dose of aspirin is to a. reduce anxiety. b. relieve pain. c. reduce inflammation. d. prevent thrombus formation.

b. He smokes two packs of cigarettes a day.

A patient who has been taking cimetidine (Tagamet) for hyperacidity calls the clinic to say that the medication has not been effective. The nurse reviews his history and notes that which factor may be influencing the effectiveness of this drug? a. He drinks a glass of water with each dose. b. He smokes two packs of cigarettes a day. c. He takes an antacid 3 hours after the cimetidine dose. d. He takes the cimetidine with meals.

c. diazepam (Valium)

A patient who has received some traumatic news is panicking and asks for some medication to help settle down. The nurse anticipates giving which drug that is most appropriate for this situation? a. zolpidem (Ambien) b. phenobarbital c. diazepam (Valium) d. cyclobenzaprine (Flexeril)

B. Take advantage of a teachable moment to inform the patient of potential drug interactions with anticoagulants.

A patient who is taking an anticoagulant requests an aspirin for headache relief. What is the nurse's best action? A. Administer 650 mg of acetylsalicylic acid (ASA), and reassess pain in 30 minutes. B. Take advantage of a teachable moment to inform the patient of potential drug interactions with anticoagulants. C. Explain to the patient that ASA is contraindicated, and administer ibuprofen as ordered. D. Explain that the headache is an expected side effect and will subside shortly.

c. Signs of cinchonism, such as tinnitus, loss of hearing, or slight blurring of vision, may occur.

A patient will be discharged on quinidine sulfate (Quinidex) extended-release tablets for the treatment of ventricular ectopy. The nurse will include which information in the teaching plan? a. If any tablet or capsule is visible in the stool, contact the prescriber immediately. b. It is important to use sunscreen products when outside because of increased photosensitivity. c. Signs of cinchonism, such as tinnitus, loss of hearing, or slight blurring of vision, may occur. d. The medication should be stopped once the cardiac symptoms subside.

a. How to prevent constipation

A patient will be discharged with a 1-week supply of an opioid analgesic for pain management after abdominal surgery. The nurse will include which information in the teaching plan? a. How to prevent constipation b. How to prevent dehydration due to diarrhea c. Importance of taking the drug only when the pain becomes severe d. Importance of taking the drug on an empty stomach

c. Oral hypoglycemic drugs

A patient will be taking bismuth subsalicylate (Pepto-Bismol) to control diarrhea. When reviewing the patient's other ordered medications, the nurse recognizes that which medication or medication class will interact significantly with the Pepto-Bismol? a. Antidepressants b. acetaminophen (Tylenol) c. Oral hypoglycemic drugs d. Antibiotics

A Use of benzodiazepines decrease the therapeutic effect of the levadopa and may result in an increase in the symptoms of Parkinson's disease.

A patient with Parkinson's disease is discussing a recent bout of insomnia with the nurse. The patient asks if he can take an old prescription he has to treat insomnia. What does the nurse know about the use of benzodiazepines in patients taking levodopa? A Use of benzodiazepines decrease the therapeutic effect of the levadopa and may result in an increase in the symptoms of Parkinson's disease. B Use of benzodiazepines increases the therapeutic effect of the levadopa and may result in a decrease in the symptoms of Parkinson's disease. C Use of benzodiazepines decreases the therapeutic effect of the levadopa and may result in a decrease in the symptoms of Parkinson's disease. D Use of benzodiazepines increase the therapeutic effect of the levadopa and may result in an increase in the symptoms of Parkinson's disease.

A. oprelvekin (Neumega)

A patient with low platelets is prescribed a drug to stimulate platelet production. Which drug would the nurse anticipate being prescribed? A. oprelvekin (Neumega) B. epoetin (Epogen) C. aldesleukin (Proleukin) D. interferon beta-1a (Avonex)

b. "I will change the patch every 3 days."

A patient with motion sickness is planning a cross-country car trip and has a new prescription for a scopolamine transdermal patch mc013-1.jpgThe nurse provides teaching for the use of this patch medication. The patient shows a correct understanding of the teaching with which statement? a. "I will change the patch every other day." b. "I will change the patch every 3 days." c. "I will remove the patch only if it stops working." d. "I will change the patch every day."

c. High-density lipoproteins (HDLs)

A patient with risk factors for coronary artery disease asks the nurse about the "good cholesterol" laboratory values. The nurse knows that "good cholesterol" refers to which lipids? a. Triglycerides b. Low-density lipoproteins (LDLs) c. High-density lipoproteins (HDLs) d. Very-low-density lipoproteins (VLDLs)

d. Captopril is not a prodrug and does not need to be metabolized by the liver before becoming active.

A patient with severe liver disease is receiving the angiotensin converting enzyme (ACE) inhibitor, captopril (Capoten). The nurse is aware that the advantage of this drug for this patient is which characteristic? a. Captopril is a prodrug and is metabolized by the liver before becoming active. b. Captopril has little effect on electrolyte levels. c. Captopril rarely causes first-dose hypotensive effects. d. Captopril is not a prodrug and does not need to be metabolized by the liver before becoming active.

C. hetastarch (Hespan)

A patient with severe trauma is admitted to the intensive care unit. The patient has received 5000 mL of normal saline, is exhibiting peripheral edema, and remains hypotensive. The nurse anticipates administering which substance to correct fluid balance? A. Ringer's lactate B. A 3% saline solution C. hetastarch (Hespan) D. D5W

c. "I am looking forward to a cure for my condition with this hormone replacement."

A pituitary drug is prescribed for a patient with a hormone deficiency, and the nurse provides instructions about the medication. Which statement by the patient indicates a need for further instruction? a. "I will have to stop drinking my nightly glass of wine." b. "I will call my doctor if I have a fever or sore throat." c. "I am looking forward to a cure for my condition with this hormone replacement." d. "I will not stop the drug unless my doctor tells me to stop it."

a. "This drug may cause increased redness of your skin."

A teenage boy is taking tretinoin (Retin-A) for acne. Which statement will the nurse include in the teaching plan? a. "This drug may cause increased redness of your skin." b. "Avoid foods that are heavy in salt and oils." c. "Being out in the sunlight will help your skin heal." d. "Try using an abrasive cleanser to remove old skin layers."

Dexmethylphenidate

ADD/ADHD - GI, anorexia, dizziness, mood swings, no abuse - may cause seizures, nervousness, insomnia, dizziness, HA, dyskinesia, tachycardia, anorexia - dry mouth, disguise, no seizure increase

Dextroamphetamine

ADD/ADHD - GI, anorexia, dizziness, mood swings, no abuse - may cause seizures, nervousness, insomnia, dizziness, HA, dyskinesia, tachycardia, anorexia - dry mouth, disguise, no seizure increase - less abuse potential but still schedule II CS

GP IIb/IIIa inhibitors (drugs, mechanism, use, toxicity)

Abciximab, eptifibatide, tirofiban -Mechanism: bind to the glycoprotein receptor IIb/IIIa on activated platelets, preventing aggregation. Abciximab is made from monoclonal antibody Fab fragments. -Use: unstable angina, percutaneous transluminal coronary angioplasty. -Toxicity: bleeding, thrombocytopenia.

Schedule V

Abuse Potential Least codeine containing cough syrups

Schedule III

Abuse Potential Moderate codeine mixtures (Tylenol 3)

Schedule II

Abuse Potential high • opioid narcotics (oxycodone, morphine, ADHD (Adderall, Ritalin, Concerta)

α-glucosidase inhibitors

Acarbose, miglitol

a, c, d

Acetazolamide (Diamox) is used to treat which disorders? (Select all that apply.) A. Edema associated with heart failure B. Metabolic acidosis C. High-altitude sickness D. Open-angle glaucoma

Low molecular weight heparins (e.g., enoxaparin, dalteparin) and fondaparinux

Act more on factor Xa, have better bioavailability, and 2-4 times longer half-life; can be administered subcutaneously and without laboratory monitoring. Not easily reversible

D. Serum glucose levels

After administering somatropin (Serostim), the nurse would assess for adverse effects by monitoring which parameters? A. Serum potassium levels B. Mental status C. Respiratory rate D. Serum glucose levels

b. "This is an expected adverse effect. Let me take your pulse."

After receiving a nebulizer treatment with a beta agonist, the patient complains of feeling slightly nervous and wonders if her asthma is getting worse. What is the nurse's best response? a. "We will hold the treatment for 24 hours." b. "This is an expected adverse effect. Let me take your pulse." c. "The next scheduled nebulizer treatment will be skipped." d. "I will notify the physician about this adverse effect."

c. "I will take my vitamins while I am on this drug."

After teaching the client with severe rheumatoid arthritis about prescribed methotrexate (Rheumatrex), which of the following statements indicates the need for further teaching? a. "I will continue taking my birth control pills." b. "I must not drink any alcohol while I am on this drug." c. "I will take my vitamins while I am on this drug." d. "I should brush my teeth after every meal."

Bisphosphonates (drugs, mechanism, use, toxicity)

Alendronate, other -dronates. -Mechanism: pyrophosphate analogs; bind hydroxyapatite in bone, inhibiting osteoclast activity. -Use: osteoporosis, hypercalcemia, Paget disease of bone. -Toxicity: corrosive esophagitis (patients are advised to take with water and remain upright for 30 minutes), osteonecrosis of jaw.

Doxazosin

Alpha blocker - dizziness - HA - NSAIDs reduce effectiveness, caution with position change

Tamsulosin

Alpha blocker - dizziness - HA - NSAIDs reduce effectiveness, caution with position change

Thrombolytics (drugs, mechanism, use, toxicity)

Alteplase (tPA), reteplase (rPA), streptokinase, tenecteplase (TNK-tPA) -Mechanism: Directly or indirectly aid conversion of plasminogen to plasmin, which cleaves thrombin and fibrin clots. Increase PT, increase PTT, no change in platelet count. -Use: Early MI, early ischemic stroke, direct thrombolysis of severe PE. -Toxicity: Bleeding. Contraindicated in patients with active bleeding, history of intracranial bleeding, recent surgery, known bleeding diatheses, or severe hypertension. Treat toxicity with aminocaproic acid, an inhibitor of fibrinolysis. Fresh frozen plasma and cryoprecipitate can also be used to correct factor deficiencies.

List the potassium channel blockers (class III antiarhythmics)

Amiodarone, ibutilide, dofetilide, sotalol

High risk for endocarditis and undergoing surgical or dental procedures

Amoxicillin

a. Nonopioid analgesic, such as indomethacin (Indocin)

An 18-year-old basketball player fell and twisted his ankle during a game. The nurse will expect to administer which type of analgesic? a. Nonopioid analgesic, such as indomethacin (Indocin) b. Opium alkaloid, such as morphine sulfate c. Opioid antagonist, such as naloxone HCL (Narcan) d. Synthetic opioid, such as meperidine (Demerol)

Eszopiclone (Lunesta)

Anti-anxiety - xerostomia, CNS depressants additive -head ache, unpleasant taste, drowsiness - Macrolides, azole antifungals and doxycycline increase Lunesta levels

Aspirin

Anti-platelet effect - GI disturbances, GI bleeding, tinnitus

Prasugrel

Anti-platelet effect - major bleed 2-5%, syncope, stroke risk

Amiodarone

Antiarrhythmic - oral ulcers, neuraligic pain, pulmonary toxicity - may increase lidocaine levels

Warfarin (Coumadin)

Anticoagulation effect - monitor with INR - II, VII, IX and Xa inhibitor (vitamin K dependent factors--vitamin K antagonist) - many drug and food interactions - antidote= Vitamin-K-1 and fresh frozen plasma - Bridged with enoxaparin - NO NSAID - GI bleeding, INR 1.5-3.5 times normal can be managed without dose change, hemorrhage, hematuria, abdominal pain, skin necrosis, gangrene

Levetiracetam

Anticonvulsant - drowsiness, dizziness - low stress environment (consider BZDP), seizure control history often

Pregabalin

Anticonvulsant - drowsiness, dry mouth, peripheral edema - low stress environment (consider BZDP), seizure control history often

Topiramate

Anticonvulsant - low stress environment (consider BZDP), seizure control history often - drowsiness, dizziness, fatigue

a, c, d

Antidopaminergic drugs are useful to treat not only nausea and vomiting but which other conditions? (Select all that apply.) A. Anxiety B. Seizures C. Intractable hiccups D. Schizophrenia E. Bone marrow suppression

Digoxin immune Fab clinical use

Antidote for digoxin toxicity

Azathioprine MoA

Antimetabolite precursor of 6-mercaptopurine. Inhibits lymphocyte proliferation by blocking nucleotide synthesis.

Abciximab clinical use

Antiplatelet agent for prevention of ischemic complications in patients undergoing percutaneous coronary intervention

Ziprasidone

Antipsychotic--second generation atypical - HA, drowsiness, dizziness, weight gain

Risperidone

Antipsychotic--second generation atypical - HA, insomnia, agitation, weight gain, EPS - most commonly used

Aripipazole (Abilify)

Antipsychotic--second generation atypical - headache, agitation, anxiety, insomnia, weight gain

Olanzepine

Antipsychotic--second generation atypical - weight gain, sedation, good for refractory - most commonly used

DOC for Tourette syndrome

Antipsychotics(e.g. fluphenazine, pimozide), tetrabenazine, clonidine

Drugs that cause Parkinson-like syndrome

Antipsychotics, Reserpine, Metoclopramide

Drugs that cause tardive dyskinesia

Antipsychotics, metoclopramide

Aspirin use

Antipyretic, analgesic, anti-inflammatory, antiplatelet (decreased aggregation).

Direct factor Xa inhibitors (drugs, mechanism, use, toxicity)

Apixaban, rivaroxaban -Mechanism: bind to and directly inhibit factor Xa. -Use: treatment and prophylaxis for DVT and PE (rivaroxaban); stroke prophylaxis in patients with atrial fibrillation. Oral agents do not usually require coagulation monitoring. -Toxicity: bleeding (no reversal agent available).

a, b, c, d

Assessment of a patient receiving a positive inotropic drug would include reviewing which values? (Select all that apply.) A. Lung sounds B. Daily weights C. Apical pulse D. Serum electrolytes E. Complete blood count

D. Deficient fluid volume related to side effects of nausea, vomiting, and diarrhea

Assessment of the patient taking colchicine should include consideration of which nursing diagnoses? A. Constipation related to side effect of medication B. Risk for infection related to medication-induced leukocytosis C. Risk for injury related to side effect of seizures D. Deficient fluid volume related to side effects of nausea, vomiting, and diarrhea

c. Give the medications at 0900, and then the antacid at 1000.

At 0900, the nurse is about to give morning medications, and the patient has asked for a dose of antacid for severe heartburn. Which schedule for the antacid and medications is correct? a. Give both the antacid and medications at 0900. b. Give the medications at 0900, and then the antacid at 0915. c. Give the medications at 0900, and then the antacid at 1000. d. Give the antacid at 0900, and then the medications at 0930.

C. Neisseria gonorrhoeae

At the time of birth, infants are often treated with erythromycin eye ointment as prophylactic treatment against what possible organism? A. Aspergillus fumigatus B. Candida albicans C. Neisseria gonorrhoeae D. Malassezia furfur

a, b, c

Before administering an antileukotriene medication, the nurse would assess the patient for allergies to which substance? (Select all that apply.) A. Lactose B. Povidone C. Cellulose D. Chlorhexedine

A. Glaucoma

Before administering the anticholinergic drug scopolamine, the nurse would be careful to assess the patient for a history of which condition? A. Glaucoma B. Hyperthyroidism C. Rheumatoid arthritis D. Gastroenteritis

c. Anticoagulants

Before beginning a patient's therapy with selective serotonin reuptake inhibitor (SSRI) antidepressants, the nurse will assess for concurrent use of which medications or medication class? a. Aspirin b. Diuretics c. Anticoagulants d. Nonsteroidal antiinflammatory drugs

Prophylaxis of strep pharyngitis in child with prior rheumatic fever

Benzanthine penicillin G or or penicillin V

D. Binding to diarrhea-causing bacteria for excretion

Bismuth subsalicylate (Pepto-Bismol), as an adsorbent, has which mechanism of action? A. Decreased gastrointestinal motility B. Decreased gastric secretions C. Increased fluid absorption D. Binding to diarrhea-causing bacteria for excretion

Warfarin toxicity

Bleeding, teratogenic, skin/tissue necrosis A , drug-drug interactions. Proteins C and S have shorter half-lives than clotting factors II, VI, IX, and X, resulting in early transient hypercoagulability with warfarin use. Skin/tissue necrosis believed to be due to small vessel microthromboses. -For reversal of warfarin, give vitamin K. -For rapid reversal, give fresh frozen plasma. -Heparin "bridging": heparin frequently used when starting warfarin. Heparin's activation of antithrombin enables anticoagulation during initial, transient hypercoagulable state caused by warfarin. Initial heparin therapy reduces risk of recurrent venous thromboembolism and skin/tissue necrosis.

Drugs that cause pulmonary fibrosis

Bleomycin, amiodarone, methotrexate, busulfan

Local anesthetics mechanism

Block Na+ channels by binding to specific receptors on inner portion of channel. Preferentially bind to activated Na+ channels, so most effective in rapidly firing neurons. 3° amine local anesthetics penetrate membrane in uncharged form, then bind to ion channels as charged form

B. Coronary vasodilation

Calcium channel blockers have which pharmacodynamic effect? A. Positive inotropic effect B. Coronary vasodilation C. Positive chronotropic effect D. Shortened refractory period

C. It is the pressure against which the heart must pump.

Calcium channel blockers reduce myocardial oxygen demand by reducing afterload. How would the nurse explain afterload to the patient? A. It is the blood volume within the heart. B. It is the pressure within the heart. C. It is the pressure against which the heart must pump. D. It is the contractility of the heart muscle.

Diuretics that decrease blood pH (acidemia)

Carbonic anhydrase inhibitors: decrease HCO3− reabsorption. K+ sparing: aldosterone blockade prevents K+ secretion and H+ secretion. Additionally, hyperkalemia leads to K+ entering all cells (via H+/K+ exchanger) in exchange for H+ exiting cells.

Nitrosoureas (drugs, mechanism, use, toxicity)

Carmustine, lomustine, semustine, streptozocin -Mechanism: require bioactivation. Cross blood-brain barrier into CNS. Cross-link DNA. -Use: brain tumors (including glioblastoma multiforme). -Toxicity: CNS toxicity (convulsions, dizziness, ataxia)

Sulfonylureas mechanism

Close K+ channel in β-cell membrane then cell depolarizes causing insulin release via increased Ca2+ influx.

Hydralazine toxicity

Compensatory tachycardia (contraindicated in angina/CAD), fluid retention, headache, angina. Lupus-like syndrome!!!!

NONOPIOID ANALGESICS Pain ibuprofen (IBU), naproxen

Considered (NSAID) **Most useful pain medication in dentistry 3A's (not antiplatelet)

What effect do β-blockers have on BP?

Decrease

What effect do β-blockers have on HR?

Decrease

What effect do β-blockers have on MVO2 (myocardial volume oxygen concentration)?

Decrease

What effect do β-blockers have on contractility?

Decrease

What effect does nitrates + β-blockers have on BP?

Decrease

Class IB sodium channel blockers mechanism

Decrease AP duration. Preferentially affect ischemic or depolarized Purkinje and ventricular tissue.

Goal of glaucoma drugs

Decrease IOP via reducing amount of aqueous humor (inhibit synthesis/secretion or increase drainage)

Iron toxicity antidote

Deferoxamine, deferasirox

Type IV:

Delayed hypersensitivity- contact dermatitis

Side Effects of corticosteroids:

Delayed wound healing Hyperglycemia

Benzodiazepines (drugs, mechanism, use, toxicity)

Diazepam, lorazepam, triazolam, temazepam, oxazepam, midazolam, chlordiazepoxide, alprazolam -Mechanism: Facilitate GABA-A action by increasinf the frequency of Cl− channel opening. Decrease REM sleep. Most have long half-lives and active metabolites (exceptions: Alprazolam, Triazolam, Oxazepam, and Midazolam are short acting higher addictive potential) -Use: anxiety, spasticity, status epilepticus (lorazepam and diazepam), detoxification (especially alcohol withdrawal-DTs), night terrors, sleepwalking, general anesthetic (amnesia, muscle relaxation), hypnotic (insomnia) -Toxicity: dependence, additive CNS depression effects with alcohol. Less risk of respiratory depression and coma than with barbiturates. Treat overdose with flumazenil (competitive antagonist at GABA benzodiazepine receptor)

c. be eliminated by the body.

Drug half-life is defined as the amount of time required for 50% of a drug to a. be absorbed by the body. b. exert a response. c. be eliminated by the body. d. reach a therapeutic level.

Schedule V

Drugs with the lowest potential for abuse are found in which schedule?

b. Infiltration of the puncture wound with lidocaine

During a fishing trip, a patient pierced his finger with a large fishhook. He is now in the emergency department to have it removed. The nurse anticipates that which type of anesthesia will be used for this procedure? a. Topical benzocaine spray on the area b. Infiltration of the puncture wound with lidocaine c. Topical prilocaine (EMLA) cream around the site d. No anesthesia

a. "The provider is making sure the treatment is effective over the long-term."

During a follow-up visit, the health care provider examines the fundus of the patient's eye. Afterward, the patient asks the nurse, "Why is he looking at my eyes when I have high blood pressure? It does not make sense to me!" What is the best response by the nurse? a. "The provider is making sure the treatment is effective over the long-term." b. "The provider is assessing for visual changes that may occur with drug therapy." c. "We must watch for increased intraocular pressure." d. "We need to monitor for drug toxicity."

d. It must not be handled by pregnant women.

During the administration of finasteride (Proscar), the nurse must remember which important precaution? a. The patient needs to be warned that alopecia is a common adverse effect. b. It must be taken on an empty stomach. c. It is given by deep intramuscular injection to avoid tissue irritation. d. It must not be handled by pregnant women.

c. Check the patient's armband before administering the medication.

During the implementation phase of the nursing process, which action will the nurse perform when administering medications? a. Switch the route of administration based on drug availability. b. Call the patient by name when entering the room to verify the drug is for the right person. c. Check the patient's armband before administering the medication. d. Prepare medications for all patients first, then administer by room to manage time appropriately.

Atorvastatin

Dyslipidemia/ Prevent CVD Dental considerations:Muscle weakness/ Myopathy

Hydrochlorothiazide

Edema/ Hypertension/ Hepatic cirrhosis Dental considerations: Orthostatic hypotension/ Hypotension

a, c, d

Elemental formulations are used for patients with conditions that cause limited ability to digest nutrition. Which substances may be included in elemental formulations? (Select all that apply.) A. Amino acids B. Cow-based proteins C. Dipeptides D. Tripeptides E. Fats

Adverse effect of clarithromycin during pregnancy

Embryotoxic

Alcohol intoxication

Emotional lability, slurred speech, ataxia, coma, blackouts. Serum γ-glutamyltransferase (GGT)—sensitive indicator of alcohol use. AST value is twice ALT value.

A. Low-molecular-weight heparin

Enoxaparin sodium (Lovenox) is an anticoagulant used to prevent and treat deep vein thrombosis and pulmonary embolism. This drug is in which drug group? A. Low-molecular-weight heparin B. Oral anticoagulant C. Glycoprotein IIb/IIIa inhibitor D. Thrombolytic drug

Opioids (e.g., morphine, heroin, methadone) intoxication

Euphoria, respiratory and CNS depression, gag reflex, pupillary constriction (pinpoint pupils), seizures (overdose). Treatment: naloxone, naltrexone.

True/false, you capitalize the generic name?

False, the Trade name only.

Usually a specific drug will bind with any receptor in a lock-and key fashion. true false?

False. only one key for each lock.

For NMS, think FEVER

Fever Encephalopathy Vitals unstable Enzymes up Rigidity of muscles

a. "Be sure to change positions slowly and rise slowly after sitting or lying so as to prevent dizziness and possible fainting because of blood pressure changes."

Furosemide (Lasix) is prescribed for a patient who is about to be discharged, and the nurse provides instructions to the patient about the medication. Which statement by the nurse is correct? a. "Be sure to change positions slowly and rise slowly after sitting or lying so as to prevent dizziness and possible fainting because of blood pressure changes." b. "Take this medication in the evening." c. "If you experience weight gain, such as 5 pounds or more per week, be sure to tell your physician during your next routine visit." d. "Avoid foods high in potassium, such as bananas, oranges, fresh vegetables, and dates."

β-blocker toxicity antidote

Glucagon

Drugs that cause focal to massive hepatic necrosis

Halothane, Amanita phalloides (death cap mushroom), Valproic acid, Acetaminophen

C. By forming a protective barrier over the gastric mucosa

How does sucralfate (Carafate) achieve a therapeutic effect? A. By neutralizing gastric acid B. By enhancing gastric absorption C. By forming a protective barrier over the gastric mucosa D. By inhibiting gastric acid secretion

Simvastatin

Hyperlipidemia/ Secondary prevention of CHD and CVD Dental considerations: Muscle weakeness/ Myopathy

Lisinopril

Hypertension/ Heart failure/ Acute MI Dental Considerations: Orthostatic hypotension, cough may interfere with longer procedures

Hydrocodone/ APAP

Hypertension/Edema/ Congestive heart failure/ Cirrhosis of the liver with edema No effects on dental treatment.

Rapid acting insulin toxicity

Hypoglycemia, rare hypersensitivity reaction

c Notify the physician.

Immediate postoperative assessment of a patient reveals a body temperature of 102° F. Which action will the nurse perform? a Administer acetaminophen as ordered and recheck in 1 hour. b Apply a cooling blanket per protocol. c Notify the physician. d Assess for signs and symptoms of infection related to the surgical wound.

β-blockers (class II antiarrhythmics) toxicity

Impotence, exacerbation of COPD and asthma, cardiovascular effects (bradycardia, AV block, HF), CNS effects (sedation, sleep alterations). May mask the signs of hypoglycemia. β-blockers cause unopposed α1-agonism if given alone for pheochromocytoma or cocaine toxicity.

C. Apply the nitroglycerin patch for 16 hours each and remove for 8 hours at night.

In order to prevent the development of tolerance, the nurse instructs the patient to perform which action? A. Apply the nitroglycerin patch every other day. B. Switch to sublingual nitroglycerin when the patient's systolic blood pressure elevates to >140 mm Hg. C. Apply the nitroglycerin patch for 16 hours each and remove for 8 hours at night. D. Use the nitroglycerin patch for acute episodes of angina only.

a, b, c, e

In which step of the medication process can a medication error occur? (Select all that apply.) a. Procurement b. Prescribing c. Transcribing d. Verification e. Administration

Potassium channel blockers mechanism

Increase AP duration, increase ERP, increase QT interval

Class IA sodium channel blocker mechanism

Increase action potential duration, increase effective refractory period in ventricular action potential, and increase QT

What are the major functions of the alpha-1 receptor?

Increase vascular smooth muscle contraction, increase pupillary dilator muscle contraction (mydriasis), increase intestinal and bladder sphincter muscle contraction

Urine K+ changes with diuretic therapy

Increase with loop and thiazide diuretics. Serum K+ may decrease as a result.

Zolpidem

Insomnia Dental considerations: Xerostomia

Use and side effects of ketoconazole and spironolactone

Ketoconazole and spironolactone are used to treat polycystic ovarian syndrome to reduce androgenic symptoms. Both have side effects of gynecomastia and amenorrhea

Daclizumab, basiliximab clinical use

Kidney transplant rejection prophylaxis

HMG-CoA reductase effect on lipid levels

LDL Δ: big time triple decrease!!! HDL Δ: increase TG Δ: decrease

DOC for alcohol withdrawal

Long-acting benzodiazepine (e.g. chlordiazepoxide, lorazepam, diazepam)

Diuretics that increase blood pH (alkalemia)

Loop diuretics and thiazides cause alkalemia through several mechanisms: -Volume contraction increase AT II increasing Na+/H+ exchange in PCT increasing HCO3− reabsorption ("contraction alkalosis") -K+ loss leads to K+ exiting all cells (via H+/K+ exchanger) in exchange for H+ entering cells -In low K+ state, H+ (rather than K+) is exchanged for Na+ in cortical collecting tubule causing alkalosis and "paradoxical aciduria"

Treatment strategy with type 1 DM

Low-carb diet and insulin replacement

Isocarboxazid

MAOI - orthostatic hypotension, tachycardia, HA, restlessness, insomnia, dizziness, overstimulation including anxiety, agitation, and manic symptoms, dry mouth, paresthesias, diarrhea - limit epi dose to .04mg and aspirate - record baseline and post tx BP - hypertensive crisis with tyramine rich foods

Phenelzine

MAOI - orthostatic hypotension, tachycardia, HA, restlessness, insomnia, dizziness, overstimulation including anxiety, agitation, and manic symptoms, dry mouth, paresthesias, diarrhea - limit epi dose to .04mg and aspirate - record baseline and post tx BP - hypertensive crisis with tyramine rich foods

Tranylcypromine

MAOI - orthostatic hypotension, tachycardia, HA, restlessness, insomnia, dizziness, overstimulation including anxiety, agitation, and manic symptoms, dry mouth, paresthesias, diarrhea - limit epi dose to .04mg and aspirate - record baseline and post tx BP - hypertensive crisis with tyramine rich foods

Osmotic laxatives (drugs, mechanism, clinical use and toxicity)

Magnesium hydroxide, magnesium citrate, polyethylene glycol, lactulose. -Mechanism: provide osmotic load to draw water into the GI lumen. -Use: Constipation. Lactulose also treats hepatic encephalopathy since gut flora degrade it into metabolites (lactic acid and acetic acid) that promote nitrogen excretion as NH4+. -Toxicity: Diarrhea, dehydration; may be abused by bulimics.

c. Intravenously, through a filter

Mannitol (Osmitrol) has been ordered for a patient with acute renal failure. The nurse will administer this drug using which procedure? a. By mouth in a single morning dose b. Through a gravity intravenous drip with standard tubing c. Intravenously, through a filter d. By rapid intravenous bolus

b massaging muscles activates large sensory nerve fibers that send signals to the spinal cord to close the gate, thus blocking painful stimuli from reaching the brain.

Massage therapy is ordered as adjunct treatment for a patient with musculoskeletal pain. The patient asks the nurse how "rubbing my muscles" will help the pain go away. The nurse responds based on the knowledge that a massaging muscles decreases the inflammatory response that initiates the painful stimuli. b massaging muscles activates large sensory nerve fibers that send signals to the spinal cord to close the gate, thus blocking painful stimuli from reaching the brain. c massaging muscles activates small sensory nerve fibers that send signals to the spinal cord to open the gate and allow endorphins to reach the muscles and relieve the pain. d massaging muscles helps relax the contracted fibers and decrease painful stimuli.

Clindamycin

Mean Clean drug -Bacteriostatic -Can lead to suprainfection of Candida albicans (yeast infection/thrush) -**Can lead to pseudomembranous colitis (C-diff → Clostridium difficile) •Severe persistent diarrhea

Nitroimidazoles

Mean clean drug -treat parasitic infections -****If alcohol is ingested, will make you very sick - similar to disulfiram (Antabuse) with alcoholic - avoid Listerine (high alcohol content) Examples o metronidazole (Flagyl)

Most common drug used for endoscopy?

Midazolam; used adjunctively with gaseous anesthetics and narcotics. May cause severe postoperative respiratory depression, BP (treat overdose with flumazenil), anterograde amnesia.

Alcohol withdrawal

Mild alcohol withdrawal: symptoms similar to other depressants. Severe alcohol withdrawal can cause autonomic hyperactivity and DTs (5-15% mortality rate). Treatment for DTs: benzodiazepines.

A. Increasing water absorption into the colon

Milk of magnesia alleviates constipation by which action? A. Increasing water absorption into the colon B. Lubricating the passage of stool C. Increasing bulk within the colon D. Stimulating nerves that regulate defecation

Trazodone

Misc Antidepressant - increase serotonin, used for insomnia - increase CNS depression when combined with opioids

Rasagiline

Misc Parkinson's Disease - arthralgias, depression, dyspepsia, falls - MAOI (type B) inhibitor - Limit EPI dose to .04 mg

Selegiline

Misc Parkinson's Disease - nausea, dizziness, confusion, dry mouth - MAOI--limit epi .04mg

Phenothiazines (typical anti-psychotic, anti-cholinergic)

MoA-D2 dopamine antagonist USE-schizophrenia, depression with psychotic features AR-short term: extrapyramidal effect, dystonia, parkinsonion, resting tremor, akathisia long term: tardive dyskinesia, agranulocytosis, neuroleptic syndrome EX-chlorpromazine, perphenazine, thioridazine, fluphenazine

Atypical Anti-Psychotics (Benzodiazepines)

MoA-long acting, increase the action of the neurotransmitter GABA to GABA receptors AR-bradycardia EX-Lorazepam TEACH -avoid alcohol -avoid smoking

Nonspecific stimulant intoxication

Mood elevation, psychomotor agitation, insomnia, cardiac arrhythmias, tachycardia, anxiety.

Lithium use

Mood stabilizer for bipolar disorder; blocks relapse and acute manic events. Also SIADH.

If the value of the TI is small, then toxicity is _________ likely

More

warfarin (Coumadin) anticoagulant

Most common —-Drug interactions: -Most serious is with aspirin - increased bleeding -Antibiotics can increase the bleeding effects (due to effects on vitamin K)

Gastroesophageal reflux disease (GERD)

Most common GI disease in US Aka heartburn

Selective Serotonin Reuptake Inhibitors (SSRIS)

Most common antidepressant -ine and -pram fluoxetine (Prozac) ocitalopram (Celexa) oescitalopram (Lexapro) sertraline (Zoloft) paroxetine (Paxil)

Acute gout drugs

NSAIDs (naproxen, indomethacin), glucocorticoids, colchicine

Nondepolarizing neuromuscular blocking drugs blockade reversal

Neostigmine (must be given with atropine to prevent muscarinic effects such as bradycardia), edrophonium, and other cholinesterase inhibitors.

Foscarnet toxicity

Nephrotoxicity, electrolyte abnormalities (hypo- or hypercalcemia, hypo- or hyperphosphatemia, hypokalemia, hypomagnesemia) can lead to seizures.

A. nitroglycerin

Patients prescribed sildenafil (Viagra) should be instructed regarding the potential fatal drug interaction with which medication(s)? A. nitroglycerin B. aspirin C. acetaminophen D. Anticoagulants

B. Increased risk of bleeding

Patients taking levothyroxine (Synthroid) and warfarin (Coumadin) concurrently would be monitored for which adverse effect? A. Cardiac arrhythmias B. Increased risk of bleeding C. Excessive weight loss D. Increased risk of deep vein thrombosis

Bile acid resin mechanism

Prevents intestinal absorption of bile acids; liver must use cholesterol to make more

Sulfa drugs

Probenecid, Furosemide, Acetazolamide, Celecoxib, Thiazides, Sulfonamide antibiotics, Sulfasalazine, Sulfonylureas. Patients with sulfa allergies may develop fever, urinary tract infection, Stevens- Johnson syndrome, hemolytic anemia, thrombocytopenia, agranulocytosis, and urticaria (hives). Symptoms range from mild to life threatening.

B nonselective beta-adrenergic antagonist.

Propranolol (Inderal) is an effective A selective alpha-adrenergic antagonist. B nonselective beta-adrenergic antagonist. C beta1-adrenergic antagonist. D beta2-adrenergic antagonist.

What is the heparin toxicity antidote, and how does it work?

Protamine sulfate, positively charged molecule that binds negatively charged heparin

Amiodarone toxicity

Pulmonary fibrosis, hepatotoxicity, hypothyroidism/hyperthyroidism, act as hapten (corneal deposits, blue/gray skin deposits resulting in photodermatitis), neurologic effects, constipation, CV effects (bradycardia, heart block, HF)

Drugs that cause gout

Pyrazinamide, Thiazides, Furosemide, Niacin, Cyclosporine

Morphine clinical use

Relief of Moderate-Severe Acute & Chronic Pain associated with: Cancer, MI (vasodilating properties), Relief of Dyspnea (caused by acute LV failure & pulmonary edema), Preanesthetic Medication

Class IC sodium channel blockers clinical use

SVTs, including atrial fibrillation. Only as a last resort in refractory VT.

β-blockers overdose treatment

Saline, atropine, glucagon

SLUD =

Salivation, Lacrimation, Urination, Defecation

Nystatin MoA

Same as amphotericin B. Topical use only as too toxic for systemic use.

A Blocking the reuptake of neurotransmitters at nerve endings

Selective serotonin reuptake inhibitors (SSRIs) and tricyclic antidepressants (TCAs) both function by which mechanism? A Blocking the reuptake of neurotransmitters at nerve endings B Increasing alertness levels in the brain C Decreasing levels of epinephrine and serotonin at nerve endings D Increasing the placebo effect

Serotonin Norepinephrine Reuptake Inhibitors (SNRIS)

Similar to the SSRIS venlafaxine (Effexor) desvenlafaxine (Pristiq) duloxetine (Cymbalta)

Anesthetic cartridges o Alkalinizing agent

Sodium hydroxide -raises the pH to between 6-7 (makes onset faster)

Interference with natural defense mechanisms

Some drugs decrease body's ability to fight infection Ex: corticosteroids, immunosuppressive drugs

OPIOID NARCOTIC ANALGESICS Addiction Potential

Stronger the opioid, more addiction potential -Build tolerance to the Not to the myosis or constipation -methadone (Dolophine) opioid agonist to treat opioid addicts -naloxone (Narcan) parenteral opioid for treating overdose

Anti-Tuberculosis Agents

TB is caused by Mycobacterium tuberculosis -Treatment is difficult treat with multiple drugs at same time (RIPE) rifampin isoniazid pyrazinamide ethambutol (Ripe looking lungs)

Desipramine

TCA, +1 - sedation, dry mouth, orthostatic hypotension, tachycardia - greater than 100 mg per day can cause interaction with epic (limit 2.5 carpules) - Additive CNS depression with opioids and anti-anxiety agents

Nortriptyline

TCA, +1-2 - sedation, dry mouth, orthostatic hypotension, tachycardia - greater than 50 mg per day can cause interaction with epic (limit 2.5 carpules) - Additive CNS depression with opioids and anti-anxiety agents

Doxepin

TCA, +2 - sedation, dry mouth, orthostatic hypotension, tachycardia - greater than 100 mg per day can cause interaction with epic (limit 2.5 carpules) - Additive CNS depression with opioids and anti-anxiety agents

Imipramine

TCA, +2 - sedation, dry mouth, orthostatic hypotension, tachycardia - greater than 100 mg per day can cause interaction with epic (limit 2.5 carpules) - Additive CNS depression with opioids and anti-anxiety agents

Amitriptyline

TCA, +4 (worst for xerostomia) - sedation, dry mouth, orthostatic hypotension, tachycardia - greater than 100 mg per day can cause interaction with epic (limit 2.5 carpules) - Additive CNS depression with opioids and anti-anxiety agents

Drugs that cause hyperglycemia

Tacrolimus, Protease inhibitors, Niacin, HCTZ, Corticosteroids

Drugs that cause teeth discoloration

Tetracyclines (TETra=bad TEeTh)

c. Determine if the client has any known allergies

The 72-year-old client is admitted to the medical unit diagnosed with an acute exacerbation of diverticulosis. The health-care provider has prescribed the intravaneous antibiotic ceftriaxone (Rocephin). Which intervention should the nurse implement first? a. Assesss the client's most recent vital signs b. Send a stool specimen to the laboratory c. Determine if the client has any known allergies d. Monitor the client's white blood cell count

Dose

The ___ is the amount of a drug that is necessary to produce the desired response.

Somatic Nervous system

The ______ deals with skeletal muscles.

a, b, d

The acetic acid derivative indomethacin (Indocin) has which properties? (Select all that apply.) A. Antiinflammatory B. Antirheumatic C. Antinausea D. Antipyretic E. Anticonvulsant

B. Endometrial cancer

The addition of continuous administration of progestin to an estrogen regimen reduces the risk of which cancer? A. Ovarian cancer B. Endometrial cancer C. Breast cancer D. Vaginal cancer

a. Do not drink alcohol with this medication

The client receives metronidizole (Flagyl) for treatment of a vaginal infection. What does the best medication education by the nurse include? a. Do not drink alcohol with this medication b. Eat at least two cups of yogurt daily while on this medication. c. Do not take this medication with milk or milk products. d. Drink at least 2,000 mL of fluid with this medication

c. "No. Differences such as your sex can result in different side effects."

The client says to the nurse "My wife and I take the same drug, but we have different side effects. "Are we doing something wrong?" What is the best response by the nurse? a. "I'll have to check. What is the name of the drug you were using?" b. "Possibly. This could happen if one uses generic or brand name drugs." c. "No. Differences such as your sex can result in different side effects." d. "I'm not sure. Maybe the drug is not the same; you should check it."

a. The medication must be tapered off when discontinued.

The client with chronic low back pain has been taking baclofen (Lioresal), a muscle relaxant. Which instruction should the nurse review with the client? a. The medication must be tapered off when discontinued. b. The client may consume no more than one glass of wine per day. c. The client should not take the medication before bedtime. d. The medication can cause gastric ulcer formation.

b. Ensure the client drinks at least 2000 mL of water daily

The client with inflammatory bowel diease is prescribed sulfasalazine (Azulfidine), a sulfonamide antibiotic. Which intervention should the nurse implement when administering this medication? a. Administer the medication once a day with breakfast b. Ensure the client drinks at least 2000 mL of water daily c. Assess the client's stool for steatorrhea and mucus. d. Explain that the medication may cause slight bruising

a. "I take aspirin daily to help prevent heart disease."

The client with osteoarthritis is prescribed the COX-2 inhibitor celecoxib (Celebrex), a nonsteroidal anti-inflammatory drug (NSAID). Which statement by the client warrants intervention by the nurse? a. "I take aspirin daily to help prevent heart disease." b. "I know I am overweight and need to lose 50 pounds." c. "I walk 30 minutes at least three times a week.) d. "I am allergic to penicillin and aminoglycosides."

a. "I drink at least eight ounces of water a day."

The client with renal calculi was prescribed allopurinol (Zyloprim) for uric acid stones. Which statement indicates the client understands the teaching? a. "I drink at least eight ounces of water a day." b. "I really enjoy a glass of wine with my evening meal." c. "i should have at least 8 ounces of milk with my pill." d. "My joints ache so I take a couple of aspirin."

Time response curve

The curve that allows us to determine how often a drug needs to be taken in order to maintain the ceiling effect.

c Use of herbs or over-the-counter medications

The emergency department nurse is documenting the history of a patient of Asian culture. The patient states that she does not take any medications, but the nurse notes a bottle of capsules in the patient's purse. What information will the nurse collect next? a Vital signs b Primary care provider name c Use of herbs or over-the-counter medications d Insurance information

B. Reduce inflammation

The health care provider has ordered ciprofloxacin/dexamethasone (Ciprodex) otic drops for a patient with OE. The nurse knows the combination medication has both antibacterial medication and topical steroid medication. What is the purpose of the steroid in this medication? A. Relieve pain B. Reduce inflammation C. Treat infection D. Decrease the amount of earwax

Therapeutic effect

The intended effect of a drug.

A atomoxetine (Strattera)

The mother of a child with attention deficit hyperactivity disorder (ADHD) who has been prescribed methylphenidate (Ritalin) expresses concern regarding the use of a controlled substance to treat her child and asks if there are any other options. The nurse's response is based on the knowledge that an option for treatment for ADHD might include which non-controlled central nervous system (CNS) stimulant? A atomoxetine (Strattera) B dextroamphetamine sulfate (Dexedrine) C methylphenidate (Concerta) D amphetamine aspartate (Adderall)

B increase levels of acetylcholine in the brain by blocking its breakdown.

The nurse administering donepezil (Aricept) to a patient understands that the expected therapeutic action of this drug is to A relieve anxiety and restless behavior of the patient. B increase levels of acetylcholine in the brain by blocking its breakdown. C block the effects of acetylcholine at the presynaptic neurons. D help control associated urinary incontinence.

d Notify the provider and document the error on an incident report.

The nurse administers a medication to the wrong client. Which is the appropriate nursing action following this error? a Assess the client for an adverse reaction and report if an adverse event occurs. b Document the medication error. No further action is required. c Report the error and document the medication on the patient chart. d Notify the provider and document the error on an incident report.

D. octreotide (Sandostatin)

The nurse admitting a patient with acromegaly anticipates administering which medication? A. desmopressin (DDAVP) B. corticotropin (Acthar) C. somatropin (Nutropin) D. octreotide (Sandostatin)

b. Dilute urine

The nurse caring for a patient post colon resection is assessing the patient on the first postoperative day. The nasogastric tube (NG) remains patent and continues at low intermittent wall suction. The IV is patent and infusing at 125 mL per hour. The patient reports pain at the incision site rated at a three on a zero-to-ten rating scale. During your initial shift assessment, the patient complains of cramps in her legs and a tingling sensation in her feet. Your assessment indicates decreased deep tendon reflexes (DTRs). You suspect the patient has hypokalemia. What other signs or symptoms would you expect this patient to exhibit? a. Increased bowel motility b. Dilute urine c. Increased muscle strength d. Excessive thirst

b. Stop the transfusion immediately.

The nurse caring for a patient receiving a transfusion notes that 15 minutes after the infusion of packed red blood cells (RBCs) has begun, the patient is having difficulty breathing and chest tightness. What is the most appropriate initial action for the nurse to take? a. Assess the patient's chest sounds and vital signs. b. Stop the transfusion immediately. c. Remove the patient's intravenous access. d. Notify the patient's physician.

D. "I should use a soft toothbrush for dental hygiene."

The nurse evaluates and determines the patient has a good understanding of the discharge instructions regarding warfarin (Coumadin) when the patient responds with which statement? A. "I will double my dose if I forget to take it the day before." B. "I should keep taking ibuprofen for my arthritis." C. "I should decrease the dose if I start bruising easily." D. "I should use a soft toothbrush for dental hygiene."

b. "I will need to take extra care of my teeth and gums while on this medication."

The nurse has given medication instructions to a patient receiving phenytoin (Dilantin). Which statement by the patient indicates that the patient has an adequate understanding? a. "I will be able to stop taking this drug once the seizures stop." b. "I will need to take extra care of my teeth and gums while on this medication." c. "I can skip doses if the side effects bother me." d. "I can go out for a beer while on this medication."

B. The patient's warfarin dose is therapeutic.

The nurse interprets a patient's international normalized ratio (INR) value of 2.5. What is the meaning of this reported value? A. The patient is not receiving enough warfarin for a therapeutic effect. B. The patient's warfarin dose is therapeutic. C. The patient is not receiving enough subcutaneous heparin for a therapeutic effect. D. The patient is receiving a dangerously high amount of heparin.

b. Cephalexin (Keflex), a cephalosporing, to a client who is allergic to penicillin

The nurse is administering medications to clients on a urology floor. Which medication should the nurse question? a. Trimethoprim sulfa (Bactrim), a sulfa antibiotic, to a client post-prostate surgery b. Cephalexin (Keflex), a cephalosporing, to a client who is allergic to penicillin c. Nitrofurantoin (Macrodantin), a sulfa antibiotic, to a client with urinary stasis d. Ceftriaxone (Rocephin), a third-generation cephalosporin, to a client who is pregnant

D. prolong the effectiveness of penicillin therapy.

The nurse is administering probenecid (Benemid) to a patient with recurrent strep throat. The nurse teaches the patient that the most likely reason for taking this medication is to A. prevent the occurrence of gout. B. increase uric acid excretion. C. inhibit bacterial growth and replication. D. prolong the effectiveness of penicillin therapy.

b. Hyperglycemia

The nurse is administering total parenteral nutrition (TPN) to a client who underwent surgery for gastric cancer. What is a major complication of TPN? a. Extreme hunger b. Hyperglycemia c. Hypotension d. Hypoglycemia

b. Take the drops out of the refrigerator 1 hour before the dose is due.

The nurse is administrating eardrops that have been refrigerated. Which action by the nurse is correct before administering the drops? a. Heat the chilled solution for 10 seconds in the microwave. b. Take the drops out of the refrigerator 1 hour before the dose is due. c. Leave the drops in the refrigerator until use. d. Soak the bottle for 60 seconds in a container of very hot water.

A carvedilol (Coreg)

The nurse is admitting a patient with a history of angina and hypertension who is currently experiencing moderate heart failure. The patient's current medication regimen includes digoxin (Lanoxin), furosemide (Lasix), and quinapril (Accupril). Which medication would be most beneficial for the health care provider to add to this patient's treatment plan? A carvedilol (Coreg) B propranolol (Inderal) C esmolol (Brevibloc) D sotalol (Betapace)

c. Beef liver & onions

The nurse is assisting the patient with gout to mark his menu. Which of the following would the nurse recommend he avoid? a. Cheese pizza b. Tossed salad with cucumbers c. Beef liver & onions d. Egg salad sandwich

B. metformin (Glucophage)

The nurse is caring for a patient scheduled to undergo a cardiac catheterization procedure utilizing iodine-based contrast material. The nurse would question an order for which medication to be given to this patient the day before the scheduled procedure? A. acarbose (Precose) B. metformin (Glucophage) C. repaglinide (Prandin) D. pioglitazone (Actos)

D. Use for anemic patient with renal failure

The nurse is caring for a patient who is prescribed epoetin alfa (Epogen). What does the nurse expect as the reason for use of this medication in this patient? A. To replace blood loss from surgery B. To replace blood loss from a trauma C. Treatment of pregnancy associated anemia D. Use for anemic patient with renal failure

B. Diuretics and calcium channel blockers

The nurse is conducting a community education program. When explaining different medication regimens to treat hypertension, it would be accurate to state that African Americans probably respond best to which combination of medications? A. ACE inhibitors and diuretics B. Diuretics and calcium channel blockers C. Diuretics and beta blockers D. ACE inhibitors and beta blockers

a, b, c, d

The nurse is discussing management of symptoms of an upper respiratory tract infection. Which classes of medications are often used in treating the symptoms of upper respiratory tract infections? (Select all that apply.) A. Antihistamines B. Nasal decongestants C. Antitussives D. Expectorants E. Antibiotics

A. Drowsiness and dizziness

The nurse is discussing use of antitussive medications with a patient. What common side effect does the nurse include in the patient teaching? A. Drowsiness and dizziness B. Diarrhea and abdominal cramping C. Tremors and anxiety D. Bradycardia and increased lacrimation

b, d, e

The nurse is discussing vitamins with a group of women at a community center. The nurse is discussing water-soluble vitamins and fat-soluble vitamins. Which vitamins are fat-soluble vitamins? Select all that apply. a. Vitamin C b. Vitamin D c. Folic Acid d. Vitamin A e. Vitamin E

A. Mother demonstrates accurate medication administration technique.

The nurse is educating the mother of an infant on how to instill eardrops for otitis media with a perforated eardrum. Which is an appropriate nursing outcome criterion for this patient? A. Mother demonstrates accurate medication administration technique. B. Mother takes infant's temperature prior to administration of medication. C. Patient lies still for administration of medication and 5 minutes after administration. D. Patient stops using medication when the ear no longer has active drainage.

B. 30 minutes before a meal

The nurse is providing education to a patient about the time to take glipizide (Glucotrol). For maximum benefit, the nurse will tell the patient to administer glipizide at which time? A. In the morning B. 30 minutes before a meal C. 15 minutes postprandial D. At bedtime

C. "I will use the eardrops in the ear canal to treat this condition."

The nurse is providing education to a patient diagnosed with otitis externa (OE). Which statement by the patient indicates an understanding of the OE treatment plan? A. "I will finish taking all the pills for my ear infection." B. "This is contagious, and I need to stay away from others for 24 hours." C. "I will use the eardrops in the ear canal to treat this condition." D. "The cold I had last week caused this to occur; it will go away on its own."

e, f

The nurse is providing instructions to a patient who has a new prescription for a corticosteroid metered-dose inhaler. Which statement by the patient indicates that further instruction is needed? (Select all that apply.) a. "I will clean the plastic inhaler casing weekly by removing the canister and then washing the casing in warm soapy water. I will then let it dry before reassembling." b. "I will continue to use this inhaler, even if I am feeling better." c. "I will rinse my mouth with water after using the inhaler and then spit out the water." d. "I will use a peak flow meter to measure my response to therapy." e. "I will use this inhaler for asthma attacks." f. "I will gargle after using the inhaler and then swallow."

d. "Take the tablets with water, not milk or juice."

The nurse is providing teaching to a patient who will be taking the laxative bisacodyl (Dulcolax). Which statement by the nurse is appropriate during this teaching session? a. "This laxative is not known to cause dependence." b. "In a normal bowel pattern, a bowel movement occurs daily." c. "You can crush the laxative tablets for improved action." d. "Take the tablets with water, not milk or juice."

c. Call the prescriber to question the route that is ordered.

The nurse is reviewing new postoperative orders and notes that the order reads, "Give hydroxyzine (Vistaril) 50 mg IV prn nausea or vomiting." The patient is complaining of slight nausea. Which action by the nurse is correct at this time? a. Give the dose orally instead of intravenously. b. Hold the dose until the patient complains of severe nausea. c. Call the prescriber to question the route that is ordered. d. Give the patient the IV dose of hydroxyzine as ordered.

a. Prolonging action potential duration

The nurse is reviewing the classes of antidysrhythmic drugs. Amiodarone (Cordarone) is classified on the Vaughan Williams classification as a class III drug, which means it works by which mechanism of action? a. Prolonging action potential duration b. Blocking slow calcium channels c. Blocking sodium channels and affecting phase 0 d. Decreasing spontaneous depolarization and affecting phase 4

b, c, d, a

The nurse is reviewing the various types of insulins. For each insulin listed below, place in order from shortest duration (1) to longest duration (4). a. Glargine insulin b. Aspart insulin c. Regular insulin d. NPH insulin

A. "Hold the pinna down and back."

The nurse is teaching the mother of a 1-year-old patient how to instill ear drops for OE. What instructions on instillation does the nurse provide to the mother? A. "Hold the pinna down and back." B. "Just put in as many drops as you can get into the ear." C. "Put a cotton ball in to keep the medication in the ear." D. "Keep the ear drops in cold place so they work better."

A. Decrease in gastrointestinal motility

The nurse monitors a patient prescribed dicyclomine (Bentyl) for which therapeutic effect? A. Decrease in gastrointestinal motility B. Decrease in urinary frequency C. Increase in heart rate D. Increase in blood pressure

c pain relief is best obtained by administering analgesics around the clock.

The nurse plans pharmacologic therapy for a patient with pain based on the knowledge that a narcotic analgesics should not be used for more than 24 hours secondary to the risk of addiction. b analgesics should be administered as needed (prn) to minimize side effects. c pain relief is best obtained by administering analgesics around the clock. d patients should request analgesics when the pain level reaches a "6" on a scale of 1 to 10.

D. Undiluted over 2 minutes

The nurse plans to administer 50 mg diphenhydramine (Benadryl) intravenously. How will the nurse administer this medication? A. Diluted in 50 mL normal saline over 30 minutes B. Undiluted over 1 minute C. Diluted in 100 mL D5W over 20 minutes D. Undiluted over 2 minutes

D. furosemide (Lasix)

The nurse prepares to administer which diuretic to treat a patient with acute pulmonary edema? A. spironolactone (Aldactone) B. amiloride (Midamor) C. triamterene (Dyrenium) D. furosemide (Lasix)

D. promotes motility in the small intestine.

The nurse recognizes that metoclopramide (Reglan) is useful in treating postoperative nausea and vomiting because it A. decreases motility in the gastrointestinal tract. B. decreases chemoreceptor stimulation. C. improves the body's response to analgesia. D. promotes motility in the small intestine.

a, b, c, e

The nurse recognizes that patient teaching regarding warfarin (Coumadin) has been successful when the patient acknowledges an increased risk of bleeding with concurrent use of which herbal product? (Select all that apply.) A. Garlic B. Ginkgo C. Dong quai D. Glucosamine E. St. John's wort

D. digoxin immune Fab.

The nurse reviews a patient's laboratory values and finds a digoxin level of 10 ng/mL and a serum potassium level of 6.2 mEq/L. The nurse would notify the health care provider and anticipate administering A. sodium polystyrene sulfonate. B. atropine. C. epinephrine. D. digoxin immune Fab.

b. Three times

The nurse should check a medication how many times before administration of a medication under the "right drug" part of the Six Rights? a. One time b. Three times c. Five times d. Depends on the drug being administered

d. Therapeutic effect on patient

The nurse should include which information when evaluating the outcome after a patient's medication has been administered? a. Dose of medication administered b. Time medication administered c. Patient allergies to medications d. Therapeutic effect on patient

C. 14-year-old boy with a history of flulike symptoms

The nurse should question an order written to administer acetylsalicylic acid (aspirin) to which patient? A. 45-year-old man with a history of heart attack B. 28-year-old woman with a history of sports injury C. 14-year-old boy with a history of flulike symptoms D. 62-year-old woman with a history of stroke

a, c, d

The nurse understands that drugs exert their actions on the body by (Select all that apply.) a. interacting with receptors. b. making the cell perform a new function. c. inhibiting the action of a specific enzyme. d. altering metabolic chemical processes.

B. glucagon

The nurse will advise the patient to treat hypoglycemia with which drug? A. propranolol (Inderal) B. glucagon C. acarbose (Precose) D. bumetanide (Bumex)

d. mannitol (Osmitrol)

The nurse will be giving ophthalmic drugs to a patient with glaucoma. Which drug is used to reduce intraocular pressure? a. tobramycin (Tobrex) b. bacitracin (AK-Tracin) c. ketorolac (Acular) d. mannitol (Osmitrol)

b Two

The nurse will check how many patient identifiers, required by The Joint Commission's standards, prior to administering a medication to a patient? a One b Two c Three d Four

a. Dry mouth

The nurse will monitor for which adverse effect when administering an anticholinergic drug? a. Dry mouth b. Pupillary constriction c. Diaphoresis d. Excessive urination

B. Constipation

The nurse will monitor for which adverse reaction to aluminum-containing antacids such as aluminum hydroxide (Amphojel)? A. Diarrhea B. Constipation C. Gastrointestinal upset D. Fluid retention

B. Activated partial thromboplastin time (aPTT)

The nurse would assess which laboratory value to determine the effectiveness of intravenous heparin? A. Complete blood count (CBC) B. Activated partial thromboplastin time (aPTT) C. Prothrombin time (PT) D. Blood urea nitrogen (BUN)

A. scopolamine (Transderm-Scōp)

The nurse would expect to teach a patient about which antiemetic commonly used to prevent motion sickness? A. scopolamine (Transderm-Scōp) B. droperidol (Inapsine) C. prochlorperazine (Compazine) D. metoclopramide (Reglan)

C. Hyperparathyroidism

The nurse would question a physician's prescription for vitamin D in a patient with which diagnosis? A. Chronic renal failure B. Hypothyroidism C. Hyperparathyroidism D. Graves' disease

C. Hypotension

The nurse would question an order for a calcium channel blocker in a patient with which condition? A. Angina pectoris B. Increased intracranial pressure C. Hypotension D. Dysrhythmia

D. Addison's disease

The nurse would question an order for aminoglutethimide in a patient with which condition? A. Metastatic breast cancer B. Cushing's syndrome C. Adrenal malignancy D. Addison's disease

B. Complete bowel obstruction

The nurse would question an order for colesevelam (Welchol) in a patient with which condition? A. Renal disease B. Complete bowel obstruction C. Glaucoma D. Hepatic disease

C. Pregnancy

The nurse would question an order to administer misoprostol (Cytotec) to a patient with which condition? A. Chronic obstructive pulmonary disease B. Gastroesophageal reflux disease C. Pregnancy D. Peptic ulcer

C. Hyperkalemia

The nurse would question the administration of testosterone to a patient with which condition? A. Hypocalcemia B. Hyponatremia C. Hyperkalemia D. Hypovolemia

A. Uncontrolled hypertension

The nurse would question the order for epoetin in a patient with which condition? A. Uncontrolled hypertension B. Anemia C. Chronic renal failure D. HIV/AIDS

c. Prothrombin time (PT) 1 1/2 to 2 times the control

The nursing instructor is talking about anticoagulant therapy. The instructor explains that when administering warfarin (Coumadin) to a client with deep vein thrombophlebitis, the nurse monitors the PTT, PT , INR, and hematocrit. Which laboratory value would the instructor teach the class that indicate warfarin is at therapeutic levels? a. International normalized ratio (INR) of 3 to 4 b. Hematocrit of 32% c. Prothrombin time (PT) 1 1/2 to 2 times the control d. Partial thromboplastin time (PTT) 1 1/2 to 2 times the control

d. Assessment, Diagnoses, Planning, Implementation, Evaluation

The nursing process is important as a well-established, research-supported framework for professional nursing practice. Which is the correct order for the steps of the nursing process? a. Evaluation, Planning, Diagnoses, Assessment, Implementation b. Planning, Assessment, Diagnoses, Implementation, Evaluation c. Diagnoses, Assessment, Planning, Evaluation, Implementation d. Assessment, Diagnoses, Planning, Implementation, Evaluation

B. "It's best to keep it in its original container away from heat and light."

The patient asks how nitroglycerin should be stored while traveling. What is the nurse's best response? A. "You can protect it from heat by placing the bottle in an ice chest." B. "It's best to keep it in its original container away from heat and light." C. "You can put a few tablets in a resealable bag and carry in your pocket." D. "It's best to lock them in the glove compartment of your car to keep them away from heat and light."

D. "Glucosamine sulfate with chondroitin has demonstrated promising results in the treatment of joint stiffness and pain. Discuss this with your health care provider."

The patient questions a nurse about herbal treatments for arthritic pain. What is the nurse's best response? A. "Ginkgo biloba has shown tremendous benefit as an antiinflammatory drug." B. "High doses of vitamins have been used for many years to help maintain joint health." C. "There really are no safe herbal treatments for pain. Your best action would be to take your prescription medications." D. "Glucosamine sulfate with chondroitin has demonstrated promising results in the treatment of joint stiffness and pain. Discuss this with your health care provider."

A. 0.2 mg

The patient weighs 44 pounds and is to receive a loading dose of digoxin. The loading dose is to be 0.03 mg/kg in three divided doses. How much will the nurse administer per dose? A. 0.2 mg B. 0.3 mg C. 0.4 mg D. 0.6 mg

d. Tell the woman to get the hepatitis B immune globulin (HBIG)

The public health nurse notified a young woman that one of her sexual contacts was positive for hepatitis B. The woman denied ever having hepatitis B or having received the hepatitis B vaccinations. Which information is most important for the nurse to discuss with the woman? a. Instruct the woman not tho unprotected intercourse b. Advise the woman not to drink any type of alcoholic beverage c. Encourage the client to get the hepatitis B vaccination d. Tell the woman to get the hepatitis B immune globulin (HBIG)

a, b, c, d, e, f, g

There are multiple factors that affect medication response. The nurse recognizes which factors as having a possible effect on medication response? (Select all that apply.) a. Patient compliance with therapy b. Genetic influences c. Body composition d. Use of alternative therapies e. Diet and nutrition f. Level of education g. Socioeconomic factors

Onset of a drug

Time required for the drug to produce its therapeutic effect

C. 8

To avoid fecal impaction, psyllium (Metamucil) should be administered with at least how many ounces of fluid? A. 4 B. 6 C. 8 D. 10

A. Rinse the mouth after each use.

To prevent oral candidiasis, it is most important for the nurse to teach a patient using a steroid inhaler to perform which action? A. Rinse the mouth after each use. B. Minimize use of an inhaler to every other day. C. Swish and swallow with mycostatin after each use. D. Report any gingival irritation to the health care provider.

C. Avoid drinking grapefruit juice.

To prevent the occurrence of cinchonism in a patient prescribed quinidine, what does the nurse emphasize as important? A. Wear sunscreen. B. Change positions slowly. C. Avoid drinking grapefruit juice. D. Increase dietary intake of potassium.

B Limit dietary intake of fat to <30% of total calories.

To reduce the gastrointestinal side effects of orlistat (Xenical), what will the nurse encourage the patient to do? A Take the medication with an antacid. B Limit dietary intake of fat to <30% of total calories. C Supplement diet with fat-soluble vitamins. D Increase fluid and fiber in the diet.

Sotalol toxicity

Torsades de pointes, excessive β blockade.

Glucocorticoids clinical use

Transplant rejection prophylaxis (immunosuppression), many autoimmune disorders, inflammation

Cyclosporine clinical use

Transplant rejection prophylaxis, psoriasis, rheumatoid arthritis

Therapeutically equivalent-

Two formulations of drug have equal therapeutic effect on the body Ex: ibuprofen

Biologically equivalent

Two formulations of drug produce similar concentrations in blood and tissues

Intermediate acting insulin (NPH) clinical use

Type 1 DM, type 2 DM, GDM

Rapid acting insulin clinical use

Type 1 DM, type 2 DM, GDM (postprandial glucose control)

Pramlintide clinical use

Type 1 and type 2 DM

A person is having anaphylatic shock. this is a what type of reaction?

Type 1. it is mediated by the immunoglobulin E (IGE) antibodies. binding to histamine, leukotrienes, and prostoglandin are released, producing vasodilation, edema and inflammation.

a person is wearing a cheap watch, their skin breaks out underneath the watch, this is called a what?

Type 4 (delayed hypersensitivity) mediated by the sensitized T lymphocytes. when the cell contacts the antigen, an inflammatory reaction is produced by lymphokines, neutrophils, and macrophages. contact dermatitis is typical.

B. Absolute neutrophil count (ANC) count of 12,000 cells/mm3

Upon which patient finding would the nurse hold the ordered dose of filgrastim (Neupogen) and notify the provider? A. Fever of 99.5° F B. Absolute neutrophil count (ANC) count of 12,000 cells/mm3 C. White blood cell (WBC) count of 4.5/mm3 D. Blood pressure of 142/88 mm Hg

pharmacotherapeutics

Use of drug to treat different disease states.

Propofol

Used for sedation in ICU, rapid anesthesia induction, short procedures. Less postoperative nausea than thiopental. Potentiates GABA-A

anticoagulant clopidogrel (Plavix) - BOARD ALERT

Used to prevent blood clots after recent heart attack or stroke

adrenergic Cardiac stimulators

Used to stimulate and restore the heartbeat in emergency situations -epinephrine

Drugs that cause cutaneous flushing

Vancomycin, Adenosine, Niacin, Ca2+ channel blockers (VANC)

Treatment of MRSA

Vancomycin, daptomycin, linezolid, tigecycline, ceftaroline

What are the major functions of the βeta-2 receptor?

Vasodilation, bronchodilation, increase lipolysis, increase insulin release, decrease uterine tone (tocolysis), ciliary muscle relaxation, increase aqueous humor production

Reversal of warfarin toxicity

Vitamin K; for rapid reversal give fresh frozen plasma

Orthostatic hypotension, NSAID diminishes effects

What are 2 common side effects of ACE Inhibitors?

Muscle pain, abdominal pain, joint pain, possible liver damage

What are 4 major side effects of taking Lipitor?

DEA

What branch of the Department of Justice requires all persons dealing with drugs to register annually to obtain a permit number in order to manufacture or prescribe controlled drugs that have a potential for abuse?

C. Ejaculatory dysfunction

What common side effect would the nurse include in the discharge teaching for a patient receiving finasteride (Proscar)? A. Hair loss B. Increased libido C. Ejaculatory dysfunction D. Muscle weakness

A. Metabolic alkalosis

What condition will the nurse monitor for in a patient using sodium bicarbonate to treat gastric hyperacidity? A. Metabolic alkalosis B. Metabolic acidosis C. Hyperkalemia D. Hypercalcemia

D. "Continue your exercise program, and maintain a diet high in omega-3 fatty acids."

What education and discharge information for a patient receiving an antilipemic medication would the nurse include for the patient? A. "This medication will take over for other interventions you have been trying to decrease your cholesterol." B. "It is important for you to double your dose if you miss one in order to maintain therapeutic blood levels." C. "Stop taking the medication if it causes nausea and vomiting." D. "Continue your exercise program, and maintain a diet high in omega-3 fatty acids."

Time response curve

What indicates the frequency that a drug should be taken?

Hypothyroidism

What is Synthroid commonly used for?

C. Reducing ammonia levels

What is an additional function of lactulose, a hyperosmotic laxative? A. Decreasing peripheral edema B. Correcting sodium imbalances C. Reducing ammonia levels D. Alleviating galactose intolerance

d. Metabolism

What is an alternate name for biotransformation of a drug? a. Absorption b. Dilution c. Excretion d. Metabolism

B Smoking cessation

What is another approved and indicated use for bupropion (Zyban), a second-generation antidepressant? A Orthostatic hypotension B Smoking cessation C Anorexia in patients undergoing chemotherapy D Nocturnal enuresis in children

C. Blood pressure

What is included in the priority assessment before administering isosorbide mononitrate (Imdur)? A. Serum electrolytes B. Blood urea nitrogen (BUN) and creatinine C. Blood pressure D. Level of consciousness

The amount of a drug necessary to produce a desired effect in 50% of the people who take the drug.

What is the Effective Dose 50 (ED50)?

b. They will kill the bacteria.

What is the action of bactericidal drugs? a. They have a high potency. b. They will kill the bacteria. c. They disrupt normal cell function. d. They will slow the slow growth of the bacteria.

A. They compete with histamine for binding sites on the parietal cells.

What is the action of histamine2-receptor antagonists? A. They compete with histamine for binding sites on the parietal cells. B. They irreversibly bind to H+/K+ ATPase. C. They cause a decrease in stomach pH. D. They decrease signs and symptoms of allergies related to histamine release.

Tenormin

What is the common Trade Name for the drug atenolol?

atorvasatin

What is the generic name for Lipitor?

levothyroxine

What is the generic name for the drug Synthroid?

C. It decreases peristalsis in the intestinal wall.

What is the mechanism of action of diphenoxylate (Lomotil)? A. It increases intestinal excretion of water. B. It increases intestinal motility. C. It decreases peristalsis in the intestinal wall. D. It decreases the reabsorption of water in the bowel.

B. Digestive intolerance

What is the most common adverse effect for infants receiving nutritional supplements? A. Seizures B. Digestive intolerance C. Reflux D. Poor weight gain

Hypertension

What is the most common use of ACE Inhibitors such as lisinopril?

a Monitoring for and reporting any adverse effects noted during Phase IV studies

What is the nurse's role in the development of new and investigational drugs? a Monitoring for and reporting any adverse effects noted during Phase IV studies b Selecting patients to participate in Phase I studies c Identifying patients who are receiving placebo drugs during Phase III studies d Informing patients of the specific drug they are receiving within a blinded investigational study during Phase III

Opioid (narcotic), analgesic combination

What is the pharmacologic category for hydrocodone with acetaminophen?

ACE Inhibitor (Angiotensin-Converting Enzyme)

What is the pharmacologic category for lisinopril?

FDA

What regulatory agency regulates the labeling and advertising of prescription drugs?

D. "This inhaler is not to be used alone to treat an acute asthma attack."

What statement by the patient, who is using the anticholinergic inhaler ipratropium bromide (Atrovent), indicates to the nurse that teaching has been successful? A. "I will increase my intake of caffeine." B. "I may gain weight as a result of taking this medication." C. "Nausea and vomiting are common adverse effects of this medication, so I will always take it with meals." D. "This inhaler is not to be used alone to treat an acute asthma attack."

B. Neurologic status

What system assessment will the nurse monitor in a patient due to a known common side effect of traditional antihistamines? A. Cardiac status B. Neurologic status C. Respiratory status D. Gastrointestinal status

A. Fluid balance

What would the nurse assess when monitoring for the therapeutic effectiveness of vasopressin? A. Fluid balance B. Patient's pain scale C. Serum albumin levels D. Adrenocorticotropic hormone (ACTH) levels

d. Plasmin

When a human body receives an injury that causes blood loss, there are several processes involved in maintaining hemostasis. When a blood clot is no longer needed, what digests the fibrinogen and fibrin? a. Thrombin b. Plasminogen c. Prothrombin d. Plasmin

a. Decreased paranoia and delusions

When a patient is receiving a second-generation antipsychotic drug, such as risperidone (Risperdal), the nurse will monitor for which therapeutic effect? a. Decreased paranoia and delusions b. Fewer panic attacks c. Improved tardive dyskinesia d. Decreased feeling of hopelessness

c. Intake, output, and daily weight

When a patient is receiving diuretic therapy, which of these assessment measures would best reflect the patient's fluid volume status? a. Measurements of abdominal girth and calf circumference b. Blood pressure and pulse c. Intake, output, and daily weight d. Serum potassium and sodium levels

d. Use artificial saliva, sugarless gum, or hard candy to counteract dry mouth.

When a patient is taking an anticholinergic such as benztropine (Cogentin) as part of the treatment for Parkinson's disease, the nurse should include which information in the teaching plan? a. Discontinue the medication if adverse effects occur. b. Minimize the amount of fluid taken while on this drug. c. Take the medication on an empty stomach to enhance absorption. d. Use artificial saliva, sugarless gum, or hard candy to counteract dry mouth.

b Wear gloves to place the tablet under the patient's tongue.

When administering a sublingual medication, which action will the nurse perform? a Ask the patient to chew the medication so that it will absorb faster. b Wear gloves to place the tablet under the patient's tongue. c Instruct the patient to drink room temperature water to enhance dissolution. d Have the patient swallow several times while the tablet dissolves.

B Flush the line with normal saline before and after administration to prevent precipitation.

When administering intravenous phenytoin (Dilantin), which action will the nurse perform? A Always use an infusion pump. B Flush the line with normal saline before and after administration to prevent precipitation. C Administer through peripheral intravenous sites only. D Monitor for hypertension.

B. Hypotension

When administering nitroprusside (Nipride) intravenously, the nurse would monitor for which sign of toxicity? A. Fever B. Hypotension C. Extreme fatigue D. Hyperglycemia

d Ask the patient and/or family to bring in all medications the patient was taking at home.

When admitting an elderly patient to an acute care setting, which nursing strategy is most appropriate to prevent medication errors? a Call the primary care physician to verify current medications. b Ask the patient's family to verify medications the patient was taking at home. c Ask the patient to provide you with a written list of all medications being taken at home. d Ask the patient and/or family to bring in all medications the patient was taking at home.

C. Hyperkalemia

When assessing a patient taking triamterene (Dyrenium), the nurse would specifically monitor for which adverse effect? A. Hypokalemia B. Hypoglycemia C. Hyperkalemia D. Hypernatremia

b, c, e

When assessing a patient who is receiving a loop diuretic, the nurse looks for the manifestations of potassium deficiency, which would include what symptoms? (Select all that apply.) a. Dyspnea b. Muscle weakness c. Lethargy d. Tinnitus e. Anorexia f. Constipation

A a positive inotropic, positive chronotropic, and positive dromotropic effect.

When assessing for cardiovascular effects of an adrenergic (sympathomimetic) drug, the nurse understands that these drugs produce A a positive inotropic, positive chronotropic, and positive dromotropic effect. B a positive inotropic, negative chronotropic, and negative dromotropic effect. C a negative inotropic, positive chronotropic, and positive dromotropic effect. D a negative inotropic, negative chronotropic, and negative dromotropic effect.

A Insomnia

When assessing for side effects expected in a patient taking analeptics, the nurse would monitor for which effect? A Insomnia B Bradycardia C Hypotension D Decreased mental alertness

d Dosage calculation by body surface area is the most accurate method because it takes into account the difference in size of the child and/or neonate.

When calculating pediatric dosages, what will the nurse take into consideration? a Utilization of drug reference recommendations based on mg/kg of body weight is the preferred method. b Calculated doses based on body weight need to be increased by 10% due to immature renal and hepatic function. c Dosage calculation according to body weight is the most accurate method because it takes into account differences in maturational development. d Dosage calculation by body surface area is the most accurate method because it takes into account the difference in size of the child and/or neonate.

B. glipizide (Glucotrol)

When caring for a patient newly diagnosed with gestational diabetes, the nurse would question an order for which drug? A. insulin glargine (Lantus) B. glipizide (Glucotrol) C. insulin glulisine (Apidra) D. NPH insulin

C. Bronchial asthma

When caring for a patient with angina, the nurse would question an order for a noncardioselective beta blocker in a patient with what coexisting medical diagnosis? A. Hypertension B. Atrial fibrillation C. Bronchial asthma D. Myocardial infarction

C. "The different color pills are due to different amounts of hormones in each week."

When educating a patient about the use of oral contraceptives, the nurse provides what explanation for the pills having different colors each week? A. "They help you remember which week you are taking." B. "There isn't a reason for the color, just the choice of the drug company." C. "The different color pills are due to different amounts of hormones in each week." D. "They are color coded for the weeks of the month."

A. Decreased intracranial pressure

When evaluating for therapeutic effects of mannitol, what does the nurse anticipate? A. Decreased intracranial pressure B. Decreased excretion of therapeutic medications C. Increased urine osmolality D. Decreased serum osmolality

a. Elevated blood glucose

When monitoring a patient who has diabetes and is receiving a carbonic anhydrase inhibitor for edema, the nurse will monitor for which possible adverse effect? a. Elevated blood glucose b. Mental alertness c. Hyperkalemia d. Metabolic alkalosis

d. Digitalis

When monitoring a patient who is taking hydrochlorothiazide (HydroDIURIL), the nurse notes that which drug is most likely to cause a severe interaction with the diuretic? a. Penicillin b. Aspirin c. Potassium supplements d. Digitalis

C. different vomiting pathways are blocked.

When planning administration of antiemetic medications to a patient, the nurse is aware that combination therapy is preferred because A. the risk of constipation is decreased. B. lower doses of medication are cost-effective. C. different vomiting pathways are blocked. D. increased sedation is achieved by higher doses of medication.

a Completing a medication reconciliation between units

When receiving a patient transferred from another unit, which action is most useful to prevent medication errors? a Completing a medication reconciliation between units b Participating in a verbal report from the transferring nurse c Asking the patient what medications were received upon transfer d Asking the physician to rewrite all medication orders upon transfer

c. Decreased white blood cell count

When reviewing the laboratory values of a patient who is taking antithyroid drugs, the nurse will monitor for which adverse effect? a. Decreased glucose levels b. Increased platelet count c. Decreased white blood cell count d. Increased red blood cell count

a, c, f

When teaching a patient who will be receiving antihistamines, the nurse will include which instructions? (Select all that apply.) a. "Drink extra fluids if possible." b. "Take the medication on an empty stomach to maximize absorption of the drug." c. "Take the medication with food to minimize gastrointestinal distress." d. "Antihistamines are generally safe to take with over-the-counter medications." e. "Antihistamines may cause restlessness and disturbed sleep." f. "Avoid activities that require alertness until you know how adverse effects are tolerated."

b Encourage the patient to question medications if the medications are different than he or she expects.

Which action assists the nurse in prevention of a potential medication error? a Ask the patient what disease the medication is for before administering. b Encourage the patient to question medications if the medications are different than he or she expects. c Allow the patient to take home medications when desired. d Administer the patient's medications from his or her personal bottles while in the hospital.

C selegiline

Which antiparkinson drug causes an increase in the levels of dopaminergic stimulation in the central nervous system and therefore allows a decreased dose of other medications? A levodopa B carbidopa C selegiline D diphenhydramine

b, c, d

Which are mechanisms of action of biologic response-modifying drugs? (Select all that apply.) A. Interruption of the cell lifecycle at the genetic level to interfere with growth of the cancer cell B. Direct toxic effect on tumor cells, causing them to rupture C. Adverse modification of a tumor's biology, making it harder for the tumor cells to survive and reproduce D. Enhancement or restoration of the host's immune system defenses against a tumor

a, c

Which are nursing responsibilities associated with the care of a patient receiving total parenteral nutrition (TPN)? (Select all that apply.) A. Monitoring capillary serum glucose every 6 hours B. Protecting the solution from exposure to light at all times C. Maintaining sterile asepsis when handling the site or solution D. Accelerating the rate of infusion to keep the infusion on time as needed

c. The dorsogluteal region is no longer recommended for injection; select a different site.

Which are the appropriate landmarks for an intramuscular injection into the dorsogluteal region? a. A "V" formation between the anterior superior iliac spine and the greater trochanter b. A handbreadth below the greater trochanter and the sacroiliac joint c. The dorsogluteal region is no longer recommended for injection; select a different site. d. Below the iliac spine and between the greater trochanter and the iliac crest

A. Positive inotropic, negative chronotropic, and negative dromotropic

Which are therapeutic effects of digoxin? A. Positive inotropic, negative chronotropic, and negative dromotropic B. Positive inotropic, positive chronotropic, and negative dromotropic C. Negative inotropic, negative chronotropic, and negative dromotropic D. Positive inotropic, negative chronotropic, and positive dromotropic

B. Calcium channel blockers

Which class of drugs is used to treat both hypertension and dysrhythmias? A. Sodium channel blockers B. Calcium channel blockers C. Angiotensin-converting enzyme (ACE) inhibitors D. Direct-acting vasodilators

A. Allergies

Which information is of highest priority when obtaining a pharmacologic history from a patient? A. Allergies B. Use of over-the-counter medications C. Home remedy use D. Alcohol intake

a, b, d

Which information should be included in a teaching plan for patients taking oral hypoglycemic drugs? (Select all that apply.) A. Limit your alcohol consumption. B. Report symptoms of anorexia and fatigue. C. Take your medication only as needed. D. Notify your physician if blood glucose levels rise above the level set for you.

A. "Apply the patch to a nonhairy, nonirritated area of the upper torso or arms."

Which instruction should be included in the discharge teaching for a patient with a transdermal nitroglycerin patch? A. "Apply the patch to a nonhairy, nonirritated area of the upper torso or arms." B. "Apply the patch to the same site each day to maintain consistent drug absorption." C. "If you get a headache, remove the patch for 4 hours and then reapply." D. "If you get chest pain, apply a second patch right next to the first patch."

C. Excess fluid volume

Which is a priority nursing diagnosis for a patient receiving desmopressin (DDAVP)? A. Risk for injury B. Acute pain C. Excess fluid volume D. Deficient knowledge regarding medication

A. Ineffective cerebral tissue perfusion related to disease process and/or medication

Which is a priority nursing diagnosis for a patient taking an antihypertensive medication? A. Ineffective cerebral tissue perfusion related to disease process and/or medication B. Deficient knowledge related to medication regimen C. Fatigue related to side effects of medication D. Risk for injury to mucous membranes related to medication side effects

B. oxytocin

Which is the most common drug used for induction of labor in pregnant women and to promote ejection of milk during lactation? A. clomiphene B. oxytocin C. misoprostol D. mifepristone

c. Benzathine penicillin G, an antibiotuc, IM one time only

Which is the preferred treatment for the diagnosis of syphilis in a teenage client? a. Nitrofurantoin, a macrolide, PO BID for 1 month b. Doxycycline, a tetracycline, po every 4 hours for 10 days c. Benzathine penicillin G, an antibiotuc, IM one time only d. Miconazole, and antifungal, topical for one week

B. Liver function tests

Which laboratory test would the nurse expect to be ordered to monitor a patient for adverse effects related to progestin medications? A. Cardiac enzymes B. Liver function tests C. BUN and creatinine D. Complete blood count

A. insulin glargine (Lantus)

Which long-acting insulin mimics natural, basal insulin with no peak action and a duration of 24 hours? A. insulin glargine (Lantus) B. insulin glulisine (Apidra) C. regular insulin (Humulin R) D. NPH insulin

a naloxone (Narcan)

Which medication is used to treat a patient suffering from severe adverse effects of a narcotic analgesic? a naloxone (Narcan) b acetylcysteine (Mucomyst) c methylprednisolone (Solu-Medrol) d flumazenil (Romazicon)

D. potassium

Which medication would the nurse question if prescribed concurrently with ACE inhibitors? A. furosemide (Lasix) B. morphine C. Colace D. potassium

D. Transdermal nitroglycerin patch

Which nitrate preparation or dosage form has the longest duration of action? A. Sublingual nitroglycerin B. Sublingual isosorbide dinitrate C. Oral isosorbide dinitrate D. Transdermal nitroglycerin patch

c Crush and administer each medication separately.

Which nursing intervention is most appropriate when crushing oral medications to administer to a patient with dysphagia? a Only crush enteric-coated medication. b Mix medications together for ease of administration. c Crush and administer each medication separately. d Open sustained-release capsules prior to crushing.

Morphine

Which of the following drugs has the greatest efficacy in relieving severe pain, aspirin or morphine?

C. repaglinide (Prandin)

Which oral hypoglycemic drug has a quick onset and short duration of action, enabling the patient to take the medication 30 minutes before eating and skip the dose if he or she does not eat? A. acarbose (Precose) B. metformin (Glucophage) C. repaglinide (Prandin) D. pioglitazone (Actos)

B. "This drug will be taken up by the thyroid gland and destroy the cells to reduce my hyperthyroidism."

Which patient statement demonstrates understanding of radioactive iodine (I-131) therapy? A. "I will have to isolate myself from my family for 1 week so as not to expose them to radiation." B. "This drug will be taken up by the thyroid gland and destroy the cells to reduce my hyperthyroidism." C. "This drug will help decrease my cold intolerance and weight gain." D. "I will need to take this drug on a daily basis for at least 1 year."

b Turning, coughing, and deep breathing every 2 hours

Which postoperative nursing action will help the patient recover from the effects of anesthesia? a Ambulating once a day b Turning, coughing, and deep breathing every 2 hours c Forcing fluids to 400 mL per shift d Administering loperamide (Imodium) as needed

FDA

Which regulatory agency in the U. S. must approve a drug before it can be marketed, determines if a drug is safe and effective, and determines whether a drug should be OTC or require a prescription?

B. "I will rinse my mouth with water after each use."

Which statement by a patient best demonstrates an understanding of the teaching on flunisolide (AeroBid)? A. "I will take two puffs to treat an acute asthma attack." B. "I will rinse my mouth with water after each use." C. "I will immediately stop taking my oral prednisone as soon as I start using the AeroBid." D. "I will not use my albuterol inhaler while I am taking AeroBid."

D. "I can take up to four tablets at 5-minute intervals for chest pain."

Which statement by the patient demonstrates a need for further education regarding nitroglycerin? A. "If I get a headache, I should keep taking my nitroglycerin and use Tylenol for pain relief." B. "I should keep my nitroglycerin in a cool, dry place." C. "I should change positions slowly to avoid getting dizzy from the nitroglycerin's effect on my blood pressure." D. "I can take up to four tablets at 5-minute intervals for chest pain."

A. "I will take this medication in the morning so as not to interfere with sleep."

Which statement by the patient demonstrates an understanding of discharge instructions on the use of levothyroxine (Synthroid)? A. "I will take this medication in the morning so as not to interfere with sleep." B. "I will double my dose if I gain more than 1 pound per day." C. "I will stop the medication immediately if I lose more than 2 pounds in a week." D. "I can expect to see relief of my symptoms within 1 week."

B CNS stimulants increase release of and block reuptake of neurotransmitters.

Which statement correctly identifies the pharmacodynamics of CNS stimulants? A CNS stimulants decrease the production of excitatory neurotransmitters. B CNS stimulants increase release of and block reuptake of neurotransmitters. C CNS stimulants block the activity of inhibitory neurons. D CNS stimulants enhance the effects of phosphodiesterase and subsequent breakdown of cyclic adenosine monophosphate (cAMP).

a. Centrally acting agents inhibit neurons of the central nervous system, while direct-acting agents work at the level of the neuromuscular junction and skeletal muscles.

Which statement describes the primary difference between centrally acting muscle relaxants and direct-acting antispasmodics? a. Centrally acting agents inhibit neurons of the central nervous system, while direct-acting agents work at the level of the neuromuscular junction and skeletal muscles. b. Centrally acting agents stimulate neurons of the central nervous system, while direct-acting agents stimulate neurons of the peripheral nervous system. c. Centrally acting agents stimulate the central nervous system, while direct-acting agents inhibit neuronal conduction of the central nervous system. d. Centrally acting agents inhibit neurons of the central nervous system, while direct-acting agents stimulate neurons of central nervous system

D) "Over-the-counter medications can, at times, be used in place of prescription drugs. It is important to discuss the use of these with your health care provider."

Which statement is accurate when discussing self-treatment options with a patient? A) "Over-the-counter medications are not as potent as prescription drugs." B) "The use of over-the-counter medications is gradually decreasing with the increased availability of more effective prescription medications." C) "Herbal remedies have not demonstrated any adverse effects with their use." D) "Over-the-counter medications can, at times, be used in place of prescription drugs. It is important to discuss the use of these with your health care provider."

C. "I will apply the scopolamine patches to rotating sites on my arms."

Which statement regarding antiemetic medications, if made by the patient, indicates the need for further patient teaching? A. "I will not drive while I am taking these medications because they may cause drowsiness." B. "I may take Tylenol to treat the headache caused by ondansetron." C. "I will apply the scopolamine patches to rotating sites on my arms." D. "I should take my prescribed antiemetic before receiving my chemotherapy dose."

B. "The patch should be applied to a nonhairy site, and abrupt withdrawal should be avoided."

Which statement would be most appropriate during discharge teaching for a patient receiving transdermal clonidine (Catapres)? A. "Your blood pressure should be checked by a physician at least once a year." B. "The patch should be applied to a nonhairy site, and abrupt withdrawal should be avoided." C. "Excessive exercise or prolonged standing is not a problem with clonidine as it can be with other antihypertensive drugs." D. "If you are having difficulty with the common side effect of excessive drooling, notify your physician so your dosage can be adjusted."

D. "I should mix and stir the powder in as small an amount of fluid as possible in order to maintain potency of the medication."

Which statement, made by the patient, demonstrates a knowledge deficit regarding colestipol (Colestid)? A. "The medication may cause constipation, so I will increase fluid and fiber in my diet." B. "I should take this medication 1 hour after or 4 hours before my other medications." C. "I might need to take fat-soluble vitamins to supplement my diet." D. "I should mix and stir the powder in as small an amount of fluid as possible in order to maintain potency of the medication."

b. Malnourished patients receiving parenteral nutrition are at risk for hypophosphatemia if calories are started too aggressively.

You are caring for a patient with a diagnosis of pancreatitis. The patient was admitted from a homeless shelter and is a poor historian. The patient appears malnourished and TPN has been started. Why would you know to start the TPN slowly? a. Patients receiving TPN are at risk for hypercalcemia if calories are started too rapidly. b. Malnourished patients receiving parenteral nutrition are at risk for hypophosphatemia if calories are started too aggressively. c. Malnourished patients who receive fluids too rapidly are at risk for hypernatremia. d. Patients receiving TPN are at risk for hypochloremia if fluids are introduced too rapidly.

d. Shallow respirations

You are caring for a patient with a secondary diagnosis of hypermagnesemia. What would you assess this patient for? a. Hypertension b. Kussmaul respirations c. Increased DTRs d. Shallow respirations

a. Risk for infection related to the presence of a subclavian catheter

You are caring for a patient with a subclavian catheter who is receiving parenteral nutrition (PN). In preparing a care plan for this patient, what nursing diagnosis will you give highest priority to? a. Risk for infection related to the presence of a subclavian catheter b. Risk for caregiver role strain related to the care of a subclavian catheter c. Risk for activity intolerance related to the presence of a subclavian catheter d. Risk for loneliness related to need for isolation related to the presence of a subclavian catheter

a, b, e

You are doing discharge teaching with a patient who is going home with a diagnosis of hypophosphatemia. The patient has a diet ordered that is high in phosphate. What foods would you teach this patient to include in his diet? (Mark all that apply.) a. Poultry b. Liver c. Green vegetables d. Beef e. Milk

b. Hypocalcemia

You are the nurse caring for a 65-year-old female patient who is in renal failure. During your shift assessment, the patient complains of tingling in her lips and fingers whenever anyone takes her blood pressure. She tells you that she gets a spasm in her wrist and hand and that it is very painful. What would you suspect? a. Hypophosphatemia b. Hypocalcemia c. Hypermagnesemia d. Hyperkalemia

a, b, c

Your patient is receiving dobutamine as a continuous infusion. Titration of this medication is based upon which factors? (Select all that apply.) A Heart rate B Blood pressure C Urine output D Liver enzymes E Respiratory rate

B 30 mL/hr

Your patient is to receive dopamine 5 mcg/kg/min. He weighs 176 lb. You have dopamine 400 mg in 500 mL D5W. You will infuse this at which rate? A 20 mL/hr B 30 mL/hr C 35 mL/hr D 50 mL/hr

Nonbenzodiazepine hypnotics (drugs, mechanism, use, toxicity)

Zolpidem, Zaleplon, esZopiclone. "All ZZZs put you to sleep." -Mechanism: act at BZI subtype of the GABA receptor; -Use: insomnia -Toxicity: ataxia, headaches, confusion. Short duration because of rapid metabolism by liver enzymes. Unlike older sedative-hypnotics, cause only modest day-after psychomotor depression and few amnestic effects. Decreased dependence risk than benzodiazepines.

Five half-lives

______ are normally required for a drug to be eliminated from the body.

what is a problem with Long term systematic administration of corticosteroids?

its a example of a drug that can result in decreased resistance to infection with prolong use.

anticholinergic drugs effect on cardiovascular?

large doses causes vagal blockade, resulting in tachycardia. small does causes bradycardia.

Drugs that are the least soluble will have the __________duration of action.

longest

Corticosteroid inhalers can lead to

oral candidiasis (thrush) - patient should rinse mouth after use. Treat thrush with nystatin

Sign of overdose or addiction to opioid

pinpoint pupils

Nonspecific stimulant withdrawal

post-use "crash," including depression, lethargy, weight gain, headache

Tardive dyskinesia

stereotypic oral- facial movements as a result of long-term antipsychotic use.

The adrenal medulla is innervated by ?

sympathetic preganglionic fibers. when the sans in stimulated, the adrenal medulla releases primarily epinephrine. and norepinephrine.

Membranes made of lipids are relatively impermeable to ions and polar molecules? T/F

true.

The less ionized the molecule is the easier it is to transverse the membrane?

true.


Related study sets

Module 7 Income - Assessment(s) Questions

View Set

Ch. 11 Introduction to Entrepreneurship

View Set

Personal Jurisdiction (In personam)

View Set

Continents, Countries, and Oceans

View Set

PCC: ATI/PrepU: Fluid and Electrolyte

View Set